FInal Exam

Ace your homework & exams now with Quizwiz!

13.A nurse teaches a client who is prescribed a central vascular access device. Which statement should the nurse include in this clients teaching? a. You will need to wear a sling on your arm while the device is in place. b. There is no risk of infection because sterile technique will be used during insertion. c. Ask all providers to vigorously clean the connections prior to accessing the device. d. You will not be able to take a bath with this vascular access device.

ANS: C Clients should be actively engaged in the prevention of catheter-related bloodstream infections and taught to remind all providers to perform hand hygiene and vigorously clean connections prior to accessing the device. The other statements are incorrect.

A nurse assesses a client recently bitten by a coral snake. Which assessment should the nurse complete first? a. Unilateral peripheral swelling b. Clotting times c. Cardiopulmonary status d. Electrocardiogram rhythm

ANS: C Manifestations of coral snake envenomation are the result of its neurotoxic properties. The physiologic effect is to block neurotransmission, which produces ascending paralysis, reduced perception of pain, and, ultimately, respiratory paralysis. The nurse should monitor for respiratory rate and depth. Severe swelling and clotting problems do not occur with coral snakes but do occur with pit viper snakes. Electrocardiogram rhythm is not affected by neurotoxins.

A nurse is assessing a client with peripheral artery disease ( PAD ). The client states walking five blocks is possible without pain. What question asked next by the nurse will give the best information? a. "Could you walk further than that a few months ago?" b. "Do you walk mostly uphill, downhill, or on flat surfaces?" c. "Have you ever considered swimming instead of walking?" d. "How much pain medication do you take each day?"

" Could you walk further than that a few months ago? " As PAD progresses, it takes less oxygen demand to cause pain. Needing to cut down on activity to be pain free indicates the client's disease is worsening. The other questions are useful, but not as important.

After teaching a client how to prevent altitude-related illnesses, a nurse assesses the clients understanding. Which statement indicates the client needs additional teaching? a. If my climbing partner cant think straight, we should descend to a lower altitude. b. I will ask my provider about medications to help prevent acute mountain sickness. c. My partner and I will plan to sleep at a higher elevation to acclimate more quickly. d. I will drink plenty of fluids to stay hydrated while on the mountain.

ANS: C Teaching to prevent altitude-related illness should include descending when symptoms start, staying hydrated, and taking acetazolamide (Diamox), which is commonly used to prevent and treat acute mountain sickness. The client should be taught to sleep at a lower elevation.

What is the dosage limit of acetaminophen in the treatment of osteoarthritis (OA)?___ mg

3000 mg/day Acetaminophen may cause liver disorders if the patient takes more than 3000 mg daily of acetaminophen. Liver enzymes must be monitored in patients with OA who are regularly taking acetaminophen.

A nurse is caring for a client who weighs 22o pounds and is started on enoxaparin ( Lovenox ). How much enoxaparin does the nurse anticipate administering?

90 mg 90 mg The dose of enoxaparin is 1 mg/kg body weight, not to exceed 90 mg. This client weighs 220 pounds (110 kg), and so will get the maximal dose

A nurse assesses a client who is recovering from a paracentesis 1 hour ago. Which assessment finding requires action by the nurse? a. Urine output via indwelling urinary catheter is 20 mL/hr b. Blood pressure increases from 110/58 to 120/62 mm Hg c. Respiratory rate decreases from 18 to 14 breaths/min d. A decrease in the clients weight by 6 kg

A

A nurse assesses clients at a community health fair. Which client is at greatest risk for the development of hepatitis B? a. A 20-year-old college student who has had several sexual partners b. A 46-year-old woman who takes acetaminophen daily for headaches c. A 63-year-old businessman who travels frequently across the country d. An 82-year-old woman who recently ate raw shellfish for dinner

A

A nurse cares for a client who is prescribed lactulose (Heptalac). The client states, I do not want to take this medication because it causes diarrhea. How should the nurse respond? a. Diarrhea is expected; thats how your body gets rid of ammonia. b. You may take Kaopectate liquid daily for loose stools c. Do not take any more of the medication until your stools firm up d. We will need to send a stool specimen to the laboratory

A

Which of these findings causes the critical care nurse to notify the primary care provider (PCP) for evaluation for intubation? A.Increasing somnolence B.Pallor C.Deep respirations D.Bounding pulse

A (The critical nurse notifies the primary health care provider for somnolence consistent with worsening respiratory acidosis. Other client findings related to worsening respiratory acidosis caused by CO2 retention include: headache, fatigue, lethargy, and decreased respirations which may require intubation and mechanical ventilation.Pallor is a sign of hypoxemia, lack of oxygen to the tissues. As pallor may occur with anemia, this finding alone does not represent a need for intubation. Deep respirations and bounding pulse are not consistent with respiratory acidosis.)

25.A nurse assesses a client with diabetes mellitus 3 hours after a surgical procedure and notes the clients breath has a fruity odor. Which action should the nurse take? a. Encourage the client to use an incentive spirometer. b. Increase the clients intravenous fluid flow rate. c. Consult the provider to test for ketoacidosis. d. Perform meticulous pulmonary hygiene care.

ANS: C The stress of surgery increases the action of counterregulatory hormones and suppresses the action of insulin, predisposing the client to ketoacidosis and metabolic acidosis. One manifestation of ketoacidosis is a fruity odor to the breath. Documentation should occur after all assessments have been completed. Using an incentive spirometer, increasing IV fluids, and performing pulmonary hygiene will not address this clients problem.

A nurse in a family practice clinic is preparing discharge instructions for a client reporting facial pain that is worse when bending over, tenderness across the cheeks, and postnasal discharge. What instruction will be most helpful? a. "Ice packs may help with the facial pain." b. "Limit fluids to dry out your sinuses." c. "Try warm, moist heat packs on your face." d. "We will schedule you for a computed tomography scan this week."

ANS: C This client has rhinosinusitis. Comfort measures for this condition include breathing in warm steam, hot packs, nasal saline irrigations, sleeping with the head elevated, increased fluids, and avoiding cigarette smoke. The client does not need a CT scan.

A nurse auscultates a harsh hollow sound over a clients trachea and larynx. Which action should the nurse take first? a. Document the findings. b. Administer oxygen therapy. c. Position the client in high-Fowlers position. d. Administer prescribed albuterol.

A Bronchial breath sounds, including harsh, hollow, tubular, and blowing sounds, are a normal finding over the trachea and larynx. The nurse should document this finding. There is no need to implement oxygen therapy, administer albuterol, or change the clients position because the finding is normal.

ANS: B Social anxiety and apprehension are common in clients with a new ileostomy. The nurse should encourage the client to discuss concerns. The nurse should not minimize the client's concerns or provide false reassurance.

A nurse cares for a client with a new ileostomy. The client states, "I don't think my friends will accept me with this ostomy." How should the nurse respond? a. "Your friends will be happy that you are alive." b. "Tell me more about your concerns." c. "A therapist can help you resolve your concerns." d. "With time you will accept your new body."

ANS: A Clients with ulcerative colitis often express that the disorder is disruptive to their lives. Stress factors can increase symptoms. These factors should be identified so that the client will have more control over his or her condition. Prescription medications and anorexia will not eliminate exacerbations. Although a therapist may assist the client, this is not an appropriate response.

A nurse cares for a client with ulcerative colitis. The client states, "I feel like I am tied to the toilet. This disease is controlling my life." How should the nurse respond? a. "Let's discuss potential factors that increase your symptoms." b. "If you take the prescribed medications, you will no longer have diarrhea." c. "To decrease distress, do not eat anything before you go out." d. "You must retake control of your life. I will consult a therapist to help."

ANS: B Protecting the client's skin is the priority action for a client who has a heavily draining fistula. Intestinal fluid enzymes are caustic and can cause skin breakdown or fungal infections if the skin is not protected. The plan of care for a client who has Crohn's disease includes adequate nutrition focused on high-calorie, high-protein, high-vitamin, and low-fiber meals, antibiotic administration, and glucocorticoids.

A nurse plans care for a client with Crohn's disease who has a heavily draining fistula. Which intervention should the nurse indicate as the priority action in this client's plan of care? a. Low-fiber diet b. Skin protection c. Antibiotic administration d. Intravenous glucocorticoids

ANS: A Fistulas place the client with Crohn's disease at risk for hypokalemia which can lead to serious dysrhythmias. This potassium level is low and should cause the nurse to intervene. The white blood cell count is normal. The other two findings are abnormal and also warrant intervention, but the potassium level takes priority.

A nurse reviews the chart of a client who has Crohn's disease and a draining fistula. Which documentation should alert the nurse to urgently contact the provider for additional prescriptions? a. Serum potassium of 2.6 mEq/L b. Client ate 20% of breakfast meal c. White blood cell count of 8200/mm3 d. Client's weight decreased by 3 pounds

A nurse collaborates with a respiratory therapist to complete pulmonary function tests (PFTS) for a client. Which statements should the nurse include in communications with the respiratory therapist prior to the tests? (Select all that apply.) a. I held the clients morning bronchodilator medication. b. The client is ready to go down to radiology for this examination c. Physical therapy states the client can run on a treadmill d. I advised the client not to smoke for 6 hours prior to the test. e. The client is alert and can follow your commands.

A, D, E To ensure the PFTs are accurate, the therapist needs to know that no bronchodilators have been administered in the past 4 to 6 hours, the client did not smoke within 6 to 8 hours prior to the test, and the client can follow basic commands, including different breathing maneuvers. The respiratory therapist can perform PFTs at the bedside. A treadmill is not used for this test.

A nurse teaches a client who is diagnosed with diabetes mellitus. Which statement should the nurse include in this clients plan of care to delay the onset of microvascular and macrovascular complications? a. Maintain tight glycemic control and prevent hyperglycemia. b. Restrict your fluid intake to no more than 2 liters a day. c. Prevent hypoglycemia by eating a bedtime snack. d. Limit your intake of protein to prevent ketoacidosis.

ANS: A Hyperglycemia is a critical factor in the pathogenesis of long-term diabetic complications. Maintaining tight glycemic control will help delay the onset of complications. Restricting fluid intake is not part of the treatment plan for clients with diabetes. Preventing hypoglycemia and ketosis, although important, are not as important as maintaining daily glycemic control.

An emergency department nurse assesses a client with ketoacidosis. Which clinical manifestation should the nurse correlate with this condition? a. Increased rate and depth of respiration b. Extremity tremors followed by seizure activity c. Oral temperature of 102 F (38.9 C) d. Severe orthostatic hypotension

ANS: A Ketoacidosis decreases the pH of the blood, stimulating the respiratory control areas of the brain to buffer the effects of increasing acidosis. The rate and depth of respiration are increased (Kussmaul respirations) in an attempt to excrete more acids by exhalation. Tremors, elevated temperature, and orthostatic hypotension are not associated with ketoacidosis.

A nurse cares for a client who has a family history of diabetes mellitus. The client states, My father has type 1 diabetes mellitus. Will I develop this disease as well? How should the nurse respond? a. Your risk of diabetes is higher than the general population, but it may not occur. b. No genetic risk is associated with the development of type 1 diabetes mellitus. c. The risk for becoming a diabetic is 50% because of how it is inherited. d. Female children do not inherit diabetes mellitus, but male children will.

ANS: A Risk for type 1 diabetes is determined by inheritance of genes coding for HLA-DR and HLA-DQ tissue types. Clients who have one parent with type 1 diabetes are at increased risk for its development. Diabetes (type 1) seems to require interaction between inherited risk and environmental factors, so not everyone with these genes develops diabetes. The other statements are not accurate.

7. A nurse cares for a client who is experiencing epistaxis. Which action should the nurse take first? a. Initiate Standard Precautions. b. Apply direct pressure. c. Sit the client upright. d. Loosely pack the nares with gauze.

ANS: A The nurse should implement Standard Precautions and don gloves prior to completing the other actions.

12. A nurse is caring for four clients in the neurologic intensive care unit. After receiving the hand-off report, which client should the nurse see first? a. Client with a Glasgow Coma Scale score that was 10 and is now is 8 b. Client with a Glasgow Coma Scale score that was 9 and is now is 12 c. Client with a moderate brain injury who is amnesic for the event d. Client who is requesting pain medication for a headache

ANS: A A 2-point decrease in the Glasgow Coma Scale score is clinically significant and the nurse needs to see this client first. An improvement in the score is a good sign. Amnesia is an expected finding with brain injuries, so this client is lower priority. The client requesting pain medication should be seen after the one with the declining Glasgow Coma Scale score.

A nurse in the family clinic is teaching a client newly diagnosed with osteoarthritis (OA) about drugs used to treat the disease. For which medication does the nurse plan primary teaching? a. Acetaminophen (Tylenol) b. Cyclobenzaprine hydrochloride (Flexeril) c. Hyaluronate (Hyalgan) d. Ibuprofen (Motrin)

ANS: A All of the drugs are appropriate to treat OA. However, the first-line drug is acetaminophen. Cyclobenzaprine is a muscle relaxant given to treat muscle spasms. Hyaluronate is a synthetic joint fluid implant. Ibuprofen is a nonsteroidal anti-inflammatory drug.

A nurse assesses a client who is recovering from a Whipple procedure. Which assessment finding alerts the nurse to urgently contact the health care provider? a. Drainage from a fistula b. Absent bowel sounds c. Pain at the incision site d. Nasogastric (NG) tube drainage

ANS: A Complications of a Whipple procedure include secretions that drain from a fistula and peritonitis. Absent bowel sounds, pain at the incision site, and NG tube drainage are normal postoperative findings.

A nurse cares for a client with right-sided heart failure. The client asks, "Why do I need to weigh myself every day?" How should the nurse respond? a. "Weight is the best indication that you are gaining or losing fluid." b. "Daily weights will help us make sure that you're eating properly." c. "The hospital requires that all inpatients be weighed daily." d. "You need to lose weight to decrease the incidence of heart failure."

ANS: A Daily weights are needed to document fluid retention or fluid loss. One liter of fluid equals 2.2 pounds. The other responses do not address the importance of monitoring fluid retention or loss.

9. A client with a stroke has damage to Brocas area. What intervention to promote communication is best for this client? a. Assess whether or not the client can write. b. Communicate using yes-or-no questions. c. Reinforce speech therapy exercises. d. Remind the client not to use neologisms.

ANS: A Damage to Brocas area often leads to expressive aphasia, wherein the client can understand what is said but cannot express thoughts verbally. In some instances the client can write. The nurse should assess to see if that ability is intact. Yes-or-no questions are not good for this type of client because he or she will often answer automatically but incorrectly. Reinforcing speech therapy exercises is good for all clients with communication difficulties. Neologisms are made-up words often used by clients with sensory aphasia.

A client is started on etanercept (Enbrel). What teaching by the nurse is most appropriate? a. Giving subcutaneous injections b. Having a chest x-ray once a year c. Taking the medication with food d. Using heat on the injection site

ANS: A Etanercept is given as a subcutaneous injection twice a week. The nurse should teach the client how to self-administer the medication. The other options are not appropriate for etanercept.

The nurse is teaching a client with chronic kidney disease (CKD) about the sodium restriction needed in the diet to prevent edema and hypertension. Which statement by the client indicates more teaching is needed? a. "I am thrilled that I can continue to eat fast food." b. "I will cut out bacon with my eggs every morning." c. "My cooking style will change by not adding salt." d. "I will probably lose weight by cutting out potato chips."

ANS: A Fast food restaurants usually serve food that is high in sodium. This statement indicates that more teaching needs to occur. The other statements show a correct understanding of the teaching.

A client recently diagnosed with systemic lupus erythematosus (SLE) is in the clinic for a follow-up visit. The nurse evaluates that the client practices good self-care when the client makes which statement? a. "I always wear long sleeves, pants, and a hat when outdoors." b. "I try not to use cosmetics that contain any type of sunblock." c. "Since I tend to sweat a lot, I use a lot of baby powder." d. "Since I can't be exposed to the sun, I have been using a tanning bed."

ANS: A Good self-management of the skin in SLE includes protecting the skin from sun exposure, using sunblock, avoiding drying agents such as powder, and avoiding tanning beds.

A client comes to the family medicine clinic and reports joint pain and stiffness. The nurse is asked to assess the client for Heberden's nodules. What assessment technique is correct? a. Inspect the client's distal finger joints. b. Palpate the client's abdomen for tenderness. c. Palpate the client's upper body lymph nodes. d. Perform range of motion on the client's wrists.

ANS: A Herberden's nodules are seen in osteoarthritis and are bony nodules at the distal interphalangeal joints. To assess for this finding, the nurse inspects the client's distal fingertips. These nodules are not found in the abdomen, lymph nodes, or wrists.

After delegating to an unlicensed assistive personnel (UAP) the task of completing a bladder scan examination for a client, the nurse evaluates the UAP's performance. Which action by the UAP indicates the nurse must provide additional instructions when delegating this task? a. Selecting the female icon for all female clients and male icon for all male clients b. Telling the client, "This test measures the amount of urine in your bladder." c. Applying ultrasound gel to the scanning head and removing it when finished d. Taking at least two readings using the aiming icon to place the scanning head

ANS: A The UAP should use the female icon for women who have not had a hysterectomy. This allows the scanner to subtract the volume of the uterus from readings. If a woman has had a hysterectomy, the UAP should choose the male icon. The UAP should explain the procedure to the client, apply gel to the scanning head and clean it after use, and take at least two readings.

A nurse assesses a client who has aortic regurgitation. In which location in the illustration shown below should the nurse auscultate to best hear a cardiac murmur related to aortic regurgitation? a. Location A b. Location B c. Location C d. Location D

ANS: A The aortic valve is auscultated in the second intercostal space just to the right of the sternum.

A nurse cares for a client with infective endocarditis. Which infection control precautions should the nurse use? a. Standard Precautions b. Bleeding precautions c. Reverse isolation d. Contact isolation

ANS: A The client with infective endocarditis does not pose any specific threat of transmitting the causative organism. Standard Precautions should be used. Bleeding precautions or reverse or contact isolation is not necessary.

29. A client in the emergency department is having a stroke and needs a carotid artery angioplasty with stenting. The clients mental status is deteriorating. What action by the nurse is most appropriate? a. Attempt to find the family to sign a consent. b. Inform the provider that the procedure cannot occur. c. Nothing; no consent is needed in an emergency. d. Sign the consent form for the client.

ANS: A The nurse should attempt to find the family to give consent. If no family is present or can be found, under the principle of emergency consent, a life-saving procedure can be performed without formal consent. The nurse should not just sign the consent form.

20. A nurse assesses a client with the National Institutes of Health (NIH) Stroke Scale and determines the clients score to be 36. How should the nurse plan care for this client? a. The client will need near-total care. b. The client will need cuing only. c. The client will need safety precautions. d. The client will be discharged home.

ANS: A This client has severe neurologic deficits and will need near-total care. Safety precautions are important but do not give a full picture of the clients dependence. The client will need more than cuing to complete tasks. A home discharge may be possible, but this does not help the nurse plan care for a very dependent client.

1.A registered nurse (RN) delegates client care to an experienced licensed practical nurse (LPN). Which standards should guide the RN when delegating aspects of IV therapy to the LPN? (Select all that apply.) a. State Nurse Practice Act b. The facilitys Policies and Procedures manual c. The LPNs level of education and experience d. The Joint Commissions goals and criterion e. Client needs and prescribed orders

ANS: A, B The state Nurse Practice Act will have the information the RN needs, and in some states, LPNs are able to perform specific aspects of IV therapy. However, in a client care situation, it may be difficult and time-consuming to find it and read what LPNs are permitted to do, so another good solution would be for the nurse to check facility policy and follow it.

A nurse is teaching a client about possible complications and hazards of home oxygen therapy. About which complications does the nurse plan to teach the client? (Select all that apply.) a. Absorptive atelectasis b. Combustion c. Dried mucous membranes d. Oxygen-induced hyperventilation e. Toxicity

ANS: A, B, C, E Complications of oxygen therapy include absorptive atelectasis, combustion, dried mucous membranes, and oxygen toxicity. Oxygen-induced hypoventilation is also a complication.

A nurse assesses a client who has had two episodes of bacterial cystitis in the last 6 months. Which questions should the nurse ask? (Select all that apply.) a. "How much water do you drink every day?" b. "Do you take estrogen replacement therapy?" c. "Does anyone in your family have a history of cystitis?" d. "Are you on steroids or other immune-suppressing drugs?" e. "Do you drink grapefruit juice or orange juice daily?"

ANS: A, B, D Fluid intake, estrogen levels, and immune suppression all can increase the chance of recurrent cystitis. Family history is usually insignificant, and cranberry juice, not grapefruit or orange juice, has been found to increase the acidic pH and reduce the risk for bacterial cystitis.

A nurse teaches a client who has severe allergies to prevent bug bites. Which statements should the nurse include in this clients teaching? (Select all that apply.) a. Consult an exterminator to control bugs in and around your home. b. Do not swat at insects or wasps. c. Wear sandals whenever you go outside. d. Keep your prescribed epinephrine auto-injector in a bedside drawer. e. Use screens in your windows and doors to prevent flying insects from entering.

ANS: A, B, E To prevent arthropod bites and stings, clients should wear protective clothing, cover garbage cans, use screens in windows and doors, inspect clothing and shoes before putting them on, consult an exterminator, remove nests, avoid swatting at insects, and carry a prescription epinephrine auto-injector at all times if they are known to be allergic to bee or wasp stings.

A nurse cares for a client who is recovering from a right-sided heart catheterization. For which complications of this procedure should the nurse assess? (Select all that apply.) a. Thrombophlebitis b. Stroke c. Pulmonary embolism d. Myocardial infarction e. Cardiac tamponade

ANS: A, C, E Complications from a right-sided heart catheterization include thrombophlebitis, pulmonary embolism, and vagal response. Cardiac tamponade is a risk of both right- and left-sided heart catheterizations. Stroke and myocardial infarction are complications of left-sided heart catheterizations.

A nurse is caring for a postoperative 70-kg client who had major blood loss during surgery. Which findings by the nurse should prompt immediate action to prevent acute kidney injury? (Select all that apply.) a. Urine output of 100 mL in 4 hours b. Urine output of 500 mL in 12 hours c. Large amount of sediment in the urine d. Amber, odorless urine e. Blood pressure of 90/60 mm Hg

ANS: A, C, E The low urine output, sediment, and blood pressure should be reported to the provider. Postoperatively, the nurse should measure intake and output, check the characteristics of the urine, and report sediment, hematuria, and urine output of less than 0.5 mL/kg/hour for 3 to 4 hours. A urine output of 100 mL is low, but a urine output of 500 mL in 12 hours should be within normal limits. Perfusion to the kidneys is compromised with low blood pressure. The amber odorless urine is normal.

1.A nurse assesses clients at a health fair. Which clients should the nurse counsel to be tested for diabetes? (Select all that apply.) a. 56-year-old African-American male b. Female with a 30-pound weight gain during pregnancy c. Male with a history of pancreatic trauma d. 48-year-old woman with a sedentary lifestyle e. Male with a body mass index greater than 25 kg/m2 f. 28-year-old female who gave birth to a baby weighing 9.2 pounds

ANS: A, D, E, F Risk factors for type 2 diabetes include certain ethnic/racial groups (African Americans, American Indians, Hispanics), obesity and physical inactivity, and giving birth to large babies. Pancreatic trauma and a 30-pound gestational weight gain are not risk factors.

Which teaching point is most important for the client with bacterial pharyngitis? a. Gargle with warm salt water. b. Take all antibiotics as directed. c. Use a humidifier in the bedroom. d. Wash hands frequently.

ANS: B Any client on antibiotics must be instructed to complete the entire course of antibiotics. Not completing them can lead to complications or drug-resistant strains of bacteria. The other instructions are appropriate, just not the most important.

An older adult is brought to the emergency department by a family member, who reports a moderate change in mental status and mild cough. The client is afebrile. The health care provider orders a chest x-ray. The family member questions why this is needed since the manifestations seem so vague. What response by the nurse is best? a. "Chest x-rays are always ordered when we suspect pneumonia." b. "Older people often have vague symptoms, so an x-ray is essential." c. "The x-ray can be done and read before laboratory work is reported." d. "We are testing for any possible source of infection in the client."

ANS: B It is essential to obtain an early chest x-ray in older adults suspected of having pneumonia because symptoms are often vague. Waiting until definitive manifestations are present to obtain the x-ray leads to a costly delay in treatment. Stating that chest x-rays are always ordered does not give the family definitive information. The x-ray can be done while laboratory values are still pending, but this also does not provide specific information about the importance of a chest x-ray in this client. The client has manifestations of pneumonia, so the staff is not testing for any possible source of infection but rather is testing for a suspected disorder.

A nurse teaches a client about self-monitoring of blood glucose levels. Which statement should the nurse include in this clients teaching to prevent bloodborne infections? a. Wash your hands after completing each test. b. Do not share your monitoring equipment. c. Blot excess blood from the strip with a cotton ball. d. Use gloves when monitoring your blood glucose.

ANS: B Small particles of blood can adhere to the monitoring device, and infection can be transported from one user to another. Hepatitis B in particular can survive in a dried state for about a week. The client should be taught to avoid sharing any equipment, including the lancet holder. The client should be taught to wash his or her hands before testing. The client would not need to blot excess blood away from the strip or wear gloves.

3. A nurse is assessing clients on a medical-surgical unit. Which client should the nurse identify as being at greatest risk for atrial fibrillation? a. A 45-year-old who takes an aspirin daily b. A 50-year-old who is post coronary artery bypass graft surgery c. A 78-year-old who had a carotid endarterectomy d. An 80-year-old with chronic obstructive pulmonary disease

ANS: B Atrial fibrillation occurs commonly in clients with cardiac disease and is a common occurrence after coronary artery bypass graft surgery. The other conditions do not place these clients at higher risk for atrial fibrillation.

3. After teaching a client with early polycystic kidney disease (PKD) about nutritional therapy, the nurse assesses the client's understanding. Which statement made by the client indicates a correct understanding of the teaching? a. "I will take a laxative every night before going to bed." b. "I must increase my intake of dietary fiber and fluids." c. "I shall only use salt when I am cooking my own food." d. "I'll eat white bread to minimize gastrointestinal gas."

ANS: B Clients with PKD often have constipation, which can be managed with increased fiber, exercise, and drinking plenty of water. Laxatives should be used cautiously. Clients with PKD should be on a restricted salt diet, which includes not cooking with salt. White bread has a low fiber count and would not be included in a high-fiber diet.

A nurse assesses a client with mitral valve stenosis. What clinical manifestation should alert the nurse to the possibility that the client's stenosis has progressed? a. Oxygen saturation of 92% b. Dyspnea on exertion c. Muted systolic murmur d. Upper extremity weakness

ANS: B Dyspnea on exertion develops as the mitral valvular orifice narrows and pressure in the lungs increases. The other manifestations do not relate to the progression of mitral valve stenosis.

12. A nurse assists with the cardioversion of a client experiencing acute atrial fibrillation. Which action should the nurse take prior to the initiation of cardioversion? a. Administer intravenous adenosine. b. Turn off oxygen therapy. c. Ensure a tongue blade is available. d. Position the client on the left side.

ANS: B For safety during cardioversion, the nurse should turn off any oxygen therapy to prevent fire. The other interventions are not appropriate for a cardioversion. The client should be placed in a supine position.

8. A nurse cares for a client with an intravenous temporary pacemaker for bradycardia. The nurse observes the presence of a pacing spike but no QRS complex on the clients electrocardiogram. Which action should the nurse take next? a. Administer intravenous diltiazem (Cardizem). b. Assess vital signs and level of consciousness. c. Administer sublingual nitroglycerin. d. Assess capillary refill and temperature.

ANS: B In temporary pacing, the wires are threaded onto the epicardial surface of the heart and exit through the chest wall. The pacemaker spike should be followed immediately by a QRS complex. Pacing spikes seen without subsequent QRS complexes imply loss of capture. If there is no capture, then there is no ventricular depolarization and contraction. The nurse should assess for cardiac output via vital signs and level of consciousness. The other interventions would not determine if the client is tolerating the loss of capture.

An emergency room nurse assesses a client after a motor vehicle crash and notes ecchymotic areas across the client's lower abdomen. Which action should the nurse take first? a. Measure the client's abdominal girth. b. Assess for abdominal guarding or rigidity. c. Check the client's hemoglobin and hematocrit. d. Obtain the client's complete health history.

ANS: B On noticing the ecchymotic areas, the nurse should check to see if abdominal guarding or rigidity is present, because this could indicate major organ injury. The nurse should then notify the provider. Measuring abdominal girth or obtaining a complete health history is not appropriate at this time. Laboratory test results can be checked after assessment for abdominal guarding or rigidity.

1. A nurse assesses a client with polycystic kidney disease (PKD). Which assessment finding should alert the nurse to immediately contact the health care provider? a. Flank pain b. Periorbital edema c. Bloody and cloudy urine d. Enlarged abdomen

ANS: B Periorbital edema would not be a finding related to PKD and should be investigated further. Flank pain and a distended or enlarged abdomen occur in PKD because the kidneys enlarge and displace other organs. Urine can be bloody or cloudy as a result of cyst rupture or infection.

20. A nurse cares for a client with a fractured fibula. Which assessment should alert the nurse to take immediate action? a. Pain of 4 on a scale of 0 to 10 b. Numbness in the extremity c. Swollen extremity at the injury site d. Feeling cold while lying in bed

ANS: B The client with numbness and/or tingling of the extremity may be displaying the first signs of acute compartment syndrome. This is an acute problem that requires immediate intervention because of possible decreased circulation. Moderate pain and swelling is an expected assessment after a fracture. These findings can be treated with comfort measures. Being cold can be treated with additional blankets or by increasing the temperature of the room.

24. A nurse cares for a client who had a long-leg cast applied last week. The client states, I cannot seem to catch my breath and I feel a bit light-headed. Which action should the nurse take next? a. Auscultate the clients lung fields anteriorly and posteriorly. b. Administer oxygen to keep saturations greater than 92%. c. Check the clients blood glucose level. d. Ask the client to take deep breaths.

ANS: B The clients symptoms are consistent with the development of pulmonary embolism caused by leg immobility in the long cast. The nurse should check the clients pulse oximetry reading and provide oxygen to keep saturations greater than 92%. Auscultating lung fields, checking blood glucose level, or deep breathing will not assist this client.

A client is admitted with acute kidney injury (AKI) and a urine output of 2000 mL/day. What is the major concern of the nurse regarding this client's care? a. Edema and pain b. Electrolyte and fluid imbalance c. Cardiac and respiratory status d. Mental health status

ANS: B This client may have an inflammatory cause of AKI with proteins entering the glomerulus and holding the fluid in the filtrate, causing polyuria. Electrolyte loss and fluid balance is essential. Edema and pain are not usually a problem with fluid loss. There could be changes in the client's cardiac, respiratory, and mental health status if the electrolyte imbalance is not treated.

1. A client is in the emergency department reporting a brief episode during which he was dizzy, unable to speak, and felt like his legs were very heavy. Currently the clients neurologic examination is normal. About what drug should the nurse plan to teach the client? a. Alteplase (Activase) b. Clopidogrel (Plavix) c. Heparin sodium d. Mannitol (Osmitrol)

ANS: B This clients manifestations are consistent with a transient ischemic attack, and the client would be prescribed aspirin or clopidogrel on discharge. Alteplase is used for ischemic stroke. Heparin and mannitol are not used for this condition.

A nurse cares for a client with a urine specific gravity of 1.018. Which action should the nurse take? a. Evaluate the client's intake and output for the past 24 hours. b. Document the finding in the chart and continue to monitor. c. Obtain a specimen for a urine culture and sensitivity. d. Encourage the client to drink more fluids, especially water.

ANS: B This specific gravity is within the normal range for urine. There is no need to evaluate the client's intake and output, obtain a urine specimen, or increase fluid intake.

A nurse collaborates with an unlicensed assistive personnel (UAP) to provide care for a client who is in the healing phase of acute pancreatitis. Which statements focused on nutritional requirements should the nurse include when delegating care for this client? (Select all that apply.) a. "Do not allow the client to eat between meals." b. "Make sure the client receives a protein shake." c. "Do not allow caffeine-containing beverages." d. "Make sure the foods are bland with little spice." e. "Do not allow high-carbohydrate food items."

ANS: B, C, D During the healing phase of pancreatitis, the client should be provided small, frequent, moderate- to high-carbohydrate, high-protein, low-fat meals. Protein shakes can be provided to supplement the diet. Foods and beverages should not contain caffeine and should be bland.

A nurse assesses an older client. Which assessment findings should the nurse identify as normal changes in the nervous system related to aging? (Select all that apply.) a. Long-term memory loss b. Slower processing time c. Increased sensory perception d. Decreased risk for infection e. Change in sleep patterns

ANS: B, E Normal changes in the nervous system related to aging include recent memory loss, slower processing time, decreased sensory perception, an increased risk for infection, changes in sleep patterns, changes in perception of pain, and altered balance and/or decreased coordination. DIF: Remembering/Knowledge REF: 836 KEY: Aging| older adult MSC: Integrated Process: Nursing Process: Assessment NOT: Client Needs Category: Health Promotion and Maintenance

The nurse is caring for four hypertensive clients. Which drug-laboratory value combination should the nurse report immediately to the health care provider? a. Furosemide (Lasix)/potassium: 2.1 mEq/L b. Hydrochlorothiazide (Hydrodiuril)/potassium: 4.2 mEq/L c. Spironolactone (Aldactone)/potassium: 5.1 mEq/L d. Torsemide (Demadex)/sodium: 142 mEq/L

Furosemide ( Lasix )/ potassium: 2.1 mEq/L Lasix is a loop diuretic and can cause hypokalemia. A potassium level of 2.1 mEq/L is quite low and should be reported immediately. Spironolactone is a potassium-sparing diuretic that can cause hyperkalemia. A potassium level of 5.1 mEq/L is on the high side, but it is not as critical as the low potassium with furosemide. The other two laboratory values are normal.

Which factor has been shown to delay stroke intervention? Age Gender Ethnic background Religious affiliation

Gender Studies have shown that although women have a better understanding of stroke symptomology, they may not seek care as quickly as men do. Strokes tend to be more prominent in older people, but does not necessarily lead to a delay in treatment. Ethnic background and religious affiliation have no direct bearing on timing of seeking stroke treatment.

Which potential disorder should be ruled out before a patient's level of consciousness is attributed to stroke? Hypoglycemia Hyperkalemia Hyponatremia Hypertension

Hypoglycemia Hypoglycemia can present as a decreased level of alertness/consciousness. If this is the case, the condition is easily reversed. Hyperkalemia, hyponatremia, and hypertension do not alter consciousness.

A nurse wants to provide community service that helps meet the goals of Healthy People 2020 related to cardiovascular disease and stroke. What activity would best meet this goal? a. Teach high school students heart-healthy living. b. Participate in blood pressure screenings at the mall. c. Provide pamphlets on heart disease at the grocery store. d. Set up an "Ask the nurse" booth at the pet store.

Participate in blood pressure screenings at the mall

The nurse is caring for a patient with DM. The patient's urine is positive for ketones. What does the nurse instruct the patient with regard to exercise? a. "When urine ketones are present, you should not exercise." b. "You may exercise as long as serum ketones are negative." c. "If you exercise now, be sure to perform aerobic exercises." d. "Exercise is always a good option because it helps with glucose utilization.

a. "When urine ketones are present, you should not exercise." QUESTION FROM THE STUDY GUIDE

6. A nurse administers prescribed adenosine (Adenocard) to a client. Which response should the nurse assess for as the expected therapeutic response? a. Decreased intraocular pressure b. Increased heart rate c. Short period of asystole d. Hypertensive crisis

ANS: C Clients usually respond to adenosine with a short period of asystole, bradycardia, hypotension, dyspnea, and chest pain. Adenosine has no conclusive impact on intraocular pressure.

A nurse cares for a client who is recovering from a myocardial infarction. The client states, "I will need to stop eating so much chili to keep that indigestion pain from returning." How should the nurse respond? a. "Chili is high in fat and calories; it would be a good idea to stop eating it." b. "The provider has prescribed an antacid for you to take every morning." c. "What do you understand about what happened to you?" d. "When did you start experiencing this indigestion?"

ANS: C Clients who experience myocardial infarction often respond with denial, which is a defense mechanism. The nurse should ask the client what he or she thinks happened, or what the illness means to him or her. The other responses do not address the client's misconception about recent pain and the cause of that pain.

A nurse assesses clients on the medical-surgical unit. Which client is at greatest risk for the development of bacterial cystitis? a. A 36-year-old female who has never been pregnant b. A 42-year-old male who is prescribed cyclophosphamide c. A 58-year-old female who is not taking estrogen replacement d. A 77-year-old male with mild congestive heart failure

ANS: C Females at any age are more susceptible to cystitis than men because of the shorter urethra in women. Postmenopausal women who are not on hormone replacement therapy are at increased risk for bacterial cystitis because of changes in the cells of the urethra and vagina. The middle-aged woman who has never been pregnant would not have a risk potential as high as the older woman who is not using hormone replacement therapy.

A nurse assesses a client who has cholecystitis. Which clinical manifestation indicates that the condition is chronic rather than acute? a. Temperature of 100.1° F (37.8° C) b. Positive Murphy's sign c. Light-colored stools d. Upper abdominal pain after eating

ANS: C Jaundice, clay-colored stools, and dark urine are more commonly seen with chronic cholecystitis. The other symptoms are seen equally with both chronic and acute cholecystitis.

A nurse is working with a community group promoting healthy aging. What recommendation is best to help prevent osteoarthritis (OA)? a. Avoid contact sports. b. Get plenty of calcium. c. Lose weight if needed. d. Engage in weight-bearing exercise.

ANS: C Obesity can lead to OA, and if the client is overweight, losing weight can help prevent OA or reduce symptoms once it occurs. Arthritis can be caused by contact sports, but this is less common than obesity. Calcium and weight-bearing exercise are both important for osteoporosis.

A nurse assesses an older adult client who has multiple chronic diseases. The client's heart rate is 48 beats/min. Which action should the nurse take first? a. Document the finding in the chart. b. Initiate external pacing. c. Assess the client's medications. d. Administer 1 mg of atropine.

ANS: C Pacemaker cells in the conduction system decrease in number as a person ages, resulting in bradycardia. The nurse should check the medication reconciliation for medications that might cause such a drop in heart rate, then should inform the health care provider. Documentation is important, but it is not the priority action. The heart rate is not low enough for atropine or an external pacemaker to be needed.

A nurse teaches a client who is at risk for colon cancer. Which dietary recommendation should the nurse teach this client? a. "Eat low-fiber and low-residual foods." b. "White rice and bread are easier to digest." c. "Add vegetables such as broccoli and cauliflower to your new diet." d. "Foods high in animal fat help to protect the intestinal mucosa."

ANS: C The client should be taught to modify his or her diet to decrease animal fat and refined carbohydrates. The client should also increase high-fiber foods and Brassica vegetables, including broccoli and cauliflower, which help to protect the intestinal mucosa from colon cancer.

A nurse is discharging a client after a total hip replacement. What statement by the client indicates good potential for self-management? a. "I can bend down to pick something up." b. "I no longer need to do my exercises." c. "I will not sit with my legs crossed." d. "I won't wash my incision to keep it dry."

ANS: C There are many precautions clients need to take after hip replacement surgery, including not bending more than 90 degrees at the hips, continuing prescribed exercises, not crossing the legs, and washing the incision daily and patting it dry.

1. A nurse is assessing a client who has suffered a nasal fracture. Which assessment should the nurse perform first? a. Facial pain b. Vital signs c. Bone displacement d. Airway patency

ANS: D A patent airway is the priority. The nurse first should make sure that the airway is patent and then should determine whether the client is in pain and whether bone displacement or blood loss has occurred.

15.A nurse assesses a client who was started on intraperitoneal therapy 5 days ago. The client reports abdominal pain and feeling warm. For which complication of this therapy should the nurse assess this client? a. Allergic reaction b. Bowel obstruction c. Catheter lumen occlusion d. Infection

ANS: D Fever, abdominal pain, abdominal rigidity, and rebound tenderness may be present in the client who has peritonitis related to intraperitoneal therapy. Peritonitis is preventable by using strict aseptic technique in handling all equipment and infusion supplies. An allergic reaction would occur earlier in the course of treatment. Bowel obstruction and catheter lumen occlusion can occur but would present clinically in different ways.

A client is in the family practice clinic reporting a severe "cold" that started 4 days ago. On examination, the nurse notes the client also has a severe headache and muscle aches. What action by the nurse is best? a. Educate the client on oseltamivir (Tamiflu). b. Facilitate admission to the hospital. c. Instruct the client to have a flu vaccine. d. Teach the client to sneeze in the upper sleeve.

ANS: D Sneezing and coughing into one's sleeve helps prevent the spread of upper respiratory infections. The client does have manifestations of the flu (influenza), but it is too late to start antiviral medications; to be effective, they must be started within 24 to 48 hours of symptom onset. The client does not need hospital admission. The client should be instructed to have a flu vaccination, but now that he or she has the flu, vaccination will have to wait until next year.

A nurse cares for a client who is experiencing deteriorating neurologic functions. The client states, "I am worried I will not be able to care for my young children." How should the nurse respond? a. "Caring for your children is a priority. You may not want to ask for help, but you have to." b. "Our community has resources that may help you with some household tasks so you have energy to care for your children." c. "You seem distressed. Would you like to talk to a psychologist about adjusting to your changing status?" d. "Give me more information about what worries you, so we can see if we can do something to make adjustments."

ANS: D Investigate specific concerns about situational or role changes before providing additional information. The nurse should not tell the client what is or is not a priority for him or her. Although community resources may be available, they may not be appropriate for the client. Consulting a psychologist would not be appropriate without obtaining further information from the client related to current concerns. DIF: Applying/Application REF: 844 KEY: Patient-centered care| therapeutic communication MSC: Integrated Process: Communication and Documentation NOT: Client Needs Category: Psychosocial Integrity

A nurse is discharging a client to a short-term rehabilitation center after a joint replacement. Which action by the nurse is most important? a. Administering pain medication before transport b. Answering any last-minute questions by the client c. Ensuring the family has directions to the facility d. Providing a verbal hand-off report to the facility

ANS: D As required by The Joint Commission and other accrediting agencies, a hand-off report must be given to the new provider to prevent error. The other options are valid responses but do not take priority.

1. A nurse assesses a clients electrocardiograph tracing and observes that not all QRS complexes are preceded by a P wave. How should the nurse interpret this observation? a. The client has hyperkalemia causing irregular QRS complexes. b. Ventricular tachycardia is overriding the normal atrial rhythm. c. The clients chest leads are not making sufficient contact with the skin. d. Ventricular and atrial depolarizations are initiated from different sites.

ANS: D Normal rhythm shows one P wave preceding each QRS complex, indicating that all depolarization is initiated at the sinoatrial node. QRS complexes without a P wave indicate a different source of initiation of depolarization. This finding on an electrocardiograph tracing is not an indication of hyperkalemia, ventricular tachycardia, or disconnection of leads.

A nurse cares for a client with a urine specific gravity of 1.040. Which action should the nurse take? a. Obtain a urine culture and sensitivity. b. Place the client on restricted fluids. c. Assess the client's creatinine level. d. Increase the client's fluid intake.

ANS: D Normal specific gravity for urine is 1.005 to 1.030. A high specific gravity can occur with dehydration, decreased kidney blood flow (often because of dehydration), and the presence of antidiuretic hormone. Increasing the client's fluid intake would be a beneficial intervention. Assessing the creatinine or obtaining a urine culture would not provide data necessary for the nurse to make a clinical decision.

A nurse is caring for a client after joint replacement surgery. What action by the nurse is most important to prevent wound infection? a. Assess the client's white blood cell count. b. Culture any drainage from the wound. c. Monitor the client's temperature every 4 hours. d. Use aseptic technique for dressing changes.

ANS: D Preventing surgical wound infection is a primary responsibility of the nurse, who must use aseptic technique to change dressings or empty drains. The other actions do not prevent infection but can lead to early detection of an infection that is already present.

4. A client with a stroke is being evaluated for fibrinolytic therapy. What information from the client or family is most important for the nurse to obtain? a. Loss of bladder control b. Other medical conditions c. Progression of symptoms d. Time of symptom onset

ANS: D The time limit for initiating fibrinolytic therapy for a stroke is 3 to 4.5 hours, so the exact time of symptom onset is the most important information for this client. The other information is not as critical.

A nurse is interested in providing community education and screening on hypertension. In order to reach a priority population, to target audience should the nurse provide this service? a. African-American churches b. Asian-American groceries c. High school sports camps d. Women's health clinics

African American churches African Americans in the United States have one of the highest rates of hypertension in the world. The nurse has the potential to reach this priority population by providing services at African-American churches. Although hypertension education and screening are important for all groups, African Americans are the priority population for this intervention.

The nurse advises a patient with a lower extremity fracture to take a multivitamin with iron after eating to prevent which condition? 1 Nausea 2 Dizziness 3 Palpitations 4 Constipation

Answer: 1 Nausea Taking the multi with iron AFTER EATING (as opposed to without food) will prevent nausea. Umm, so why is the nurse advising the patient to take it? Ooh, that's not the question....

What instructions for safe drug administration should be given to the patient with rheumatoid arthritis who is prescribed methotrexate (MTX)? Select all that apply. 1 "Avoid crowds." 2 "Reduce salt intake." 3 "Avoid alcohol intake." 4 "Refrigerate the medication." 5 "Report any side effects to the primary health care provider."

Answer: 1, 3, 5

A 40-year-old patient who is 36 hours into recovering from a fractured pelvis begins experiencing hypoxemia, dyspnea, and tachypnea. For which complication should the nurse first assess the patient? 1 Infection 2 Fat embolism 3 Hypovolemic shock 4 Acute compartment syndrome

Answer: 2 Fat embolism syndrome (FES) can be seen in men between the ages of 20-40 with fractures of the hip or pelvis. While infection, hypovolemic shock, and acute compartment syndrome are possible complications of any fracture, FES is specific to those injuries of bones where fat can be released from bone marrow, which is typically the bones of the pelvic girdle.

Which condition is the biggest risk factor for hip fracture? 1 Anemia 2 Osteoporosis 3 Atherosclerosis 4 Diabetes mellitus

Answer: 2 Osteoporosis is the biggest risk factor for hip fractures. Osteoporosis is characterized by the demineralization of the bones, making them porous and brittle. Anemia, atherosclerosis, and diabetes mellitus are not risk factors for hip fractures since these conditions may not show any effect on the demineralization of bones.

Which are initial nursing interventions for a patient with a distal radius fracture? Select all that apply. 1 Elevate. 2 Immobilize. 3 Ensure the area is infection-free. 4 Assist with activities of daily living. 5 Remove any jewelry on the affected side. 6 Perform a neurovascular assessment on the upper extremity.

Answer: 2, 3, 5, 6 The affected extremity should be elevated and immobilized. It is important to first remove all jewelry (rings, watches, bracelets, etc.) on the affected side before edema worsens. The nurse should also assess for neurovascular trauma and activity. Infection will not be apparent upon initial assessment. Activities of daily living are not considered a part of the initial interventions.

Which long-term drug therapy increases the risk of developing ischemic bone necrosis? 1 Ketorolac 2 Ibuprofen 3 Prednisone 4 Acetaminophen

Answer: 3 Patients on long-term corticosteroid therapy, such as prednisone, are at risk for ischemic necrosis. Ketorolac, ibuprofen, and acetaminophen do not increase the risk for developing ischemic necrosis.

The nurse is teaching a patient about taking etanercept for psoriatic arthritis. Which statement by the patient indicates that further teaching is needed? 1 "I'm comfortable self-administering etanercept." 2 "I will not take etanercept if I have an infection." 3 "I can use heat at the injection site if an itchy rash develops." 4 "If I get a respiratory infection, I will seek immediate care."

Answer: 3 I can use heat at the injection site if any itchy rash develops. Ice, not heat, and hydrocortisone 1% cream should be applied if a red, itchy rash develops at the injection site. The patient will self-administer etanercept injections. The patient should discontinue use of etanercept and seek immediate care if an infection develops.

Which are the EARLY manifestations of rheumatoid arthritis (RA)? Select all that apply. 1 Anemia 2 Deformities 3 Inflammation 4 Low-grade fever 5 Morning stiffness

Answer: 3, 4

The nurse is caring for a client with acute respiratory failure and PaCO2 level of 88 mm Hg For which of these signs and symptoms will the nurse assess? Select all that apply. A. Hyperactivity B. Headache C. Shallow Breathing D. pH 7.49 E. Fatigue

b, c, e (When caring for a client with acute respiratory failure and respiratory acidosis, the nurse would assess for lethargy, flushing, headache, shallow breathing, and fatigue. Clients experiencing acidosis have problems associated with the decreased function of excitable membranes.Generally, the client with respiratory acidosis will be lethargic rather than hyperactive and have a pH <7.35, which is a characteristic of acidosis.)

A patient with rheumatoid arthritis is prescribed hydroxychloroquine for joint and muscle pain. Which statements should the nurse include in the teaching? 1 "You should report hair loss." 2 "You should report chest pain." 3 "You should follow strict birth control." 4 "You should report any sign of blurred vision or headache."

Answer: 4 Hydroxychloroquine is an antimalarial drug that helps decrease joint and muscle pain in patients with rheumatoid arthritis. The drug can cause retinal damage so the patient should report any signs of blurred vision or headache.

Which physiologic change related to compartment syndrome is responsible for symptoms of flexed posture and unequal pulses in a patient? 1 Anaerobic metabolism 2 Pressure on nerve endings 3 Increased capillary permeability 4 Increased production of lactic acid

Answer: 4 Increased production of lactic acid may result in flexed posture and unequal pulses. Anaerobic metabolism may cause cyanosis. Pain may result due to pressure on nerve endings. Increased capillary permeability would result in edema.

Which type of traction is used to correct bone deformities of the lower extremities? 1 Pelvic belt 2 Pelvic sling 3 90-90 traction 4 Russell's traction

Answer: 4 Russell's traction is a type of lower extremity traction in which the affected leg is extended and a sling under the knee suspends the leg. A pelvic belt is a type of pelvic traction in which a strap around the hips at the iliac crests is attached to weights at the foot of the bed. A pelvic sling is a type of pelvic traction where a wide strap around the hips is attached to an overhead bar to keep the pelvis off the bed. Overhead, or 90-90, traction is a type of upper extremity traction where the elbow is flexed and the arm is at a right angle to the body over the upper chest.

A patient with rheumatoid arthritis with joint swelling had an arthrocentesis. Which post-procedure instruction, provided by the student nurse, needs correction? 1 "You should take acetaminophen if you have pain." 2 "You have to apply ice on the affected joint for 24 hours after the procedure." 3 "You have to immediately notify the health care provider if swelling increases." 4 "You should move the affected joint frequently during the first 24 hours after surgery.

Answer: 4 The affected joint should be kept at rest for 24 hours after arthrocentesis. The patient should take acetaminophen, as needed, for pain. Ice should be applied on the affected joint for 24 hours after arthrocentesis. An increase in swelling should be reported to the health care provider immediately.

What is an early manifestation of fat embolism syndrome? 1 Lethargy 2 Petechiae 3 Headache 4 Low arterial oxygen

Answer: 4 The earliest manifestations of FES are a low arterial oxygen level (hypoxemia), dyspnea, and tachypnea (increased respirations). Headache, lethargy, agitation, confusion, decreased level of consciousness, seizures, and vision changes may follow. Nonpalpable, red-brown petechiae—a macular, measles-like rash—may appear over the neck, upper arms, and/or chest.

The nurse is instructing a local community group about ways to reduce the risk for musculoskeletal injury. What information does the nurse include in the teaching plan? 1 "Avoid contact sports." 2 "Avoid rigorous exercise." 3 "Avoid driving in inclement weather." 4 "Wear helmets when riding a motorcycle."

Answer: 4 Those who ride motorcycles or bicycles should wear helmets to prevent head injury. Telling the general public to avoid contact sports or to avoid driving in inclement weather is not realistic. Telling the general public to avoid rigorous exercise is not only unrealistic, it is also opposed to what many health care professionals recommend to maintain health.

A nurse is caring for a client with a nonhealing arterial ulcer. The physician has informed the client about possibly needing to amputate the client's leg. The client is cry and upset. What actions by the nurse are best? a. Ask the client to describe his or her current emotions. b. Assess the client for support systems and family. c. Offer to stay with the client if he or she desires. d. Relate how smoking contributed to this situation. e. Tell the client that many people have amputations.

Ask the client to describe his or her current emotions Assess the client for support system and family Offer to stay with the client if he or she desires When a client is upset, the nurse should offer self by remaining with the client if desired. Other helpful measures include determining what and whom the client has for support systems and asking the client to describe what he or she is feeling. Telling the client how smoking has led to this situation will only upset the client further and will damage the therapeutic relationship. Telling the client that many people have amputations belittles the client's feelings.

A client with a known abdominal aortic aneurysm reports dizziness and severe abdominal pain. The nurse assesses the client's blood pressure at 82/40 mmHg. What actions by the nurse are most important? a. Administer pain medication. b. Assess distal pulses every 10 minutes. c. Have the client sign a surgical consent. d. Notify the Rapid Response Team. e. Take vital signs every 10 minutes

Assess distal pulses ever 10 minutes Notify the Rapid Response Team Take vital signs every 10 minutes This client may have a ruptured/rupturing aneurysm. The nurse should notify the Rapid Response team and perform frequent client assessments. Giving pain medication will lower the client's blood pressure even further. The nurse cannot have the client sign a consent until the physician has explained the procedure.

Along with exercise, what is the recommended calorie reduction for a patient with diabetes who must lose weight? a. 100-200 calories/day b. 250-500 calories,day c. 501-600 calories/day d. 601/750 calories/day

b. 250-500 calories,day QUESTION FROM THE STUDY GUIDE

From which injection site is insulin absorbed most rapidly? a. Buttocks b. Abdomen c. Deltoid d. Thigh

b. Abdomen QUESTION FROM THE STUDY GUIDE

A client has hypertension and high risk factors for cardiovascular disease. The client is overwhelmed with the recommended lifestyle changes. What action by the nurse is best? a. Assess the client's support system. b. Assist in finding one change the client can control. c. Determine what stressors the client faces in daily life. d. Inquire about delegating some of the client's obligations.

Assist in finding one change the client can control. All options are appropriate when assessing stress and responses to stress. However, this client feels overwhelmed by the suggested lifestyle changes. Instead of looking at all the needed changes, the nurse should assist the client in choosing one the client feels optimistic about controlling. Once the client has mastered that change, he or she can move forward with another change. Determining support systems, daily stressors, and delegation opportunities does not directly impact the client's feelings of control

The nurse working in the emergency department knows that which factors are commonly related to aneurysm formation? a. Atherosclerosis b. Down syndrome c. Frequent heartburn d. History of hypertension e. History of smoking

Atherosclerosis History of hypertension History of smoking Atherosclerosis, hypertension, hyperlipidemia, and smoking are the most common related factors. Down syndrome and heartburn have no relation to aneurysm formation.

A nurse cares for a client who has cirrhosis of the liver. Which action should the nurse take to decrease the presence of ascites? a. Monitor intake and output. b. Provide a low-sodium diet. c. Increase oral fluid intake. d. Weigh the client daily.

B

A nurse is caring for a client who has the following arterial blood values: pH 7.12, PaO2 56 mm Hg, PaCO2 65 mm Hg, and HCO3- 22 mEq/L. Which clinical situation should the nurse correlate with these values? a. Diabetic ketoacidosis in a person with emphysema b. Bronchial obstruction related to aspiration of a hot dog c. Anxiety-induced hyperventilation in an adolescent d. Diarrhea for 36 hours in an older, frail woman

B (Arterial blood gas values indicate that the client has acidosis with normal levels of bicarbonate, suggesting that the problem is not metabolic. Arterial concentrations of oxygen and carbon dioxide are abnormal, with low oxygen and high carbon dioxide levels. Thus, this client has respiratory acidosis from inadequate gas exchange. The fact that the bicarbonate level is normal indicates that this is an acute respiratory problem rather than a chronic problem, because no renal compensation has occurred.)

After teaching a client who was malnourished and is being discharged, a nurse assesses the client's understanding. Which statement indicates the client correctly understood teaching to decrease risk for the development of metabolic acidosis? a. "I will drink at least three glasses of milk each day." b. "I will eat three well-balanced meals and a snack daily." c. "I will not take pain medication and antihistamines together." d. "I will avoid salting my food when cooking or during meals."

B (Starvation or a diet with too few carbohydrates can lead to metabolic acidosis by forcing cells to switch to using fats for fuel and by creating ketoacids as a by-product of excessive fat metabolism. Eating sufficient calories from all food groups helps reduce this risk.)

A nurse is caring for a client who received benzocaine spray prior to a recent bronchoscopy. The client presents with continuous cyanosis even with oxygen therapy. Which action should the nurse take next? a. Administer an albuterol treatment. b. Notify the Rapid Response Team. c. Assess the clients peripheral pulses. d. Obtain blood and sputum cultures.

B Cyanosis unresponsive to oxygen therapy is a manifestation of methemoglobinemia, which is an adverse effect of benzocaine spray. Death can occur if the level of methemoglobin rises and cyanosis occurs. The nurse should notify the Rapid Response Team to provide advanced nursing care. An albuterol treatment would not address the clients oxygenation problem. Assessment of pulses and cultures will not provide data necessary to treat this client.

What glucose level range does the American Association of Clinical Endocrinologists recommend for a critically ill patient? a. Between 100 and 130 mg/dL b. Between 140 and 180 mg/dL c. Between 180 and 200 mg/dL d. Between 200 and 240 mg/dL

b. Between 140 and 180 mg/dL QUESTION FROM THE STUDY GUIDE

Which statement about dietary concepts for a patient with diabetes is true? a. Alcoholic bevberage consumption is unrestricted. b. Carbohydrate counting is emphasized when adjusting dietary intake of nutrients. c. Sweeteners should be avoided becasue of the side effects. d. Both soluble and insoluble fiber foods should be limnited.

b. Carbohydrate counting is emphasized when adjusting dietary intake of nutrients. QUESTION FROM THE STUDY GUIDE

A patient with type 1 DM is planning to travel by air and asks the nurse about preparations for the trip. What does the nurse tell the patient to do? a. Pack insulin and syringes in a labeled, crushproof kit in the checked luggage. b. Carry all necessary diabetes supplies in a clearly identified pack aboard the plane. c. Ask the flight attendant to put the insulin in the galley refrigerator once on the plane. d. Take only minimal supplies and get the prescription filled at his or her destination.

b. Carry all necessary diabetes supplies in a clearly identified pack aboard the plane. QUESTION FROM THE STUDY GUIDE

Which descriptors are typical of type 2 diabetes mellitus (DM)? Select all that apply. a. Autoimmune process causes beta cell destruction. b. Cells have decreased ability to respond to insulin. c. Diagnosis is based on results of 100-g glucose tolerance test. d. Most patients diagnosed are obese adults. e. Usually has abrupt onset of thirst and weight loss.

b. Cells have decreased ability to respond to insulin. d. Most patients diagnosed are obese adults. QUESTION FROM THE STUDY GUIDE

After providing discharge teaching, a nurse assesses the client's understanding regarding increased risk for metabolic alkalosis. Which statement indicates the client needs additional teaching? a. "I don't drink milk because it gives me gas and diarrhea." b. "I have been taking digoxin every day for the last 15 years." c. "I take sodium bicarbonate after every meal to prevent heartburn." d. "In hot weather, I sweat so much that I drink six glasses of water each day."

C (Excessive oral ingestion of sodium bicarbonate and other bicarbonate-based antacids can cause metabolic alkalosis. Avoiding milk, taking digoxin, and sweating would not lead to increased risk of metabolic alkalosis.)

A nurse assesses a client after an open lung biopsy. Which assessment finding is matched with the correct intervention? a. Client states he is dizzy. Nurse applies oxygen and pulse oximetry. b. Clients heart rate is 55 beats/min. Nurse withholds pain medication. c. Client has reduced breath sounds. Nurse calls physician immediately. d. Clients respiratory rate is 18 breaths/min. Nurse decreases oxygen flow rate.

C A potentially serious complication after biopsy is pneumothorax, which is indicated by decreased or absent breath sounds. The physician needs to be notified immediately. Dizziness after the procedure is not an expected finding. If the clients heart rate is 55 beats/min, no reason is known to withhold pain medication. A respiratory rate of 18 breaths/min is a normal finding and would not warrant changing the oxygen flow rate.

A patient with type 2 DM often has which laboratory value? a. Elevated thyroid studies b. Elevated triglycerides c. Ketones in the urine d. Low hemoglobin

b. Elevated triglycerides QUESTION FROM THE STUDY GUIDE

When glucagon is administered, what does it do? a. Competes for insulin at the receptor sites b. Frees glucose from hepatic stores of glycogen c. Supplies glycogen directly to the vital tissues d. Provides a glucose substitute for rapid replacement

b. Frees glucose from hepatic stores of glycogen QUESTION FROM THE STUDY GUIDE

A patient with DM has signs and symptoms of hypoglycemia. The patient is alert and oriented with a blood glucose of 56 mg/dL. What does the nurse do next? a. Give a glass of orange juice with two packets of sugar and continue to monitor the patient. b. Give 8 oz of skim milk and then a carbohydrate and protein snack. c. Give a complex carbohydrate and continue to monitor the patient. d. Administer D50 IV push and give the patient something to eat.

b. Give 8 oz of skim milk and then a carbohydrate and protein snack. QUESTION FROM THE STUDY GUIDE

Which insulins are considered to have a rapid onset of action? Select all that apply. a. Novolin 70/30 b. Glulisine c. Humulin N d. Aspart e. Lispro

b. Glulisine d. Aspart e. Lispro QUESTION FROM THE STUDY GUIDE

Which statement about insulin is true? a. Exogenous insulin is necessary for management of all cases of type 2 DM. b. Insulin's effectiveness depends on the individual patient's absorption of the drug. c. Insulin doses should be regulated according to self- monitoring urine hglucose levels. d. Insulin administered in multiple doses per day decreases the flexibility of a patient's lifestyle.

b. Insulin's effectiveness depends on the individual patient's absorption of the drug. QUESTION FROM THE STUDY GUIDE

A patient with diabetes presents to the emergency department with a blood sugar of 640 mg/dL and reports being constantly thirsty and having to urinate "all of the time". How does the nurse document this subjective finding? a. Polydipsia and polyphagia b. Polydipsia and polyuria c. Polycoria and polyuria d. Polyphagia and polyesthesia

b. Polydipsia and polyuria QUESTION FROM THE STUDY GUIDE

What is the priority nursing intervention for a patient with systemic lupus erythematosus (SLE)? 1 Instructing that fatigue may occur 2 Instructing the patient on how to protect the skin 3 Teaching the patient to monitor the body temperature 4 Suggesting that the patient identify support systems and coping strategies

Correct = 2 Instructing the patient on how to protect the skin

Which metabolic disorder may be responsible for the development of joint degradation in the patient with osteoarthritis (OA)? 1 Aging 2 Hemophilia 3 Diabetes mellitus 4 Sickle cell disease

Correct = 3 Diabetes Mellitus Diabetes mellitus causes reduced insulin production which results in an elevated blood glucose level. This can lead to an inflammatory state resulting in OA.

Which drug is beneficial for a patient with osteoarthritis who is having painful muscle spasms? 1 Diclofenac 2 Hyaluronic acid 3 Acetaminophen 4 Cyclobenzaprine hydrochloride

Correct = 4 Cyclobenzaprine hydrochloride Cyclobenzaprine hydrochloride is a muscle relaxant used to treat muscle spasms. Diclofenac, hyaluronic acid, and acetaminophen are used to treat pain associated with osteoarthritis but are not used to relieve muscle spasms.

The diabetic patient experiences early morning hyperglycemia (Somogyi effect) as a result of the counterregulatory response to hypoglycemia. What treatment does the nurse expect for this condition? Select all that apply. a. Administer a 10pm dose of intermediate actiing insulin. b. Provide an evening snack to ensure adequate dietary intake. c. Evaluate insulin dosage and exercise program. d. Add an oral antidiabetic drug to patients's regimen. e. Increase blood glucose checks to every 2 hours around the clock.

b. Provide an evening snack to ensure adequate dietary intake. c. Evaluate insulin dosage and exercise program. QUESTION FROM THE STUDY GUIDE

A nurse assesses a client who is prescribed furosemide (Lasix) for hypertension. For which acid-base imbalance should the nurse assess to prevent complications of this therapy? a. Respiratory acidosis b. Respiratory alkalosis c. Metabolic acidosis d. Metabolic alkalosis

D (Many diuretics, especially loop diuretics, increase the excretion of hydrogen ions, leading to excess acid loss through the renal system. This situation is an acid deficit of metabolic origin.)

The nurse is caring for a client with long standing emphysema and respiratory acidosis. For which of these compensatory mechanisms will the nurse assess? A.Decreased rate of breathing B.Increased loss of bicarbonate through the kidney C.Decreased depth of breathing D.Decreased loss of bicarbonate through the kidney

D (The compensatory mechanism the nurse anticipates is present in the client with long standing emphysema and respiratory acidosis is conservation of bicarbonate. A partially compensated respiratory acidosis will typically result.Increased loss of bicarbonate through the kidney, decreased rate, and depth of breathing will promote acidosis.)

A nurse is caring for a client who is experiencing moderate metabolic alkalosis. Which action should the nurse take? a. Monitor daily hemoglobin and hematocrit values. b. Administer furosemide (Lasix) intravenously. c. Encourage the client to take deep breaths. d. Teach the client fall prevention measures.

D (The priority nursing care for a client who is experiencing moderate metabolic alkalosis is providing client safety. Clients with metabolic alkalosis have muscle weakness and are at risk for falling. The other nursing interventions are not appropriate for metabolic alkalosis.)

Which class of antidiabetic medication is most likely to cause a hypoglycemic episode because of the long duration of action? a. Alpha-glucosidase inhibitors, which include miglitol (Glyset) b. Biguanides, which include metformin (Glucophage) c. Meglitinides, which include nateglinide (Starlix) d. Second generation sulfonylureas, which include glipizide (Glucotrol)

d. Second generation sulfonylureas, which include glipizide (Glucotrol) QUESTION FROM THE STUDY GUIDE

A patient has been receiving insulin in the abdomen for 3 days. On day 4, where does the nurse give the insulin injection? a. Deltoid b. Thigh c. Abdomen, but in an area different from the previous day's injection d. Abdomen, in the same area as the previous day's injection.

c. Abdomen, but in an area different from the previous day's injection QUESTION FROM THE STUDY GUIDE

The nurse is teaching a patient with diabetes about proper foot care, Which instructions does the nurse include? Select all that apply. a. Use rubbing alcohol to toughen the skin on the soles of the feet. b. Wear open-toed shoes or sandals in warm weather to prevent perspiration. c. Apply moisturizing cream to the feet after bathing, but not between the toes. d. Use cold water for bathing the feet to prevent inadvertent thermal injury. e. Do not go barefoot. f. Inspect the feet daily.

c. Apply moisturizing cream to the feet after bathing, but not between the toes. e. Do not go barefoot. f. Inspect the feet daily. QUESTION FROM THE STUDY GUIDE

The patient with diabetes has a foot that is warm, swollen, and painful. Walking causes the arch of the foot to collapse and gives the foot a "rocker bottom" shape. Which foot deformity does the nurse recognize? a. Hallux valgus b. Claw toe deformity c. Charcot foot d. Diabetic foot ulcer

c. Charcot foot QUESTION FROM THE STUDY GUIDE

Which oral agent may cause lactic acidosis? a. Nateglinide b. Repaglinide c. Metformin d. Miglitol

c. Metformin QUESTION FROM THE STUDY GUIDE

What type of insulin is used in the emergency treatment of DKA and hyperglycemic-hyperosmolar nonketotic syndrome (HHNS)? a. NPH b. Lente c. Regular d. Protamine zinc

c. Regular QUESTION FROM THE STUDY GUIDE

The patient with DM had a pancreas transplant and takes daily doses of cyclosporine (Neoral). For which key lab assessment does the nurse monitor? a. Serum electrolytes b. CBC with differential count c. Serum creatinine d. Clotting studies

c. Serum creatinine QUESTION FROM THE STUDY GUIDE

4. A nurse assesses a male client who has symptoms of cirrhosis. Which questions should the nurse ask to identify potential factors contributing to this laboratory result? (Select all that apply.) a. How frequently do you drink alcohol? b. Have you ever had sex with a man? c. Do you have a family history of cancer? d. Have you ever worked as a plumber? e. Were you previously incarcerated?

A, B, E

A nurse is planning interventions that regulate acid-base balance to ensure the pH of a client's blood remains within the normal range. Which abnormal physiologic functions may occur if the client experiences an acid-base imbalance? (Select all that apply.) a. Reduction in the function of hormones b. Fluid and electrolyte imbalances c. Increase in the function of selected enzymes d. Excitable cardiac muscle membranes e. Increase in the effectiveness of many drugs

A, B, E (Acid-base imbalances interfere with normal physiology, including reducing the function of hormones and enzymes, causing fluid and electrolyte imbalances, making heart membranes more excitable, and decreasing the effectiveness of many drugs.)

An infection control nurse develops a plan to decrease the number of health care professionals who contract viral hepatitis at work. Which ideas should the nurse include in this plan? (Select all that apply.) a. Policies related to consistent use of Standard Precautions b. Hepatitis vaccination mandate for workers in high-risk areas c. Implementation of a needleless system for intravenous therapy d. Number of sharps used in client care reduced where possible e. Postexposure prophylaxis provided in a timely manner

A, C, D, E

13. A nurse prepares to discharge a client with cardiac dysrhythmia who is prescribed home health care services. Which priority information should be communicated to the home health nurse upon discharge? a. Medication reconciliation b. Immunization history c. Religious beliefs d. Nutrition preferences

ANS: A The home health nurse needs to know current medications the client is taking to ensure assessment, evaluation, and further education related to these medications. The other information will not assist the nurse to develop a plan of care for the client.

9.While assessing a clients peripheral IV site, the nurse observes a streak of red along the vein path and palpates a 4-cm venous cord. How should the nurse document this finding? a. Grade 3 phlebitis at IV site b. Infection at IV site c. Thrombosed area at IV site d. Infiltration at IV site

ANS: A The presence of a red streak and palpable cord indicates grade 3 phlebitis. No information in the description indicates that infection, thrombosis, or infiltration is present.

Bouchard's Nodes

Bony nodules at the proximal interphalangeal joints

Which complication is associated with compartment syndrome? 1 Osteomyelitis 2 Pulmonary edema 3 Retinal hemorrhage 4 Myoglobinuria renal failure

Myoglobinuria renal failure results from compartment syndrome. Osteomyelitis is bone infection, which occurs with open fractures in which skin integrity is lost. Pulmonary edema is seen in fat embolism syndrome and may result in respiratory failure. Retinal hemorrhage is a rare complication that is associated with fat embolism.

Which are considered the early signs of diabetic nephropathy? Select all that apply. a. Positive urine red blood cells b. Microalbuminuria c. Positive urine glucose d. Positive urine white blood cells e. Elevated serum uric acid

b. Microalbuminuria e. Elevated serum uric acid QUESTION FROM THE STUDY GUIDE

A nurse is preparing a client for a femoropopliteal bypass operation. What action does the nurse delegate to the UAP? a. Administering preoperative medication b. Ensuring the consent is signed c. Marking pulses with a pen d. Raising the siderails on the bed e. Recording baseline vital signs

Raising the side rails on the bed Recording baseline vital sings The UAP can raise the siderails of the bed for client safety and take and record the vital signs. Administering medications, ensuring a consent is on the chart, and marking the pulses for later comparison should be done by the registered nurse. This is also often done by the postanesthesia care nurse and is part of the hand-off report.

The nurse is reviewing the lipid panel of a male client who has atherosclerosis. Which finding is most concerning? a. Cholesterol: 126 mg/dL b. High-density lipoprotein cholesterol (HDL-C): 48 mg/dL c. Low-density lipoprotein cholesterol (LDL-C): 122 mg/dL d. Triglycerides: 198 mg/dL

Triglycerides: 198 mg/dL

Which diabetic patient is at greatest risk for diabetic foot ulcer formation? a. 75 year old African American male with history of cardiovascular disease b. 53 year old Caucasian female with history of renal insufficiency c. 38 year old American Indian with history of gastric ulcers d. 28 year old Caucasian male with history of chronic kidney disease

a. 75 year old African American male with history of cardiovascular disease QUESTION FROM THE STUDY GUIDE

ANS: A, B, E A client with peritonitis may present with a distended abdomen, diminished bowel sounds, inability to pass flatus or feces, tachycardia, and decreased urine output secondary to dehydration. Bradycardia and hyperactive bowel sounds are not associated with peritonitis.

A nurse assesses a client with peritonitis. Which clinical manifestations should the nurse expect to find? (Select all that apply.) a. Distended abdomen b. Inability to pass flatus c. Bradycardia d. Hyperactive bowel sounds e. Decreased urine output

A nurse cares for a client with hepatopulmonary syndrome who is experiencing dyspnea with oxygen saturations at 92%. The client states, I do not want to wear the oxygen because it causes my nose to bleed. Get out of my room and leave me alone! Which action should the nurse take? a. Instruct the client to sit in as upright a position as possible. b. Add humidity to the oxygen and encourage the client to wear it. c. Document the clients refusal, and call the health care provider. d. Contact the provider to request an extra dose of the clients diuretic.

A

A nurse teaches a client with hepatitis C who is prescribed ribavirin (Copegus). Which statement should the nurse include in this clients discharge education? a. Use a pill organizer to ensure you take this medication as prescribed. b. Transient muscle aching is a common side effect of this medication. c. Follow up with your provider in 1 week to test ypur blood for toxicity d. Take your radial pulse for 1 minute prior to taking this medication

A

ANS: A, D Washing hands after contact with animals and using separate cutting boards for meat and other foods will help prevent E. coli infection. The other statements are not related to preventing E. coli infection.

A nurse teaches a community group ways to prevent Escherichia coli infection. Which statements should the nurse include in this group's teaching? (Select all that apply.) a. "Wash your hands after any contact with animals." b. "It is not necessary to buy a meat thermometer." c. "Stay away from people who are ill with diarrhea." d. "Use separate cutting boards for meat and vegetables." e. "Avoid swimming in backyard pools and using hot tubs."

An emergency department nurse assesses a client admitted after a lightning strike. Which assessment should the nurse complete first? a. Electrocardiogram (ECG) b. Wound inspection c. Creatinine kinase d. Computed tomography of head

ANS: A Clients who survive an immediate lightning strike can have serious myocardial injury, which can be manifested by ECG and myocardial perfusion abnormalities. The nurse should prioritize the ECG. Other assessments should be completed but are not the priority

A nurse assesses a client who is prescribed alosetron (Lotronex). Which assessment question should the nurse ask this client? a. "Have you been experiencing any constipation?" b. "Are you eating a diet high in fiber and fluids?" c. "Do you have a history of high blood pressure?" d. "What vitamins and supplements are you taking?"

ANS: A Ischemic colitis is a life-threatening complication of alosetron. The nurse should assess the client for constipation. The other questions do not identify complications related to alosetron.

A nurse cares for a client with end-stage pancreatic cancer. The client asks, "Why is this happening to me?" How should the nurse respond? a. "I don't know. I wish I had an answer for you, but I don't." b. "It's important to keep a positive attitude for your family right now." c. "Scientists have not determined why cancer develops in certain people." d. "I think that this is a trial so you can become a better person because of it."

ANS: A The client is not asking the nurse to actually explain why the cancer has occurred. The client may be expressing his or her feelings of confusion, frustration, distress, and grief related to this diagnosis. Reminding the client to keep a positive attitude for his or her family does not address the client's emotions or current concerns. The nurse should validate that there is no easy or straightforward answer as to why the client has cancer. Telling a client that cancer is a trial is untrue and may diminish the client-nurse relationship.

A nurse cares for a client who had a colostomy placed in the ascending colon 2 weeks ago. The client states, "The stool in my pouch is still liquid." How should the nurse respond? a. "The stool will always be liquid with this type of colostomy." b. "Eating additional fiber will bulk up your stool and decrease diarrhea." c. "Your stool will become firmer over the next couple of weeks." d. "This is abnormal. I will contact your health care provider."

ANS: A The stool from an ascending colostomy can be expected to remain liquid because little large bowel is available to reabsorb the liquid from the stool. This finding is not abnormal. Liquid stool from an ascending colostomy will not become firmer with the addition of fiber to the client's diet or with the passage of time.

A nurse assesses a client who is recovering from an open Whipple procedure. Which action should the nurse perform first? a. Assess the client's endotracheal tube with 40% FiO2. b. Insert an indwelling Foley catheter to gravity drainage. c. Place the client's nasogastric tube to low intermittent suction. d. Start lactated Ringer's solution through an intravenous catheter.

ANS: A Using the ABCs, airway and oxygenation status should always be assessed first, so checking the endotracheal tube is the first action. Next, the nurse should start the IV line (circulation). After that, the Foley catheter can be inserted and the nasogastric tube can be set.

A nurse plans care for a client who has acute pancreatitis and is prescribed nothing by mouth (NPO). With which health care team members should the nurse collaborate to provide appropriate nutrition to this client? (Select all that apply.) a. Registered dietitian b. Nursing assistant c. Clinical pharmacist d. Certified herbalist e. Health care provider

ANS: A, C, E Clients who are prescribed NPO while experiencing an acute pancreatitis episode may need enteral or parenteral nutrition. The nurse should collaborate with the registered dietitian, clinical pharmacist, and health care provider to plan and implement the more appropriate nutritional interventions. The nursing assistant and certified herbalist would not assist with this clinical decision.

A nurse inserts a nasogastric (NG) tube for an adult client who has a bowel obstruction. Which actions does the nurse perform correctly? (Select all that apply.) a. Performs hand hygiene and positions the client in high-Fowler's position, with pillows behind the head and shoulders b. Instructs the client to extend the neck against the pillow once the NG tube has reached the oropharynx c. Checks for correct placement by checking the pH of the fluid aspirated from the tube d. Secures the NG tube by taping it to the client's nose and pinning the end to the pillowcase e. Connects the NG tube to intermittent medium suction with an anti-reflux valve on the air vent

ANS: A, C, E The client's head should be flexed forward once the NG tube has reached the oropharynx. The NG tube should be secured to the client's gown, not to the pillowcase, because it could become dislodged easily. All the other actions are appropriate.

A nurse assesses a client with cholelithiasis. Which assessment findings should the nurse identify as contributors to this client's condition? (Select all that apply.) a. Body mass index of 46 b. Vegetarian diet c. Drinking 4 ounces of red wine nightly d. Pregnant with twins e. History of metabolic syndrome f. Glycosylated hemoglobin level of 15%

ANS: A, D, F Obesity, pregnancy, and diabetes are all risk factors for the development of cholelithiasis. A diet low in saturated fats and moderate alcohol intake may decrease the risk. Although metabolic syndrome is a precursor to diabetes, it is not a risk factor for cholelithiasis. The client should be informed of the connection.

A nurse obtains a clients health history at a community health clinic. Which statement alerts the nurse to provide health teaching to this client? a. I drink two glasses of red wine each week b. I take a lot of Tylenol for my arthritis pain. c. I have a cousin who died of liver cancer. d. I got a hepatitis vaccine before traveling.

B

A nurse plans care for a client with acute pancreatitis. Which intervention should the nurse include in this client's plan of care to reduce discomfort? a. Administer morphine sulfate intravenously every 4 hours as needed. b. Maintain nothing by mouth (NPO) and administer intravenous fluids. c. Provide small, frequent feedings with no concentrated sweets. d. Place the client in semi-Fowler's position with the head of bed elevated.

ANS: B The client should be kept NPO to reduce GI activity and reduce pancreatic enzyme production. IV fluids should be used to prevent dehydration. The client may need a nasogastric tube. Pain medications should be given around the clock and more frequently than every 4 to 6 hours. A fetal position with legs drawn up to the chest will promote comfort.

A nurse cares for a client who is prescribed patient-controlled analgesia (PCA) after a cholecystectomy. The client states, "When I wake up I am in pain." Which action should the nurse take? a. Administer intravenous morphine while the client sleeps. b. Encourage the client to use the PCA pump upon awakening. c. Contact the provider and request a different analgesic. d. Ask a family member to initiate the PCA pump for the client

ANS: B The nurse should encourage the client to use the PCA pump prior to napping and upon awakening. Administering additional intravenous morphine while the client sleeps places the client at risk for respiratory depression. The nurse should also evaluate dosages received compared with dosages requested and contact the provider if the dose or frequency is not adequate. Only the client should push the pain button on a PCA pump.

A nurse assesses a client who is recovering from a Whipple procedure. Which clinical manifestations alert the nurse to a complication from this procedure? (Select all that apply.) a. Clay-colored stools b. Substernal chest pain c. Shortness of breath d. Lack of bowel sounds or flatus e. Urine output of 20 mL/6 hr

ANS: B, C, D, E Myocardial infarction (chest pain), pulmonary embolism (shortness of breath), adynamic ileus (lack of bowel sounds or flatus), and renal failure (urine output of 20 mL/6 hr) are just some of the complications for which the nurse must assess the client after the Whipple procedure. Clay-colored stools are associated with cholecystitis and are not a complication of a Whipple procedure.

A nurse prepares a client for a colonoscopy scheduled for tomorrow. The client states, "My doctor told me that the fecal occult blood test was negative for colon cancer. I don't think I need the colonoscopy and would like to cancel it." How should the nurse respond? a. "Your doctor should not have given you that information prior to the colonoscopy." b. "The colonoscopy is required due to the high percentage of false negatives with the blood test." c. "A negative fecal occult blood test does not rule out the possibility of colon cancer." d. "I will contact your doctor so that you can discuss your concerns about the procedure."

ANS: C A negative result from a fecal occult blood test does not completely rule out the possibility of colon cancer. To determine whether the client has colon cancer, a colonoscopy should be performed so the entire colon can be visualized and a tissue sample taken for biopsy. The client may want to speak with the provider, but the nurse should address the client's concerns prior to contacting the provider.

A nurse teaches a client who is recovering from a colon resection. Which statement should the nurse include in this client's plan of care? a. "You may experience nausea and vomiting for the first few weeks." b. "Carbonated beverages can help decrease acid reflux from anastomosis sites." c. "Take a stool softener to promote softer stools for ease of defecation." d. "You may return to your normal workout schedule, including weight lifting."

ANS: C Clients recovering from a colon resection should take a stool softener as prescribed to keep stools a soft consistency for ease of passage. Nausea and vomiting are symptoms of intestinal obstruction and perforation and should be reported to the provider immediately. The client should be advised to avoid gas-producing foods and carbonated beverages, and avoid lifting heavy objects or straining on defecation.

A nurse assesses clients at a community health center. Which client is at highest risk for pancreatic cancer? a. A 32-year-old with hypothyroidism b. A 44-year-old with cholelithiasis c. A 50-year-old who has the BRCA2 gene mutation d. A 68-year-old who is of African-American ethnicity

ANS: C Mutations in both the BRCA2 and p16 genes increase the risk for developing pancreatic cancer in a small number of cases. The other factors do not appear to be linked to increased risk.

A nurse cares for a client with acute pancreatitis. The client states, "I am hungry." How should the nurse reply? a. "Is your stomach rumbling or do you have bowel sounds?" b. "I need to check your gag reflex before you can eat." c. "Have you passed any flatus or moved your bowels?" d. "You will not be able to eat until the pain subsides."

ANS: C Paralytic ileus is a common complication of acute pancreatitis. The client should not eat until this has resolved. Bowel sounds and decreased pain are not reliable indicators of peristalsis. Instead, the nurse should assess for passage of flatus or bowel movement.

A nurse assesses a client admitted with a brown recluse spider bite. Which priority assessment should the nurse perform to identify complications of this bite? a. Ask the client about pruritus at the bite site. b. Inspect the bite site for a bluish purple vesicle. c. Assess the extremity for redness and swelling. d. Monitor the clients temperature every 4 hours.

ANS: D Fever and chills indicate systemic toxicity, which can lead to hemolytic reactions, kidney failure, pulmonary edema, cardiovascular collapse, and death. Assessing for a fever should be the nurses priority. All other symptoms are normal for a brown recluse bite and should be assessed, but they do not provide information about complications from the bite, and therefore are not the priority.

A nurse assesses clients at a community health center. Which client is at highest risk for the development of colorectal cancer? a. A 37-year-old who drinks eight cups of coffee daily b. A 44-year-old with irritable bowel syndrome (IBS) c. A 60-year-old lawyer who works 65 hours per week d. A 72-year-old who eats fast food frequently

ANS: D Colon cancer is rare before the age of 40, but its incidence increases rapidly with advancing age. Fast food tends to be high in fat and low in fiber, increasing the risk for colon cancer. Coffee intake, IBS, and a heavy workload do not increase the risk for colon cancer.

A nurse assesses clients on the medical-surgical unit. Which client should the nurse identify as at high risk for pancreatic cancer? a. A 26-year-old with a body mass index of 21 b. A 33-year-old who frequently eats sushi c. A 48-year-old who often drinks wine d. A 66-year-old who smokes cigarettes

ANS: D Risk factors for pancreatic cancer include obesity, older age, high intake of red meat, and cigarette smoking. Sushi and wine intake are not risk factors for pancreatic cancer.

A nurse cares for a client with hepatic portal-systemic encephalopathy (PSE). The client is thin and cachectic in appearance, and the family expresses distress that the client is receiving little dietary protein. How should the nurse respond? a. A low-protein diet will help the liver rest and will restore liver function. b. Less protein in the diet will help prevent confusion associated with liver failure. c. Increasing dietary protein will help the client gain weight and muscle mass. d. Low dietary protein is needed to prevent fluid from leaking into the abdomen.

B

An emergency room nurse assesses a client after a motor vehicle crash. The nurse notices a steering wheel mark across the clients chest. Which action should the nurse take? a. Ask the client where in the car he or she was sitting during the crash. b. Assess the client by gently palpating the abdomen for tenderness. c. Notify the laboratory to draw blood for blood type and crossmatch. d. Place the client on the stretcher in reverse Trendelenburg position.

B

A nurse assesses a client who has liver disease. Which laboratory findings should the nurse recognize as potentially causing complications of this disorder? (Select all that apply.) a. Elevated aspartate transaminase b. Elevated international normalized ratio (INR) c. Decreased serum globulin levels d. Decreased serum alkaline phosphatase e. Elevated serum ammonia f. Elevated prothrombin time (PT)

B, E, F

A nurse cares for a client who has chronic cirrhosis from substance abuse. The client states, All of my family hates me. How should the nurse respond? a. You should make peace with your family. b. This is not unusual. My family hates me too. c. I will help you identify a support system. d. You must attend Alcoholics Anonymous.

C

After teaching a client who has plans to travel to a non-industrialized country, the nurse assesses the clients understanding regarding the prevention of viral hepatitis. Which statement made by the client indicates a need for additional teaching? a I should drink bottled water during my travels, b. I will not eat off anothers plate or share utensils. c. I should eat plenty of fresh fruits and vegetables. d. I will wash my hands frequently and thoroughly.

C

A nurse assesses a client who is prescribed an infusion of vasopressin (Pitressin) for bleeding esophageal varices. Which clinical manifestation should alert the nurse to a serious adverse effect? a. Nausea and vomiting b. Frontal headache c. Vertigo and syncope d. Mid-sternal chest pain

D

A nurse cares for a client who is hemorrhaging from bleeding esophageal varices and has an esophagogastric tube. Which action should the nurse take first? a. Sedate the client to prevent tube dislodgement. b. Maintain balloon pressure at 15 and 20 mm Hg. c. Irrigate the gastric lumen with normal saline. d. Assess the client for airway patency.

D

A nurse cares for a client who is recovering from an open Whipple procedure. Which action should the nurse take? a. Clamp the nasogastric tube. b. Place the client in semi-Fowler's position. c. Assess vital signs once every shift. d. Provide oral rehydration.

Postoperative care for a client recovering from an open Whipple procedure should include placing the client in a semi-Fowler's position to reduce tension on the suture line and anastomosis sites, setting the nasogastric tube to low suction to remove free air buildup and pressure, assessing vital signs frequently to assess fluid and electrolyte complications, and providing intravenous fluids.

A client has peripheral arterial disease ( PAD ). What statement by the client indicates misunderstanding about self-management activities? a. "I can use a heating pad on my legs if it's set on low." b. "I should not cross my legs when sitting or lying down." c. "I will go out and buy some warm, heavy socks to wear." d. "It's going to be really hard but I will stop smoking."

" I can use a heating pad on my legs if it's set on low " Clients with PAD should never use heating pads as skin sensitivity is diminished and burns can result. The other statements show good understanding of self-management.

A student nurse asks what " essential hypertension " is. What response by the registered nurse is best? a. "It means it is caused by another disease." b. "It means it is 'essential' that it be treated." c. "It is hypertension with no specific cause." d. "It refers to severe and life-threatening hypertension."

" It is hypertension with no specific cause " Essential hypertension is the most common type of hypertension and has no specific cause such as an underlying disease process. Hypertension that is due to another disease process is called secondary hypertension. A severe, life-threatening form of hypertension is malignant hypertension.

An older client with PVD is explaining the daily foot care regimen to the family practices clinic nurse. What statement by the client may indicate a barrier to proper foot care? a. "I nearly always wear comfy sweatpants and house shoes." b. "I'm glad I get energy assistance so my house isn't so cold." c. "My daughter makes sure I have plenty of lotion for my feet." d. "My hands shake when I try to do things requiring coordination."

" My hands shake when I try to do things requiring coordination " Clients with PVD need to pay special attention to their feet. Toenails need to be kept short and cut straight across. The client whose hands shake may cause injury when trimming toenails. The nurse should refer this client to a podiatrist. Comfy sweatpants and house shoes are generally loose and not restrictive, which is important for clients with PVD. Keeping the house at a comfortable temperature makes it less likely the client will use alternative heat sources, such as heating pads, to stay warm. The client should keep the feet moist and soft with lotion.

A nurse assesses a client who is experiencing an acid-base imbalance. The client's arterial blood gas values are pH 7.32, PaO2 94 mm Hg, PaCO2 34 mm Hg, and HCO3- 18 mEq/L. For which clinical manifestations should the nurse assess? (Select all that apply.) a. Reduced deep tendon reflexes b. Drowsiness c. Increased respiratory rate d. Decreased urinary output e. Positive Trousseau's sign

A, B, C (Metabolic acidosis causes neuromuscular changes, including reduced muscle tone and deep tendon reflexes. Clients usually present with lethargy and drowsiness. The respiratory system will attempt to compensate for the metabolic acidosis; therefore, respirations will increase rate and depth. A positive Trousseau's sign is associated with alkalosis. Decreased urine output is not a manifestation of metabolic acidosis.)

A nurse delegates hygiene care for a client who has advanced cirrhosis to an unlicensed nursing personnel (UAP). Which statements should the nurse include when delegating this task to the UAP? (Select all that apply.) a. Apply lotion to the clients dry skin areas. b. Use a basin with warm water to bathe the client. c. For the clients oral care, use a soft toothbrush. d. Provide clippers so the client can trim the fingernails. e. Bathe with antibacterial and water-based soaps.

A, C, D

15.A nurse cares for a client with diabetes mellitus who is visually impaired. The client asks, Can I ask my niece to prefill my syringes and then store them for later use when I need them? How should the nurse respond? a. Yes. Prefilled syringes can be stored for 3 weeks in the refrigerator in a vertical position with the needle pointing up. b. Yes. Syringes can be filled with insulin and stored for a month in a location that is protected from light. c. Insulin reacts with plastic, so prefilled syringes are okay, but you will need to use glass syringes. d. No. Insulin syringes cannot be prefilled and stored for any length of time outside of the container.

ANS: A Insulin is relatively stable when stored in a cool, dry place away from light. When refrigerated, prefilled plastic syringes are stable for up to 3 weeks. They should be stored in the refrigerator in the vertical position with the needle pointing up to prevent suspended insulin particles from clogging the needle.

A client is receiving oxygen at 4 liters per nasal cannula. What comfort measure may the nurse delegate to unlicensed assistive personnel (UAP)? a. Apply water-soluble ointment to nares and lips. b. Periodically turn the oxygen down or off. c. Remove the tubing from the clients nose. d. Turn the client every 2 hours or as needed.

ANS: A Oxygen can be drying, so the UAP can apply water-soluble lubricant to the clients lips and nares. The UAP should not adjust the oxygen flow rate or remove the tubing. Turning the client is not related to comfort measures for oxygen.

8. A nurse notes crepitation when performing range-of-motion exercises on a client with a fractured left humerus. Which action should the nurse take next? a. Immobilize the left arm. b. Assess the clients distal pulse. c. Monitor for signs of infection. d. Administer prescribed steroids.

ANS: A A grating sound heard when the affected part is moved is known as crepitation. This sound is created by bone fragments. Because bone fragments may be present, the nurse should immobilize the clients arm and tell the client not to move the arm. The grating sound does not indicate circulation impairment or infection. Steroids would not be indicated.

2. A client had an embolic stroke and is having an echocardiogram. When the client asks why the provider ordered a test on my heart, how should the nurse respond? a. Most of these types of blood clots come from the heart. b. Some of the blood clots may have gone to your heart too. c. We need to see if your heart is strong enough for therapy. d. Your heart may have been damaged in the stroke too.

ANS: A An embolic stroke is caused when blood clots travel from one area of the body to the brain. The most common source of the clots is the heart. The other statements are inaccurate.

A nurse reviews laboratory results for a client who was admitted for a myocardial infarction and cardiogenic shock 2 days ago. Which laboratory test result should the nurse expect to find? a. Blood urea nitrogen (BUN) of 52 mg/dL b. Creatinine of 2.3 mg/dL c. BUN of 10 mg/dL d. BUN/creatinine ratio of 8:1

ANS: A Shock leads to decreased renal perfusion. An elevated BUN accompanies this condition. The creatinine should be normal because no kidney damage occurred. A low BUN signifies overhydration, malnutrition, or liver damage. A low BUN/creatinine ratio indicates fluid volume excess or acute renal tubular acidosis.

A nurse teaches a young female client who is prescribed amoxicillin (Amoxil) for a urinary tract infection. Which statement should the nurse include in this client's teaching? a. "Use a second form of birth control while on this medication." b. "You will experience increased menstrual bleeding while on this drug." c. "You may experience an irregular heartbeat while on this drug." d. "Watch for blood in your urine while taking this medication."

ANS: A The client should use a second form of birth control because penicillin seems to reduce the effectiveness of estrogen-containing contraceptives. She should not experience increased menstrual bleeding, an irregular heartbeat, or blood in her urine while taking the medication.

A client is getting out of bed into the chair for the first time after an uncemented hip replacement. What action by the nurse is most important? a. Have adequate help to transfer the client. b. Provide socks so the client can slide easier. c. Tell the client full weight bearing is allowed. d. Use a footstool to elevate the client's leg.

ANS: A The client with an uncemented hip will be on toe-touch only right after surgery. The nurse should ensure there is adequate help to transfer the client while preventing falls. Slippery socks will encourage a fall. Elevating the leg greater than 90 degrees is not allowed.

The nurse on the postoperative inpatient unit assesses a client after a total hip replacement. The client's surgical leg is visibly shorter than the other one and the client reports extreme pain. While a co-worker calls the surgeon, what action by the nurse is best? a. Assess neurovascular status in both legs. b. Elevate the affected leg and apply ice. c. Prepare to administer pain medication. d. Try to place the affected leg in abduction.

ANS: A This client has manifestations of hip dislocation, a critical complication of this surgery. Hip dislocation can cause neurovascular compromise. The nurse should assess neurovascular status, comparing both legs. The nurse should not try to move the extremity to elevate or abduct it. Pain medication may be administered if possible, but first the nurse should thoroughly assess the client.

A client is unsure of the decision to undergo peritoneal dialysis (PD) and wishes to discuss the advantages of this treatment with the nurse. Which statements by the nurse are accurate regarding PD? (Select all that apply.) a. "You will not need vascular access to perform PD." b. "There is less restriction of protein and fluids." c. "You will have no risk for infection with PD." d. "You have flexible scheduling for the exchanges." e. "It takes less time than hemodialysis treatments."

ANS: A, B, D PD is based on exchanges of waste, fluid, and electrolytes in the peritoneal cavity. There is no need for vascular access. Protein is lost in the exchange, which allows for more protein and fluid in the diet. There is flexibility in the time for exchanges, but the treatment takes a longer period of time compared to hemodialysis. There still is risk for infection with PD, especially peritonitis.

5. A nurse assesses a client with a cast for potential compartment syndrome. Which clinical manifestations are correctly paired with the physiologic changes of compartment syndrome? (Select all that apply.) a. Edema Increased capillary permeability b. Pallor Increased blood blow to the area c. Unequal pulses Increased production of lactic acid d. Cyanosis Anaerobic metabolism e. Tingling A release of histamine

ANS: A, C, D Clinical manifestations of compartment syndrome are caused by several physiologic changes. Edema is caused by increased capillary permeability, release of histamine, decreased tissue perfusion, and vasodilation. Unequal pulses are caused by an increased production of lactic acid. Cyanosis is caused by anaerobic metabolism. Pallor is caused by decreased oxygen to tissues, and tingling is caused by increased tissue pressure.

A nurse reviews a client's laboratory results. Which findings should alert the nurse to the possibility of atherosclerosis? (Select all that apply.) a. Total cholesterol: 280 mg/dL b. High-density lipoprotein cholesterol: 50 mg/dL c. Triglycerides: 200 mg/dL d. Serum albumin: 4 g/dL e. Low-density lipoprotein cholesterol: 160 mg/dL

ANS: A, C, E A lipid panel is often used to screen for cardiovascular risk. Total cholesterol, triglycerides, and low-density lipoprotein cholesterol levels are all high, indicating higher risk for cardiovascular disease. High-density lipoprotein cholesterol is within the normal range for both males and females. Serum albumin is not assessed for atherosclerosis.

1. A nurse cares for a client with congestive heart failure who has a regular cardiac rhythm of 128 beats/min. For which physiologic alterations should the nurse assess? (Select all that apply.) a. Decrease in cardiac output b. Increase in cardiac output c. Decrease in blood pressure d. Increase in blood pressure e. Decrease in urine output f. Increase in urine output

ANS: A, D, E Elevated heart rates in a healthy client initially cause blood pressure and cardiac output to increase. However, in a client who has congestive heart failure or a client with long-term tachycardia, ventricular filling time, cardiac output, and blood pressure eventually decrease. As cardiac output and blood pressure decrease, urine output will fall.

A nurse is teaching a client with diabetes mellitus who asks, Why is it necessary to maintain my blood glucose levels no lower than about 60 mg/dL? How should the nurse respond? a. Glucose is the only fuel used by the body to produce the energy that it needs. b. Your brain needs a constant supply of glucose because it cannot store it. c. Without a minimum level of glucose, your body does not make red blood cells. d. Glucose in the blood prevents the formation of lactic acid and prevents acidosis.

ANS: B Because the brain cannot synthesize or store significant amounts of glucose, a continuous supply from the bodys circulation is needed to meet the fuel demands of the central nervous system. The nurse would want to educate the client to prevent hypoglycemia. The body can use other sources of fuel, including fat and protein, and glucose is not involved in the production of red blood cells. Glucose in the blood will encourage glucose metabolism but is not directly responsible for lactic acid formation.

A nurse teaches a client with type 2 diabetes mellitus who is prescribed glipizide (Glucotrol). Which statement should the nurse include in this clients teaching? a. Change positions slowly when you get out of bed. b. Avoid taking nonsteroidal anti-inflammatory drugs (NSAIDs). c. If you miss a dose of this drug, you can double the next dose. d. Discontinue the medication if you develop a urinary infection

ANS: B NSAIDs potentiate the hypoglycemic effects of sulfonylurea agents. Glipizide is a sulfonylurea. The other statements are not applicable to glipizide.

A nursing student caring for a client removes the clients oxygen as prescribed. The client is now breathing what percentage of oxygen in the room air? a. 14% b. 21% c. 28% d. 31%

ANS: B Room air is 21% oxygen.

22. A client has an intraventricular catheter. What action by the nurse takes priority? a. Document intracranial pressure readings. b. Perform hand hygiene before client care. c. Measure intracranial pressure per hospital policy. d. Teach the client and family about the device.

ANS: B All of the actions are appropriate for this client. However, performing hand hygiene takes priority because it prevents infection, which is a possibly devastating complication.

After teaching a client with irritable bowel syndrome (IBS), a nurse assesses the client's understanding. Which menu selection indicates that the client correctly understands the dietary teaching? a. Ham sandwich on white bread, cup of applesauce, glass of diet cola b. Broiled chicken with brown rice, steamed broccoli, glass of apple juice c. Grilled cheese sandwich, small banana, cup of hot tea with lemon d. Baked tilapia, fresh green beans, cup of coffee with low-fat milk

ANS: B Clients with IBS are advised to eat a high-fiber diet (30 to 40 g/day), with 8 to 10 cups of liquid daily. Chicken with brown rice, broccoli, and apple juice has the highest fiber content. They should avoid alcohol, caffeine, and other gastric irritants.

A nurse assesses a client who has a history of heart failure. Which question should the nurse ask to assess the extent of the client's heart failure? a. "Do you have trouble breathing or chest pain?" b. "Are you able to walk upstairs without fatigue?" c. "Do you awake with breathlessness during the night?" d. "Do you have new-onset heaviness in your legs?"

ANS: B Clients with a history of heart failure generally have negative findings, such as shortness of breath. The nurse needs to determine whether the client's activity is the same or worse, or whether the client identifies a decrease in activity level. Trouble breathing, chest pain, breathlessness at night, and peripheral edema are symptoms of heart failure, but do not provide data that can determine the extent of the client's heart failure.

A client with rheumatoid arthritis (RA) has an acutely swollen, red, and painful joint. What nonpharmacologic treatment does the nurse apply? a. Heating pad b. Ice packs c. Splints d. Wax dip

ANS: B Ice is best for acute inflammation. Heat often helps with joint stiffness. Splinting helps preserve joint function. A wax dip is used to provide warmth to the joint which is more appropriate for chronic pain and stiffness.

The nurse is assessing a client with a diagnosis of pre-renal acute kidney injury (AKI). Which condition would the nurse expect to find in the client's recent history? a. Pyelonephritis b. Myocardial infarction c. Bladder cancer d. Kidney stones

ANS: B Pre-renal causes of AKI are related to a decrease in perfusion, such as with a myocardial infarction. Pyelonephritis is an intrinsic or intrarenal cause of AKI related to kidney damage. Bladder cancer and kidney stones are post-renal causes of AKI related to urine flow obstruction.

A nurse obtains the health history of a client who is newly admitted to the medical unit. Which statement by the client should alert the nurse to the presence of edema? a. "I wake up to go to the bathroom at night." b. "My shoes fit tighter by the end of the day." c. "I seem to be feeling more anxious lately." d. "I drink at least eight glasses of water a day."

ANS: B Weight gain can result from fluid accumulation in the interstitial spaces. This is known as edema. The nurse should note whether the client feels that his or her shoes or rings are tight, and should observe, when present, an indentation around the leg where the socks end. The other answers do not describe edema.

The nursing student studying rheumatoid arthritis (RA) learns which facts about the disease? (Select all that apply.) a. It affects single joints only. b. Antibodies lead to inflammation. c. It consists of an autoimmune process. d. Morning stiffness is rare. e. Permanent damage is inevitable.

ANS: B, C RA is a chronic autoimmune systemic inflammatory disorder leading to arthritis-type symptoms in the joints and other symptoms that can be seen outside the joints. Antibodies are created that lead to inflammation. Clients often report morning stiffness. Permanent damage can be avoided with aggressive, early treatment.

The nurse working in the rheumatology clinic assesses clients with rheumatoid arthritis (RA) for late manifestations. Which signs/symptoms are considered late manifestations of RA? (Select all that apply.) a. Anorexia b. Joint deformity c. Low-grade fever d. Weight loss

ANS: B, C, Late manifestations of RA include joint deformity, weight loss, organ involvement, osteoporosis, extreme fatigue, and anemia, among others. Anorexia and low-grade fever are both seen early in the course of the disease.

A nurse assesses a client who is recovering after a coronary catheterization. Which assessment findings in the first few hours after the procedure require immediate action by the nurse? (Select all that apply.) a. Blood pressure of 140/88 mm Hg b. Serum potassium of 2.9 mEq/L c. Warmth and redness at the site d. Expanding groin hematoma e. Rhythm changes on the cardiac monitor

ANS: B, D, E In the first few hours postprocedure, the nurse monitors for complications such as bleeding from the insertion site, hypotension, acute closure of the vessel, dye reaction, hypokalemia, and dysrhythmias. The client's blood pressure is slightly elevated but does not need immediate action. Warmth and redness at the site would indicate an infection, but this would not be present in the first few hours.

A nurse working in a geriatric clinic sees clients with "cold" symptoms and rhinitis. Which drug would be appropriate to teach these clients to take for their symptoms? a. Chlorpheniramine (Chlor-Trimeton) b. Diphenhydramine (Benadryl) c. Fexofenadine (Allegra) d. Hydroxyzine (Vistaril)

ANS: C First-generation antihistamines are not appropriate for use in the older population. These drugs include chlorpheniramine, diphenhydramine, and hydroxyzine. Fexofenadine is a second-generation antihistamine.

12.A nurse delegates care to an unlicensed assistive personnel (UAP). Which statement should the nurse include when delegating hygiene for a client who has a vascular access device? a. Provide a bed bath instead of letting the client take a shower. b. Use sterile technique when changing the dressing. c. Disconnect the intravenous fluid tubing prior to the clients bath. d. Use a plastic bag to cover the extremity with the device.

ANS: D The nurse should ask the UAP to cover the extremity with the vascular access device with a plastic bag or wrap to keep the dressing and site dry. The client may take a shower with a vascular device. The nurse should disconnect IV fluid tubing prior to the bath and change the dressing using sterile technique if necessary. These options are not appropriate to delegate to the UAP.

A client who is hospitalized with burns after losing the family home in a fire becomes angry and screams at a nurse when dinner is served late. How should the nurse respond? a. Do you need something for pain right now? b. Please stop yelling. I brought dinner as soon as I could. c. I suggest that you get control of yourself. d. You seem upset. I have time to talk if you'd like.

ANS: D Clients should be allowed to ventilate their feelings of anger and despair after a catastrophic event. The nurse establishes rapport through active listening and honest communication and by recognizing cues that the client wishes to talk. Asking whether the client is in pain as the first response closes the door to open communication and limits the clients options. Simply telling the client to stop yelling and to gain control does nothing to promote therapeutic communication.

The nurse in the rheumatology clinic is assessing clients with rheumatoid arthritis (RA). Which client should the nurse see first? a. Client taking celecoxib (Celebrex) and ranitidine (Zantac) b. Client taking etanercept (Enbrel) with a red injection site c. Client with a blood glucose of 190 mg/dL who is taking steroids d. Client with a fever and cough who is taking tofacitinib (Xeljanz)

ANS: D Tofacitinib carries a Food and Drug Administration black box warning about opportunistic infections, tuberculosis, and cancer. Fever and cough may indicate tuberculosis. Ranitidine is often taken with celecoxib, which can cause gastrointestinal distress. Redness and itchy rashes are frequently seen with etanercept injections. Steroids are known to raise blood glucose levels.

7. A nurse delegates care of a client in traction to an unlicensed assistive personnel (UAP). Which statement should the nurse include when delegating hygiene care for this client? a. Remove the traction when re-positioning the client. b. Inspect the clients skin when performing a bed bath. c. Provide pin care by using alcohol wipes to clean the sites. d. Ensure that the weights remain freely hanging at all times.

ANS: D Traction weights should be freely hanging at all times. They should not be lifted manually or allowed to rest on the floor. The client should remain in traction during hygiene activities. The nurse should assess the clients skin and provide pin and wound care for a client who is in traction; this should not be delegated to the UAP.

Which biological response modifier may cause chest pain and difficulty breathing during infusion? 1 Infliximab 2 Etanercept 3 Tocilizumab 4 Golimumab

Answer: 1

A student nurse is caring for an older adult who is recovering from hip replacement. The patient has recently spiked a high temperature and appears confused. For which complication should the nurse assess this patient? 1 Infection 2 Hemorrhage 3 Fat embolism 4 Hypovolemic shock

Answer: 1 Infection is common in older adults who are immobile due to trauma. Fat embolism would be due to fat being released from ruptured bone marrow. Hemorrhage and hypovolemic shock are complications of ruptured vascular tissue.

The nurse is teaching a patient with systemic lupus erythema (SLE) about self-management. Which statement by the patient indicates effective teaching? 1 "I will avoid sunlight." 2 "I will avoid Ivory soap." 3 "I will use clotrimazole on any skin lesions." 4 "I will use powder on any moist skin areas."

Answer: 1 "I will avoid sunlight." Patients with SLE should be instructed to avoid sunlight and any other type of ultraviolet lighting, including tanning beds, as they can cause exacerbations of the disease. Using a mild soap, such as Ivory, is gentle on the skin. Topical cortisone preparation, not clotrimazole (a topical antifungal), helps reduce inflammation and promote fading of skin lesions. Avoid powder and other drying agents, such as rubbing alcohol, due to the drying properties.

Which disease manifests as fever, weight loss, generalized weakness, fatigue, and the inflammation of the proximal interphalangeal (PIP) and metacarpophalangeal (MCP) joints of the hands? 1 Rheumatoid arthritis (RA) 2 Fibromyalgia syndrome (FMS) 3 Chronic fatigue syndrome (CFS) 4 Systemic lupus erythematosus (SLE)

Answer: 1 Rheumatoid arthritis (RA)

A client has been bedridden for several days after major abdominal surgery. What action does the nurse delegate to the UAP for deep vein thrombosis ( DVT ) prevention? a. Apply compression stockings. b. Assist with ambulation. c. Encourage coughing and deep breathing. d. Offer fluids frequently. e. Teach leg exercises.

Apply compression stockings Assist with ambulation Offer fluids frequently The UAP can apply compression stockings, assist with ambulation, and offer fluids frequently to help prevent DVT. The UAP can also encourage the client to do pulmonary exercises, but these do not decrease the risk of DVT. Teaching is a nursing function

A client is 4 hours postoperative after a femoropopliteal bypass. The client reports throbbing leg pain on the affected side, rated as a 7/10. What action by the nurse takes priority? a. Administer pain medication as ordered. b. Assess distal pulses and skin color. c. Document the findings in the client's chart. d. Notify the surgeon immediately.

Assess distal pulses and skin color

After teaching a client who has been diagnosed with hepatitis A, the nurse assesses the clients understanding. Which statement by the client indicates a correct understanding of the teaching? a. Some medications have been known to cause hepatitis A. b. I may have been exposed when we ate shrimp last weekend. c. I was infected with hepatitis A through a recent blood transfusion. d. My infection with Epstein-Barr virus can co-infect me with hepatitis A.

B

A nurse evaluates the following arterial blood gas values in a client: pH 7.48, PaO2 98 mm Hg, PaCO2 28 mm Hg, and HCO3- 22 mEq/L. Which client condition should the nurse correlate with these results? a. Diarrhea and vomiting for 36 hours b. Anxiety-induced hyperventilation c. Chronic obstructive pulmonary disease (COPD) d. Diabetic ketoacidosis and emphysema

B (The elevated pH level indicates alkalosis. The bicarbonate level is normal, and so is the oxygen partial pressure. Loss of carbon dioxide is the cause of the alkalosis, which would occur in response to hyperventilation. Diarrhea and vomiting would cause metabolic alterations, COPD would lead to respiratory acidosis, and the client with emphysema most likely would have combined metabolic acidosis on top of a mild, chronic respiratory acidosis.)

The nurse is caring for a client who has developed postoperative respiratory acidosis. Which of these interventions will the nurse use to help correct this problem? A. Medicate for pain. B. Encourage use of incentive spirometer. C.Perform fingerstick blood glucose. D.Encourage protein intake.

B (The intervention that will best help the client with postoperative respiratory acidosis is to encourage the client to use the incentive spirometer. Respiratory acidosis is caused by hypoventilation. Improving ventilation through lung expansion, suctioning, or upright positioning will help to resolve this.While pain medication may make use of the incentive spirometer easier, narcotic analgesics may suppress respirations and worsen acidosis. There is no indication the client has an unstable blood glucose level. Protein intake facilitates wound healing, not resolution of acidosis.)

A nurse observes that a clients anteroposterior (AP) chest diameter is the same as the lateral chest diameter Which question should the nurse ask the client in response to this finding? a. Are you taking any medications or herbal supplements? b. Do you have any chronic breathing problems? c. How often do you perform aerobic exercise? d. What is your occupation and what are your hobbies?

B The normal chest has a lateral diameter that is twice as large as the AP diameter. When the AP diameter approaches or exceeds the lateral diameter, the client is said to have a barrel chest. Most commonly, barrel chest occurs as a result of a long-term chronic airflow limitation problem, such as chronic obstructive pulmonary disease or severe chronic asthma. It can also be seen in people who have lived at a high altitude for many years. Therefore, an AP chest diameter that is the same as the lateral chest diameter should be rechecked but is not as indicative of underlying disease processes as an AP diameter that exceeds the lateral diameter. Medications, herbal supplements, and aerobic exercise are not associated with a barrel chest. Although occupation and hobbies may expose a client to irritants that can cause chronic lung disorders and barrel chest.

When caring for a client who has the following blood gas results, which of these interventions does the nurse plan to use to correct the acid base disturbance? pH 7.47—pCO2 37 mm hg- HCO3 30 mEq/L (30 mmol/L)—pO2 88mm hg A. Endotracheal suctioning B.Applying oxygen C.Administering an antiemetic D.Administering sodium bicarbonate

C (This blood gas demonstrates metabolic alkalosis typically caused by vomiting or NG suction. The client loses potassium and retains bicarbonate; an antiemetic will reduce vomiting and correct the imbalance.Endotracheal suction is indicated for retained respiratory secretions, which would be reflected as a respiratory acidosis. The pO2 is between 80 and 100 mmHg, which is normal, supplementary oxygen is not required. Sodium bicarbonate is used to treat metabolic acidosis in certain situations.)

A patient is diagnosed with osteoarthritis (OA). Which medication is considered the primary drug of choice for treatment? 1 Methotrexate (MTX) 2 Adalimumab 3 Acetaminophen 4 Leflunomide

Correct = 3 Acetaminophen (Methotrexate, adalimumab, and leflunomide are drugs used in the treatment of rheumatoid arthritis (RA).)

Which element is a risk factor for osteoarthritis (OA)? 1 Thin build 2 Obesity 3 Non-smoker 4 Male

Correct = 2 Obesity

Heberden's nodes

Swelling of distal interphalangeal finger joints

Which electrolyte is most affected by hyperglycemia? a. Sodium b. Chloride c. Potassium d. Magnesium

c. Potassium QUESTION FROM THE STUDY GUIDE

A patient with type 1 DM is taking a mixture of NPH and regular insulin at home. The patient has been NPO for surgery since midnight. What action does the nurse take regarding the patient's morning dose of insulin? a. Administer the dose that is routinely prescribed at home because the patient has type 1 DM and needs the insulin. b. Administer half the dose because the patient is NPO. c. Hold the insulin with all the other medications because the patient is NPO and there is no need for insulin. d. Contact the health care provider for an order regarding the insulin.

d. Contact the health care provider for an order regarding the insulin. QUESTION FROM THE STUDY GUIDE

Which laboratory test is the best indicator of a patient's average blood glucose level and/or compliance with the DM regimen over the last 3 months? a. Postprandial blood glucose test b. Oral glucose tolerance test (OGTT) c. Casual blood glucose test d. Glycosylated hemoglobin (HbA)

d. Glycosylated hemoglobin (HbA) QUESTION FROM THE STUDY GUIDE

Which are modifiable risk factors for type 2 DM? Select all that apply. a. Age b. Family history c. Working in a low-stress environment d. Maintaining ideal body weight e. Maintaining adequate physical activity

d. Maintaining ideal body weight e. Maintaining adequate physical activity QUESTION FROM THE STUDY GUIDE

Untreated hyperglycemia results in which condition? a. Respiratory acidosis b. Metabolic alkalosis c. Respiratory alkalosis d. Metabolic acidosis

d. Metabolic acidosis QUESTION FROM THE STUDY GUIDE

Early treatment of KDA and HHNS includes IV administration of which fluid? a. Glucagon b. Potassium c. Bicarbonate d. Normal saline

d. Normal saline QUESTION FROM THE STUDY GUIDE

A patient with type 2 DM, usually controlled with a second-generation sulfonylurea, develops a urinary tract infection. Due to the stress of the infection, the patient must be treated with insulin. What additional information about this treatment does the nurse relay to the patient? a. The sulfonylurea must be discontinued and insulin taken until the infection clears. b. Insulin will now be necessary to control the patient's diabetes for life. c. The sulfonylurea dose must be reduced until the infection clears. d. The insulin is necessary to supplement the second generation sulfonylurea until the infection clears.

d. The insulin is necessary to supplement the second generation sulfonylurea until the infection clears. QUESTION FROM THE STUDY GUIDE

A patient will be using an external insulin pump. What does the nurse tell the patient about the pump? a. SMBG levels can be done only twidce a day. b. The insulin supply must be replaced every 2-4 weeks. c. The pump's battery should be checked on a regular weekly schedule. d. The needle must be changed every two to three days.

d. The needle must be changed every two to three days. QUESTION FROM THE STUDY GUIDE

A nurse cares for a client who is recovering from laparoscopic cholecystectomy surgery. The client reports pain in the shoulder blades. How should the nurse respond? a. "Ambulating in the hallway twice a day will help." b. "I will apply a cold compress to the painful area on your back." c. "Drinking a warm beverage can relieve this referred pain." d. "You should cough and deep breathe every hour."

ANS: A The client who has undergone a laparoscopic cholecystectomy may report free air pain due to retention of carbon dioxide in the abdomen. The nurse assists the client with early ambulation to promote absorption of the carbon dioxide. Cold compresses and drinking a warm beverage would not be helpful. Coughing and deep breathing are important postoperative activities, but they are not related to discomfort from carbon dioxide.

A nurse cares for a client who has a new colostomy. Which action should the nurse take? a. Empty the pouch frequently to remove excess gas collection. b. Change the ostomy pouch and wafer every morning. c. Allow the pouch to completely fill with stool prior to emptying it. d. Use surgical tape to secure the pouch and prevent leakage.

ANS: A The nurse should empty the new ostomy pouch frequently because of excess gas collection, and empty the pouch when it is one-third to one-half full of stool. The ostomy pouch does not need to be changed every morning. Ostomy wafers with paste should be used to secure and seal the ostomy appliance; surgical tape should not be used.

After teaching a client who is recovering from laparoscopic cholecystectomy surgery, the nurse assesses the client's understanding. Which statement made by the client indicates a correct understanding of the teaching? a. "Drinking at least 2 liters of water each day is suggested." b. "I will decrease the amount of fatty foods in my diet." c. "Drinking fluids with my meals will increase bloating." d. "I will avoid concentrated sweets and simple carbohydrates."

ANS: B After cholecystectomy, clients need a nutritious diet without a lot of excess fat; otherwise a special diet is not recommended for most clients. Good fluid intake is healthy for all people but is not related to the surgery. Drinking fluids between meals helps with dumping syndrome, which is not seen with this procedure. Restriction of sweets is not required.

A nurse assesses a client with a mechanical bowel obstruction who reports intermittent abdominal pain. An hour later the client reports constant abdominal pain. Which action should the nurse take next? a. Administer intravenous opioid medications. b. Position the client with knees to chest. c. Insert a nasogastric tube for decompression. d. Assess the client's bowel sounds.

ANS: D A change in the nature and timing of abdominal pain in a client with a bowel obstruction can signal peritonitis or perforation. The nurse should immediately check for rebound tenderness and the absence of bowel sounds. The nurse should not medicate the client until the provider has been notified of the change in his or her condition. The nurse may help the client to the knee-chest position for comfort, but this is not the priority action. The nurse need not insert a nasogastric tube for decompression.

ANS: D Clients who have diverticular disease are prescribed a low-residue diet. Whole grains (rice pilaf), uncooked fruits and vegetables (salad, fresh fruit cup), and high-fiber foods (cup of bean soup) should be avoided with a low-residue diet. Canned or cooked vegetables are appropriate. Apple juice does not contain fiber and is acceptable for a low-residue diet.

After teaching a client with diverticular disease, a nurse assesses the client's understanding. Which menu selection made by the client indicates the client correctly understood the teaching? a. Roasted chicken with rice pilaf and a cup of coffee with cream b. Spaghetti with meat sauce, a fresh fruit cup, and hot tea c. Garden salad with a cup of bean soup and a glass of low-fat milk d. Baked fish with steamed carrots and a glass of apple juice

A provider prescribes Crotalidae Polyvalent Immune Fab (CroFab) for a client who is admitted after being bitten by a pit viper snake. Which assessment should the nurse complete prior to administering this medication? a. Assess temperature and for signs of fever. b. Check the clients creatinine kinase level. c. Ask about allergies to pineapple or papaya. d. Inspect the skin for signs of urticaria (hives).

ANS: C CroFab is an antivenom for pit viper snakebites. Clients should be assessed for hypersensitivity to bromelain (a pineapple derivative), papaya, and sheep protein prior to administration. During and after administration, the nurse should assess for urticaria, fever, and joint pain, which are signs of serum sickness.

ANS: B Adalimumab (Humira) is an immune modulator that must be given via subcutaneous injection. It does not need to be given with food or milk. Nausea and vomiting are two common side effects. Adalimumab can cause immune suppression, so clients receiving the medication should avoid large crowds and people who are sick, and should practice good handwashing.

After teaching a client who is prescribed adalimumab (Humira) for severe ulcerative colitis, the nurse assesses the client's understanding. Which statement made by the client indicates a need for additional teaching? a. "I will avoid large crowds and people who are sick." b. "I will take this medication with my breakfast each morning." c. "Nausea and vomiting are common side effects of this drug." d. "I must wash my hands after I play with my dog."

Before administering prednisone IV push to a middle-aged adult with rheumatoid arthritis (RA), the nurse notes that the patient's random blood glucose level is 139. Which action is most important for the nurse to take? 1 Instruct the patient to drink diet soda to prevent elevation of blood sugar. 2 Administer the prescribed prednisone on schedule. 3 Notify the health care provider of the random blood glucose result. 4 Review the patient's antinuclear antibody (ANA) level.

2 For this patient, giving the medication per schedule is essential in treating the disease. The blood glucose value will be monitored regularly because the patient is receiving prednisone. Blood sugar is only slightly elevated; encourage fluids other than soda (diet or otherwise).

A nurse assesses a 66-year-old client who is attempting to quit smoking. The client states, I started smoking at age 16, and smoked one pack each day until 10 years ago. Then I decreased to a half a pack per day. How many pack-years should the nurse document for this client? (Record your answer using a whole number.) pack-years

45 66 (current age) 16 (year started smoking) = 50 years of smoking. (40 years 1 pack per day) + (10 years 0.5 pack per day) = 45 pack-years.

Which statements by the client indicate good understanding of foot care in the peripheral vascular disease? a. "A good abrasive pumice stone will keep my feet soft." b. "I'll always wear shoes if I can buy cheap flip-flops." c. "I will keep my feet dry, especially between the toes." d. "Lotion is important to keep my feet smooth and soft." e. "Washing my feet in room-temperature water is best."

" I will keep my feet dry, especially between the toes " " Lotion is important to keep my feet smooth and soft " " Washing my feet in room temperature water is best " Good foot care includes appropriate hygiene and injury prevention. Keeping the feet dry; wearing good, comfortable shoes; using lotion; washing the feet in room-temperature water; and cutting the nails straight across are all important measures. Abrasive material such as pumice stones should not be used. Cheap flip-flops may not fit well and won't offer much protection against injury.

A client has been diagnosed with hypertension but does not take the antihypertensive medications because of a lack of symptoms. What response by the nurse is best? a. "Do you have trouble affording your medications?" b. "Most people with hypertension do not have symptoms." c. "You are lucky; most people get severe morning headaches." d. "You need to take your medicine or you will get kidney failure."

" Most people with hypertension do not have symptoms " Dehydration can cause the BUN to be elevated, but the elevation in creatinine is more specific for a kidney injury. The client does not necessarily need to Most people with hypertension are asymptomatic, although a small percentage do have symptoms such as headache. The nurse should explain this to the client. Asking about paying for medications is not related because the client has already admitted nonadherence. Threatening the client with possible complications will not increase compliance. admitted. A urinalysis may or may not be ordered.

A client is taking warfarin ( Coummadin ) and asks the nurse if taking St. John's wort is acceptable. What response by the nurse is best? a. "No, it may interfere with the warfarin." b. "There isn't any information about that." c. "Why would you want to take that?" d. "Yes, it is a good supplement for you."

" No, it may interfere with the warfarin " Many foods and drugs interfere with warfarin, St. John's wort being one of them. The nurse should advise the client against taking it. The other answers are not accurate.

A nurse is teaching a larger female client about alcohol intake and how it affects hypertension. The client asks if drinking two beers a night is an acceptable intake. What answer by the nurse is best? a. "No, women should only have one beer a day as a general rule." b. "No, you should not drink any alcohol with hypertension." c. "Yes, since you are larger, you can have more alcohol." d. "Yes, two beers per day is an acceptable amount of alcohol."

" No, women should only have one beer a day as a general rule " Alcohol intake should be limited to two drinks a day for men and one drink a day for women. A "drink" is classified as one beer, 1.5 ounces of hard liquor, or 5 ounces of wine. Limited alcohol intake is acceptable with hypertension. The woman's size does not matter.

A nurse evaluates a client's arterial blood gas values (ABGs): pH 7.30, PaO2 86 mm Hg, PaCO2 55 mm Hg, and HCO3- 22 mEq/L. Which intervention should the nurse implement first? a. Assess the airway. b. Administer prescribed bronchodilators. c. Provide oxygen. d. Administer prescribed mucolytics.

A (All interventions are important for clients with respiratory acidosis; this is indicated by the ABGs. However, the priority is assessing and maintaining an airway. Without a patent airway, other interventions will not be helpful.)

A nurse assesses a client who is experiencing an acid-base imbalance. The client's arterial blood gas values are pH 7.34, PaO2 88 mm Hg, PaCO2 38 mm Hg, and HCO3- 19 mEq/L. Which assessment should the nurse perform first? a. Cardiac rate and rhythm b. Skin and mucous membranes c. Musculoskeletal strength d. Level of orientation

A (Early cardiovascular changes for a client experiencing moderate acidosis include increased heart rate and cardiac output. As the acidosis worsens, the heart rate decreases and EKG changes will be present. Central nervous system and neuromuscular system changes do not occur with mild acidosis and should be monitored if the acidosis worsens. Skin and mucous membrane assessment is not a priority now, but will change as acidosis worsens.)

A nurse assesses a client who is admitted with an acid-base imbalance. The client's arterial blood gas values are pH 7.32, PaO2 85 mm Hg, PaCO2 34 mm Hg, and HCO3- 16 mEq/L. What action should the nurse take next? a. Assess client's rate, rhythm, and depth of respiration. b. Measure the client's pulse and blood pressure. c. Document the findings and continue to monitor. d. Notify the physician as soon as possible.

A (Progressive skeletal muscle weakness is associated with increasing severity of acidosis. Muscle weakness can lead to severe respiratory insufficiency. Acidosis does lead to dysrhythmias (due to hyperkalemia), but these would best be assessed with cardiac monitoring. Findings should be documented, but simply continuing to monitor is not sufficient. Before notifying the physician, the nurse must have more data to report.)

A nurse is caring for a client who has just experienced a 90-second tonic-clonic seizure. The client's arterial blood gas values are pH 6.88, PaO2 50 mm Hg, PaCO2 60 mm Hg, and HCO3- 22 mEq/L. Which action should the nurse take first? a. Apply oxygen by mask or nasal cannula. b. Apply a paper bag over the client's nose and mouth. c. Administer 50 mL of sodium bicarbonate intravenously. d. Administer 50 mL of 20% glucose and 20 units of regular insulin.

A (The client has experienced a combination of metabolic and acute respiratory acidosis through heavy skeletal muscle contractions and no gas exchange. When the seizures have stopped and the client can breathe again, the fastest way to return acid-base balance is to administer oxygen. Applying a paper bag over the client's nose and mouth would worsen the acidosis. Sodium bicarbonate should not be administered because the client's arterial bicarbonate level is normal. Glucose and insulin are administered together to decrease serum potassium levels. This action is not appropriate based on the information provided.)

When caring for a client with a burn injury and eschar banding the chest, the nurse plans to observe the client for which of these acid base disturbances? A.Respiratory acidosis B.Respiratory alkalosis C.Metabolic acidosis D.Metabolic alkalosis

A (The nurse plans to observe the client with a burn injury and eschar banding the chest for respiratory acidosis related to decreased chest excursion. Circumferential eschar will result in hypoventilation, accumulation of carbon dioxide and resulting respiratory acidosis.Respiratory alkalosis is caused by hyperventilation, increased rate or depth of breathing, causing carbon dioxide to be eliminated in excess. Metabolic acid base disturbances are usually caused by renal issues.)

The nurse is caring for a group of clients. Which client will the nurse carefully observe for signs and symptoms of hyperkalemia? A. The client who has metabolic acidosis B.The client receiving total parenteral nutrition C.The client who has profuse vomiting D.The client taking a thiazide diuretic

A (The nurse would carefully observe for signs of metabolic acidosis in a client with hyperkalemia. Hyperkalemia occurs as the body attempts to buffer the acidosis by moving hydrogen ions into the cells. An equal number of potassium ions move from the cells into the blood to maintain intracellular electroneutrality, resulting in hyperkalemia.The client receiving TPN is at risk for metabolic alkalosis due to an increase in base components. Metabolic alkalosis is associated with hypochloremia rather than hyperkalemia. The client with profuse vomiting or taking a diuretic is also at risk for metabolic alkalosis and hypokalemia.)

A nurse is caring for a client who has chronic emphysema and is receiving oxygen therapy at 6 L/min via nasal cannula. The following clinical data are available: Arterial Blood Gases Vital Signs pH = 7.28 Pulse rate = 96 beats/min PaO2 = 85 mm Hg Blood pressure = 135/45 PaCO2 = 55 mm Hg Respiratory rate = 6 breaths/min HCO3- = 26 mEq/L O2 saturation = 88% Which action should the nurse take first? a. Notify the Rapid Response Team and provide ventilation support. b. Change the nasal cannula to a mask and reassess in 10 minutes. c. Place the client in Fowler's position if he or she is able to tolerate it. d. Decrease the flow rate of oxygen to 2 to 4 L/min, and reassess.

A (The primary trigger for respiration in a client with chronic respiratory acidosis is a decreased arterial oxygen level (hypoxic drive). Oxygen therapy can inhibit respiratory efforts in this case, eventually causing respiratory arrest and death. The nurse could decrease the oxygen flow rate; eventually, this might improve the client's respiratory rate, but the priority action would be to call the Rapid Response Team whenever a client with chronic carbon dioxide retention has a respiratory rate less than 10 breaths/min. Changing the cannula to a mask does nothing to improve the client's hypoxic drive, nor would it address the client's most pressing need. Positioning will not help the client breathe at a normal rate or maintain client safety.)

A nurse assesses a client with diabetes mellitus who is admitted with an acid-base imbalance. The client's arterial blood gas values are pH 7.36, PaO2 98 mm Hg, PaCO2 33 mm Hg, and HCO3- 18 mEq/L. Which manifestation should the nurse identify as an example of the client's compensation mechanism? a. Increased rate and depth of respirations b. Increased urinary output c. Increased thirst and hunger d. Increased release of acids from the kidneys

A (This client has metabolic acidosis. The respiratory system compensates by increasing its activity and blowing off excess carbon dioxide. Increased urinary output, thirst, and hunger are manifestations of hyperglycemia but are not compensatory mechanisms for acid-base imbalances. The kidneys do not release acids.)

The nurse is caring for a client with sepsis and impending septic shock. Which of these interventions will help prevent lactic acidosis? A.Ensure adequate oxygenation B.Restrict carbohydrates C.Supplement potassium D.Monitor hemoglobin

A (When caring for a client with sepsis and impending shock the nurse will ensure adequate oxygenation to help prevent lactic acidosis. Cellular metabolism under anaerobic (no oxygen) conditions forms lactic acid. Shock states are due to a lack of cellular perfusion and delivery of oxygen to the tissues. Providing adequate oxygenation and perfusion will help to reverse the need for the body to make ATP without oxygen which causes lactic acid to accumulate.Carbohydrate metabolism forms carbon dioxide (CO2) and carbohydrate restriction will not prevent lactic acidosis, a form of metabolic acidosis. Supplementing potassium may worsen hyperkalemia, as this is an expected finding during episodes of metabolic acidosis. While hemoglobin is a weak buffer, monitoring the value will not prevent an acid-base disturbance.)

A nurse plans care for a client who is experiencing dyspnea and must stop multiple times when climbing a flight of stairs. Which intervention should the nurse include in this clients plan of care? a. Assistance with activities of daily living b. Physical therapy activities every day c. Oxygen therapy at 2 liters per nasal cannula d. Complete bedrest with frequent repositioning

A A client with dyspnea and difficulty completing activities such as climbing a flight of stairs has class III dyspnea. The nurse should provide assistance with activities of daily living. These clients should be encouraged to participate in activities as tolerated. They should not be on complete bedrest, may not be able to tolerate daily physical therapy, and only need oxygen if hypoxia is present.

A nurse plans care for a client who is at high risk for a pulmonary infection. Which interventions should the nurse include in this clients plan of care? (Select all that apply.) a. Encourage deep breathing and coughing. b. Implement an air mattress overlay. c. Ambulate the client three times each day. d. Provide a diet high in protein and vitamins. e. Administer acetaminophen (Tylenol) twice daily.

A, C, D Regular pulmonary hygiene and activities to maintain health and fitness help to maximize functioning of the respiratory system and prevent infection. A client at high risk for a pulmonary infection may need a specialty bed to help with postural drainage or percussion; this would not include an air mattress overlay, which is used to prevent pressure ulcers. Tylenol would not decrease the risk of a pulmonary infection.

A nurse assesses a client who is prescribed varenicline (Chantix) for smoking cessation. Which manifestations should the nurse identify as adverse effects of this medication? (Select all that apply.) a. Visual hallucinations b. Tachycardia c. Decreased cravings d. Impaired judgment e. Increased thirst

A, D Varenicline (Chantix) has a black box warning stating that the drug can cause manic behavior and hallucinations. The nurse should assess for changes in behavior and thought processes, including impaired judgment and visual hallucinations. Tachycardia and increased thirst are not adverse effects of this medication. Decreased cravings is a therapeutic response to this medication.

3. While obtaining a clients healthy history, the client states, I am allergic to avocados. Which responses by the nurse are best? (Select all that apply) a. What response do you have when you eat avocados? b. I will remove any avocados that are on your lunch tray. c. When was the last time you ate foods containing avocados? d. I will document this in your record so all of your probiders will know e. Have you ever been treated for this allergic reaction?

A, D, E Nurses should assess clients who have allergies for the specific cause, treatment, and response to treatment. The nurse should also document the allergies in a prominent place in the clients medical record. The nurse should collaborate with food services to ensure no avocados are placed on the clients meal trays. Asking about the last time the client ate avocados does not provide any pertinent information for the clients plan of care.

A nurse teaches a client who is interested in smoking cessation. Which statements should the nurse include in this clients teaching? (Select all that apply.) a. Find an activity that you enjoy and will keep your hands busy. b. Keep snacks like potato chips on hand to nibble on. c. Identify a punishment for yourself in case you backslide. d. Drink at least eight glasses of water each day. e. Make a list of reasons you want to stop smoking.

A, D, E The nurse should teach a client who is interested in smoking cessation to find an activity that keeps the hands busy, to keep healthy snacks on hand to nibble on, to drink at least 8 glasses of water each day, and to make a list of reasons for quitting smoking. The nurse should also encourage the client not to be upset if he or she backslides and has a cigarette.

A nurse assesses a client who is receiving total parenteral nutrition. For which adverse effects related to an acid-base imbalance should the nurse assess? (Select all that apply.) a. Positive Chvostek's sign b. Elevated blood pressure c. Bradycardia d. Increased muscle strength e. Anxiety and irritability

A, E (A client receiving total parenteral nutrition is at risk for metabolic alkalosis. Manifestations of metabolic alkalosis include positive Chvostek's sign, normal or low blood pressure, increased heart rate, skeletal muscle weakness, and anxiety and irritability.)

What is the first step in emergency care of a patient with an extremity fracture? Assess the patient's airway, breathing, and circulation. Remove the patient's clothing to inspect the affected area. Apply direct pressure on the area if there is bleeding and pressure over the proximal artery nearest the fracture. Immobilize the extremity by splinting, including joints above and below the fracture site.

ABCs The first step in emergency care is to assess the patient's airway, breathing, and circulation to identify any life-threatening conditions before performing a quick head-to-toe assessment. If the person is clothed, the clothing will need to be cut away from the extremity, and any jewelry should be removed from the extremity after the primary survey has been completed. This can happen after the initial assessment. At this point, direct pressure on the area and digital pressure over the artery above the fracture should be applied. Once the secondary survey has been completed and the patient has been stabilized by the prehospital team, pain should be managed and then splinting should be carried out to prevent further tissue damage, reduce pain, and increase circulation.

A nurse auscultates a client's lung fields. Which pathophysiologic process should the nurse associate with this breath sound? (Click the media button to hear the audio clip.) a. Inflammation of the pleura b. Constriction of the bronchioles c. Upper airway obstruction d. Pulmonary vascular edema

ANS: A A pleural friction rub can be heard when the pleura is inflamed and rubbing against the lung wall. The other pathophysiologic processes would not cause a pleural friction rub. Constriction of the bronchioles may be heard as a wheeze, upper airway obstruction may be heard as stridor, and pulmonary vascular edema may be heard as crackles.

A nurse cares for a client with a 40-year smoking history who is experiencing distended neck veins and dependent edema. Which physiologic process should the nurse correlate with this client's history and clinical manifestations? a. Increased pulmonary pressure creating a higher workload on the right side of the heart b. Exposure to irritants resulting in increased inflammation of the bronchi and bronchioles c. Increased number and size of mucus glands producing large amounts of thick mucus d. Left ventricular hypertrophy creating a decrease in cardiac output

ANS: A Smoking increases pulmonary hypertension, resulting in cor pulmonale, or right-sided heart failure. Increased pressures in the lungs make it more difficult for blood to flow through the lungs. Blood backs up into the right side of the heart and then into the peripheral venous system, creating distended neck veins and dependent edema. Inflammation in bronchi and bronchioles creates an airway obstruction which manifests as wheezes. Thick mucus in the lungs has no impact on distended neck veins and edema. Left ventricular hypertrophy is associated with left heart failure and is not caused by a 40-year smoking history.

A nurse auscultates a client's lung fields. Which action should the nurse take based on the lung sounds? (Click the media button to hear the audio clip.) a. Assess for airway obstruction. b. Initiate oxygen therapy. c. Assess vital signs. d. Elevate the client's head.

ANS: A Stridor is the sound heard, and it indicates severe airway constriction. The nurse must administer a bronchodilator to get air into the lungs. Administering oxygen, assessing vital signs, and elevating the client's head will not help until the client's airways are open.

A nurse assesses a client who is prescribed fluticasone (Flovent) and notes oral lesions. Which action should the nurse take? a. Encourage oral rinsing after fluticasone administration. b. Obtain an oral specimen for culture and sensitivity. c. Start the client on a broad-spectrum antibiotic. d. Document the finding as a known side effect.

ANS: A The drug reduces local immunity and increases the risk for local infection, especially Candida albicans. Rinsing the mouth after using the inhaler will decrease the risk for developing this infection. Use of mouthwash and broad-spectrum antibiotics is not warranted in this situation. The nurse should document the finding, but the best action to take is to have the client start rinsing his or her mouth after using fluticasone. An oral specimen for culture and sensitivity will not provide information necessary to care for this client.

The nurse instructs a client on the steps needed to obtain a peak expiratory flow rate. In which order should these steps occur? 1. "Take as deep a breath as possible." 2. "Stand up (unless you have a physical disability)." 3. "Place the meter in your mouth, and close your lips around the mouthpiece." 4. "Make sure the device reads zero or is at base level." 5. "Blow out as hard and as fast as possible for 1 to 2 seconds." 6. "Write down the value obtained." 7. "Repeat the process two additional times, and record the highest number in your chart." a. 4, 2, 1, 3, 5, 6, 7 b. 3, 4, 1, 2, 5, 7, 6 c. 2, 1, 3, 4, 5, 6, 7 d. 1, 3, 2, 5, 6, 7, 4

ANS: A The proper order for obtaining a peak expiratory flow rate is as follows. Make sure the device reads zero or is at base level. The client should stand up (unless he or she has a physical disability). The client should take as deep a breath as possible, place the meter in the mouth, and close the lips around the mouthpiece. The client should blow out as hard and as fast as possible for 1 to 2 seconds. The value obtained should be written down. The process should be repeated two more times, and the highest of the three numbers should be recorded in the client's chart.

3.A nurse teaches a client who is being discharged home with a peripherally inserted central catheter (PICC). Which statement should the nurse include in this clients teaching? a. Avoid carrying your grandchild with the arm that has the central catheter. b. Be sure to place the arm with the central catheter in a sling during the day. c. Flush the peripherally inserted central catheter line with normal saline daily. d. You can use the arm with the central catheter for most activities of daily living.

ANS: A A properly placed PICC (in the antecubital fossa or the basilic vein) allows the client considerable freedom of movement. Clients can participate in most activities of daily living; however, heavy lifting can dislodge the catheter or occlude the lumen. Although it is important to keep the insertion site and tubing dry, the client can shower. The device is flushed with heparin.

A nurse admits a client from the emergency department. Client data are listed below: History Physical Assessment Laboratory Values 70 years of age History of diabetes On insulin twice a day Reports new-onset dyspnea and productive cough Crackles and rhonchi heard throughout the lungs Dullness to percussion LLL Afebrile Oriented to person only WBC: 5,200/mm3 PaO2 on room air 65 mm Hg What action by the nurse is the priority? a. Administer oxygen at 4 liters per nasal cannula. b. Begin broad-spectrum antibiotics. c. Collect a sputum sample for culture. d. Start an IV of normal saline at 50 mL/hr.

ANS: A All actions are appropriate for this client who has manifestations of pneumonia. However, airway and breathing come first, so begin oxygen administration and titrate it to maintain saturations greater than 95%. Start the IV and collect a sputum culture, and then begin antibiotics.

26.A preoperative nurse assesses a client who has type 1 diabetes mellitus prior to a surgical procedure. The clients blood glucose level is 160 mg/dL. Which action should the nurse take? a. Document the finding in the clients chart. b. Administer a bolus of regular insulin IV. c. Call the surgeon to cancel the procedure. d. Draw blood gases to assess the metabolic state.

ANS: A Clients who have type 1 diabetes and are having surgery have been found to have fewer complications, lower rates of infection, and better wound healing if blood glucose levels are maintained at between 140 and 180 mg/dL throughout the perioperative period. The nurse should document the finding and proceed with other operative care. The need for a bolus of insulin, canceling the procedure, or drawing arterial blood gases is not required.

11. While assessing a client who has facial trauma, the nurse auscultates stridor. The client is anxious and restless. Which action should the nurse take first? a. Contact the provider and prepare for intubation. b. Administer prescribed albuterol nebulizer therapy. c. Place the client in high-Fowlers position. d. Ask the client to perform deep-breathing exercises.

ANS: A Facial and neck tissue edema can occur in clients with facial trauma. Airway patency is the highest priority. Clients who experience stridor and hypoxia, manifested by anxiety and restlessness, should be immediately intubated to ensure airway patency. Albuterol decreases bronchi and bronchiole inflammation, not facial and neck edema. Although putting the client in high-Fowlers position and asking the client to perform breathing exercises may temporarily improve the clients comfort, these actions will not decrease the underlying problem or improve airway patency.

10.A nurse responds to an IV pump alarm related to increased pressure. Which action should the nurse take first? a. Check for kinking of the catheter. b. Flush the catheter with a thrombolytic enzyme. c. Get a new infusion pump. d. Remove the IV catheter.

ANS: A Fluid flow through the infusion system requires that pressure on the external side be greater than pressure at the catheter tip. Fluid flow can be slowed for many reasons. A common reason, and one that is easy to correct, is a kinked catheter. If this is not the cause of the pressure alarm, the nurse may have to ascertain whether a clot has formed inside the catheter lumen, or if the pump is no longer functional. Removal of the IV catheter and placement of a new IV catheter should be completed when no other option has resolved the problem.

The emergency department (ED) manager is reviewing client charts to determine how well the staff performs when treating clients with community-acquired pneumonia. What outcome demonstrates that goals for this client type have been met? a. Antibiotics started before admission b. Blood cultures obtained within 20 minutes c. Chest x-ray obtained within 30 minutes d. Pulse oximetry obtained on all clients

ANS: A Goals for treatment of community-acquired pneumonia include initiating antibiotics prior to inpatient admission or within 6 hours of presentation to the ED. Timely collection of blood cultures, chest x-ray, and pulse oximetry are important as well but do not coincide with established goals.

24.After teaching a client who is recovering from pancreas transplantation, the nurse assesses the clients understanding. Which statement made by the client indicates a need for additional education? a. If I develop an infection, I should stop taking my corticosteroid. b. If I have pain over the transplant site, I will call the surgeon immediately. c. I should avoid people who are ill or who have an infection. d.

ANS: A Immunosuppressive agents should not be stopped without the consultation of the transplantation physician, even if an infection is present. Stopping immunosuppressive therapy endangers the transplanted organ. The other statements are correct. Pain over the graft site may indicate rejection. Anti-rejection drugs cause immunosuppression, and the client should avoid crowds and people who are ill. Changing the routine of anti-rejection medications may cause them to not work optimally.

42.A nurse prepares to administer insulin to a client at 1800. The clients medication administration record contains the following information: Insulin glargine: 12 units daily at 1800 Regular insulin: 6 units QID at 0600, 1200, 1800, 2400 Based on the clients medication administration record, which action should the nurse take? a. Draw up and inject the insulin glargine first, and then draw up and inject the regular insulin. b. Draw up and inject the insulin glargine first, wait 20 minutes, and then draw up and inject the regular insulin. c. First draw up the dose of regular insulin, then draw up the dose of insulin glargine in the same syringe, mix, and inject the two insulins together. d. First draw up the dose of insulin glargine, then draw up the dose of regular insulin in the same syringe, mix, and inject the two insulins together.

ANS: A Insulin glargine must not be diluted or mixed with any other insulin or solution. Mixing results in an unpredictable alteration in the onset of action and time to peak action. The correct instruction is to draw up and inject first the glargine and then the regular insulin right afterward.

A client admitted for pneumonia has been tachypneic for several days. When the nurse starts an IV to give fluids, the client questions this action, saying "I have been drinking tons of water. How am I dehydrated?" What response by the nurse is best? a. "Breathing so quickly can be dehydrating." b. "Everyone with pneumonia is dehydrated." c. "This is really just to administer your antibiotics." d. "Why do you think you are so dehydrated?"

ANS: A Tachypnea and mouth breathing, both seen in pneumonia, increase insensible water loss and can lead to a degree of dehydration. The other options do not give the client useful information.

16.A medical-surgical nurse is concerned about the incidence of complications related to IV therapy, including bloodstream infection. Which intervention should the nurse suggest to the management team to make the biggest impact on decreasing complications? a. Initiate a dedicated team to insert access devices. b. Require additional education for all nurses. c. Limit the use of peripheral venous access devices. d. Perform quality control testing on skin preparation products.

ANS: A The Centers for Disease Control and Prevention recommends having a dedicated IV team to reduce complications, save money, and improve client satisfaction and outcomes. In-service education would always be helpful, but it would not have the same outcomes as an IV team. Limiting IV starts to the most experienced nurses does not allow newer nurses to gain this expertise. The quality of skin preparation products is only one aspect of IV insertion that could contribute to infection.

A nurse wants to become involved in community disaster preparedness and is interested in helping set up and staff first aid stations or community acute care centers in the event of a disaster. Which organization is the best fit for this nurses interests? a. The Medical Reserve Corps b. The National Guard c. The health department d. A Disaster Medical

ANS: A The Medical Reserve Corps (MRC) consists of volunteer medical and public health care professionals who support the community during times of need. They may help staff hospitals, establish first aid stations or special needs shelters, or set up acute care centers in the community. The National Guard often performs search and rescue operations and law enforcement. The health department focuses on communicable disease tracking, treatment, and prevention. A Disaster Medical Assistance Team is deployed to a disaster area for up to 72 hours, providing many types of relief services.

After teaching a client with diabetes mellitus to inject insulin, the nurse assesses the clients understanding. Which statement made by the client indicates a need for additional teaching? a. The lower abdomen is the best location because it is closest to the pancreas. b. I can reach my thigh the best, so I will use the different areas of my thighs. c. By rotating the sites in one area, my chance of having a reaction is decreased. d. Changing injection sites from the thigh to the arm will change absorption rates.

ANS: A The abdominal site has the fastest rate of absorption because of blood vessels in the area, not because of its proximity to the pancreas. The other statements are accurate assessments of insulin administration.

2. A nurse assesses a client who has a nasal fracture. The client reports constant nasal drainage, a headache, and difficulty with vision. Which action should the nurse take next? a. Collect the nasal drainage on a piece of filter paper. b. Encourage the client to blow his or her nose. c. Perform a test focused on a neurologic examination. d. Palpate the nose, face, and neck.

ANS: A The client with nasal drainage after facial trauma could have a skull fracture that has resulted in leakage of cerebrospinal fluid (CSF). CSF can be differentiated from regular drainage by the fact that it forms a halo when dripped on filter paper. The other actions would be appropriate but are not as high a priority as assessing for CSF. A CSF leak would increase the clients risk for infection.

14.A nurse assesses a client with diabetes mellitus and notes the client only responds to a sternal rub by moaning, has capillary blood glucose of 33 g/dL, and has an intravenous line that is infiltrated with 0.45% normal saline. Which action should the nurse take first? a. Administer 1 mg of intramuscular glucagon. b. Encourage the client to drink orange juice. c. Insert a new intravenous access line. d. Administer 25 mL dextrose 50% (D50) IV push.

ANS: A The clients blood glucose level is dangerously low. The nurse needs to administer glucagon IM immediately to increase the clients blood glucose level. The nurse should insert a new IV after administering the glucagon and can use the new IV site for future doses of D50 if the clients blood glucose level does not rise. Once the client is awake, orange juice may be administered orally along with a form of protein such as a peanut butter.

A client has been diagnosed with tuberculosis (TB). What action by the nurse takes highest priority? a. Educating the client on adherence to the treatment regimen b. Encouraging the client to eat a well-balanced diet c. Informing the client about follow-up sputum cultures d. Teaching the client ways to balance rest with activity

ANS: A The treatment regimen for TB ranges from 6 to 12 months, making adherence problematic for many people. The nurse should stress the absolute importance of following the treatment plan for the entire duration of prescribed therapy. The other options are appropriate topics to educate this client on but do not take priority.

31.A nurse assesses a client who has diabetes mellitus and notes the client is awake and alert, but shaky, diaphoretic, and weak. Five minutes after administering a half-cup of orange juice, the clients clinical manifestations have not changed. Which action should the nurse take next? a. Administer another half-cup of orange juice. b. Administer a half-ampule of dextrose 50% intravenously. c. Administer 10 units of regular insulin subcutaneously. d. Administer 1 mg of glucagon intramuscularly.

ANS: A This client is experiencing mild hypoglycemia. For mild hypoglycemic manifestations, the nurse should administer oral glucose in the form of orange juice. If the symptoms do not resolve immediately, the treatment should be repeated. The client does not need intravenous dextrose, insulin, or glucagon.

A nurse wants to become part of a Disaster Medical Assistance Team (DMAT) but is concerned about maintaining licensure in several different states. Which statement best addresses these concerns? a. Deployed DMAT providers are federal employees, so their licenses are good in all 50 states. b. The government has a program for quick licensure activation wherever you are deployed. c. During a time of crisis, licensure issues would not be the governments priority concern. d. If you are deployed, you will be issued a temporary license in the state in which you are working.

ANS: A When deployed, DMAT health care providers are acting as agents of the federal government, and so are considered federal employees. Thus their licenses are valid in all 50 states. Licensure is an issue that the government would be concerned with, but no programs for temporary licensure or rapid activation are available.

A nurse assesses a client recovering from a cerebral angiography via the client's right femoral artery. Which assessment should the nurse complete? a. Palpate bilateral lower extremity pulses. b. Obtain orthostatic blood pressure readings. c. Perform a funduscopic examination. d. Assess the gag reflex prior to eating.

ANS: A Cerebral angiography is performed by threading a catheter through the femoral or brachial artery. The extremity is kept immobilized after the procedure. The nurse checks the extremity for adequate circulation by noting skin color and temperature, presence and quality of pulses distal to the injection site, and capillary refill. Clients usually are on bedrest; therefore, orthostatic blood pressure readings cannot be performed. The funduscopic examination would not be affected by cerebral angiography. The client is given analgesics but not conscious sedation; therefore, the client's gag reflex would not be compromised. DIF: Applying/Application REF: 845 KEY: Assessment/diagnostic examination MSC: Integrated Process: Nursing Process: Assessment NOT: Client Needs Category: Physiological Integrity: Reduction of Risk Potential

The nurse is working with a client who has rheumatoid arthritis (RA). The nurse has identified the priority problem of poor body image for the client. What finding by the nurse indicates goals for this client problem are being met? a. Attends meetings of a book club b. Has a positive outlook on life c. Takes medication as directed d. Uses assistive devices to protect joints

ANS: A All of the activities are appropriate for a client with RA. Clients who have a poor body image are often reluctant to appear in public, so attending public book club meetings indicates that goals for this client problem are being met.

A nurse cares for a client admitted from a nursing home after several recent falls. What prescription should the nurse complete first? a. Obtain urine sample for culture and sensitivity. b. Administer intravenous antibiotics. c. Encourage protein intake and additional fluids. d. Consult physical therapy for gait training.

ANS: A Although all interventions are or might be important, obtaining a urine sample for urinalysis takes priority. Often urinary tract infection (UTI) symptoms in older adults are atypical, and a UTI may present with new onset of confusion or falling. The urine sample should be obtained before starting antibiotics. Dietary requirements and gait training should be implemented after obtaining the urine sample.

A nurse assesses clients on a cardiac unit. Which client should the nurse identify as being at greatest risk for the development of left-sided heart failure? a. A 36-year-old woman with aortic stenosis b. A 42-year-old man with pulmonary hypertension c. A 59-year-old woman who smokes cigarettes daily d. A 70-year-old man who had a cerebral vascular accident

ANS: A Although most people with heart failure will have failure that progresses from left to right, it is possible to have left-sided failure alone for a short period. It is also possible to have heart failure that progresses from right to left. Causes of left ventricular failure include mitral or aortic valve disease, coronary artery disease, and hypertension. Pulmonary hypertension and chronic cigarette smoking are risk factors for right ventricular failure. A cerebral vascular accident does not increase the risk of heart failure.

A nurse cares for a client with an increased blood urea nitrogen (BUN)/creatinine ratio. Which action should the nurse take first? a. Assess the client's dietary habits. b. Inquire about the use of nonsteroidal anti-inflammatory drugs (NSAIDs). c. Hold the client's metformin (Glucophage). d. Contact the health care provider immediately.

ANS: A An elevated BUN/creatinine ratio is often indicative of dehydration, urinary obstruction, catabolism, or a high-protein diet. The nurse should inquire about the client's dietary habits. Kidney damage related to NSAID use most likely would manifest with elevations in both BUN and creatinine, but no change in the ratio. The nurse should obtain more assessment data before holding any medications or contacting the provider.

A nurse is teaching a client with heart failure who has been prescribed enalapril (Vasotec). Which statement should the nurse include in this client's teaching? a. "Avoid using salt substitutes." b. "Take your medication with food." c. "Avoid using aspirin-containing products." d. "Check your pulse daily."

ANS: A Angiotensin-converting enzyme (ACE) inhibitors such as enalapril inhibit the excretion of potassium. Hyperkalemia can be a life-threatening side effect, and clients should be taught to limit potassium intake. Salt substitutes are composed of potassium chloride. ACE inhibitors do not need to be taken with food and have no impact on the client's pulse rate. Aspirin is often prescribed in conjunction with ACE inhibitors and is not contraindicated.

11. A home health nurse assesses a client with diabetes who has a new cast on the arm. The nurse notes the clients fingers are pale, cool, and slightly swollen. Which action should the nurse take first? a. Raise the arm above the level of the heart. b. Encourage range of motion. c. Apply heat to the affected hand. d. Bivalve the cast to decrease pressure.

ANS: A Arm casts can impair circulation when the arm is in the dependent position. The nurse should immediately elevate the arm above the level of the heart, ensuring that the hand is above the elbow, and should re-assess the extremity in 15 minutes. If the fingers are warmer and less swollen, the cast is not too tight and adjustments do not need to be made, but a sling should be worn when the client is upright. Encouraging range of motion would not assist the client as much as elevating the arm. Heat would cause increased edema and should not be used. If the cast is confirmed to be too tight, it could be bivalved.

A nurse admits a client who is experiencing an exacerbation of heart failure. Which action should the nurse take first? a. Assess the client's respiratory status. b. Draw blood to assess the client's serum electrolytes. c. Administer intravenous furosemide (Lasix). d. Ask the client about current medications.

ANS: A Assessment of respiratory and oxygenation status is the priority nursing intervention for the prevention of complications. Monitoring electrolytes, administering diuretics, and asking about current medications are important but do not take priority over assessing respiratory status.

28. After a stroke, a client has ataxia. What intervention is most appropriate to include on the clients plan of care? a. Ambulate only with a gait belt. b. Encourage double swallowing. c. Monitor lung sounds after eating. d. Perform post-void residuals.

ANS: A Ataxia is a gait disturbance. For the clients safety, he or she should have assistance and use a gait belt when ambulating. Ataxia is not related to swallowing, aspiration, or voiding.

14. A nurse assesses a client with tachycardia. Which clinical manifestation requires immediate intervention by the nurse? a. Mid-sternal chest pain b. Increased urine output c. Mild orthostatic hypotension d. P wave touching the T wave

ANS: A Chest pain, possibly angina, indicates that tachycardia may be increasing the clients myocardial workload and oxygen demand to such an extent that normal oxygen delivery cannot keep pace. This results in myocardial hypoxia and pain. Increased urinary output and mild orthostatic hypotension are not life-threatening conditions and therefore do not require immediate intervention. The P wave touching the T wave indicates significant tachycardia and should be assessed to determine the underlying rhythm and cause; this is an important assessment but is not as critical as chest pain, which indicates cardiac cell death.

A client who has had systemic lupus erythematosus (SLE) for many years is in the clinic reporting hip pain with ambulation. Which action by the nurse is best? a. Assess medication records for steroid use. b. Facilitate a consultation with physical therapy. c. Measure the range of motion in both hips. d. Notify the health care provider immediately.

ANS: A Chronic steroid use is seen in clients with SLE and can lead to osteonecrosis (bone necrosis). The nurse should determine if the client has been taking a steroid. Physical therapy may be beneficial, but there is not enough information about the client yet. Measuring range of motion is best done by the physical therapist. Notifying the provider immediately is not warranted.

A client takes celecoxib (Celebrex) for chronic osteoarthritis in multiple joints. After a knee replacement, the health care provider has prescribed morphine sulfate for postoperative pain relief. The client also requests the celecoxib in addition to the morphine. What action by the nurse is best? a. Consult with the health care provider about administering both drugs to the client. b. Inform the client that the celecoxib will be started when he or she goes home. c. Teach the client that, since morphine is stronger, celecoxib is not needed. d. Tell the client he or she should not take both drugs at the same time.

ANS: A Despite getting an opioid analgesic for postoperative pain, the nurse should be aware that the client may be on other medications for arthritis in other joints. The nonsteroidal anti-inflammatory drug celecoxib will also help with the postoperative pain. The nurse should consult the provider about continuing the celecoxib while the client is in the hospital. The other responses are not warranted, as the client should be restarted on this medication postoperatively.

An emergency room nurse obtains the health history of a client. Which statement by the client should alert the nurse to the occurrence of heart failure? a. "I get short of breath when I climb stairs." b. "I see halos floating around my head." c. "I have trouble remembering things." d. "I have lost weight over the past month."

ANS: A Dyspnea on exertion is an early manifestation of heart failure and is associated with an activity such as stair climbing. The other findings are not specific to early occurrence of heart failure.

12. A client is taking furosemide (Lasix) 40 mg/day for management of chronic kidney disease (CKD). To detect the positive effect of the medication, what action of the nurse is best? a. Obtain daily weights of the client. b. Auscultate heart and breath sounds. c. Palpate the client's abdomen. d. Assess the client's diet history.

ANS: A Furosemide (Lasix) is a loop diuretic that helps reduce fluid overload and hypertension in clients with early stages of CKD. One kilogram of weight equals about 1 liter of fluid retained in the client, so daily weights are necessary to monitor the response of the client to the medication. Heart and breath sounds should be assessed if there is fluid retention, as in heart failure. Palpation of the client's abdomen is not necessary, but the nurse should check for edema. The diet history of the client would be helpful to assess electrolyte replacement since potassium is lost with this diuretic, but this does not assess the effect of the medication.

24. A nurse is providing community screening for risk factors associated with stroke. Which client would the nurse identify as being at highest risk for a stroke? a. A 27-year-old heavy cocaine user b. A 30-year-old who drinks a beer a day c. A 40-year-old who uses seasonal antihistamines d. A 65-year-old who is active and on no medications

ANS: A Heavy drug use, particularly cocaine, is a risk factor for stroke. Heavy alcohol use is also a risk factor, but one beer a day is not considered heavy drinking. Antihistamines may contain phenylpropanolamine, which also increases the risk for stroke, but this client uses them seasonally and there is no information that they are abused or used heavily. The 65-year-old has only age as a risk factor.

The nurse on an inpatient rheumatology unit receives a hand-off report on a client with an acute exacerbation of systemic lupus erythematosus (SLE). Which reported laboratory value requires the nurse to assess the client further? a. Creatinine: 3.9 mg/dL b. Platelet count: 210,000/mm3 c. Red blood cell count: 5.2/mm3 d. White blood cell count: 4400/mm3

ANS: A Lupus nephritis is the leading cause of death in clients with SLE. The creatinine level is very high and the nurse needs to perform further assessments related to this finding. The other laboratory values are normal.

A nurse contacts the health care provider after reviewing a client's laboratory results and noting a blood urea nitrogen (BUN) of 35 mg/dL and a creatinine of 1.0 mg/dL. For which action should the nurse recommend a prescription? a. Intravenous fluids b. Hemodialysis c. Fluid restriction d. Urine culture and sensitivity

ANS: A Normal BUN is 10 to 20 mg/dL. Normal creatinine is 0.6 to 1.2 mg/dL (males) or 0.5 to 1.1 mg/dL (females). Creatinine is more specific for kidney function than BUN, because BUN can be affected by several factors (dehydration, high-protein diet, and catabolism). This client's creatinine is normal, which suggests a non-renal cause for the elevated BUN. A common cause of increased BUN is dehydration, so the nurse should anticipate giving the client more fluids, not placing the client on fluid restrictions. Hemodialysis is not an appropriate treatment for dehydration. The lab results do not indicate an infection; therefore, a urine culture and sensitivity is not appropriate.

A nurse assesses a client who is recovering from a myocardial infarction. The client's pulmonary artery pressure reading is 25/12 mm Hg. Which action should the nurse take first? a. Compare the results with previous pulmonary artery pressure readings. b. Increase the intravenous fluid rate because these readings are low. c. Immediately notify the health care provider of the elevated pressures. d. Document the finding in the client's chart as the only action.

ANS: A Normal pulmonary artery pressures range from 15 to 26 mm Hg for systolic and from 5 to 15 mm Hg for diastolic. Although this client's readings are within normal limits, the nurse needs to assess any trends that may indicate a need for medical treatment to prevent complications. There is no need to increase intravenous fluids or notify the provider.

A client in the orthopedic clinic has a self-reported history of osteoarthritis. The client reports a low-grade fever that started when the weather changed and several joints started "acting up," especially both hips and knees. What action by the nurse is best? a. Assess the client for the presence of subcutaneous nodules or Baker's cysts. b. Inspect the client's feet and hands for podagra and tophi on fingers and toes. c. Prepare to teach the client about an acetaminophen (Tylenol) regimen. d. Reassure the client that the problems will fade as the weather changes again.

ANS: A Osteoarthritis is not a systemic disease, nor does it present bilaterally. These are manifestations of rheumatoid arthritis. The nurse should assess for other manifestations of this disorder, including subcutaneous nodules and Baker's cysts. Podagra and tophi are seen in gout. Acetaminophen is not used for rheumatoid arthritis. Telling the client that the symptoms will fade with weather changes is not accurate.

15. A nurse teaches a client who experiences occasional premature atrial contractions (PACs) accompanied by palpitations that resolve spontaneously without treatment. Which statement should the nurse include in this clients teaching? a. Minimize or abstain from caffeine. b. Lie on your side until the attack subsides. c. Use your oxygen when you experience PACs. d. Take amiodarone (Cordarone) daily to prevent PACs.

ANS: A PACs usually have no hemodynamic consequences. For a client experiencing infrequent PACs, the nurse should explore possible lifestyle causes, such as excessive caffeine intake and stress. Lying on the side will not prevent or resolve PACs. Oxygen is not necessary. Although medications may be needed to control symptomatic dysrhythmias, for infrequent PACs, the client first should try lifestyle changes to control them.

A 70-kg adult with chronic renal failure is on a 40-g protein diet. The client has a reduced glomerular filtration rate and is not undergoing dialysis. Which result would give the nurse the most concern? a. Albumin level of 2.5 g/dL b. Phosphorus level of 5 mg/dL c. Sodium level of 135 mmol/L d. Potassium level of 5.5 mmol/L

ANS: A Protein restriction is necessary with chronic renal failure due to the buildup of waste products from protein breakdown. The nurse would be concerned with the low albumin level since this indicates that the protein in the diet is not enough for the client's metabolic needs. The electrolyte values are not related to the protein-restricted diet.

The hospital administration arranges for critical incident stress debriefing for the staff after a mass casualty incident. Which statement by the debriefing team leader is most appropriate for this situation? a. You are free to express your feelings; whatever is said here stays here. b. Lets evaluate what went wrong and develop policies for future incidents. c. This session is only for nursing and medical staff, not for ancillary personnel. d. Lets pass around the written policy compliance form for everyone.

ANS: A Strict confidentiality during stress debriefing is essential so that staff members can feel comfortable sharing their feelings, which should be accepted unconditionally. Brainstorming improvements and discussing policies would occur during an administrative review. Any employee present during a mass casualty situation is eligible for critical incident stress management services.

5. A nurse assesses an older adult client who was admitted 2 days ago with a fractured hip. The nurse notes that the client is confused and restless. The clients vital signs are heart rate 98 beats/min, respiratory rate 32 breaths/min, blood pressure 132/78 mm Hg, and SpO2 88%. Which action should the nurse take first? a. Administer oxygen via nasal cannula. b. Re-position to a high-Fowlers position. c. Increase the intravenous flow rate. d. Assess response to pain medications.

ANS: A The client is at high risk for a fat embolism and has some of the clinical manifestations of altered mental status and dyspnea. Although this is a life-threatening emergency, the nurse should take the time to administer oxygen first and then notify the health care provider. Oxygen administration can reduce the risk for cerebral damage from hypoxia. The nurse would not restrain a client who is confused without further assessment and orders. Sitting the client in a high-Fowlers position will not decrease hypoxia related to a fat embolism. The IV rate is not related. Pain medication most likely would not cause the client to be restless.

A male client with chronic kidney disease (CKD) is refusing to take his medication and has missed two hemodialysis appointments. What is the best initial action for the nurse? a. Discuss what the treatment regimen means to him. b. Refer the client to a mental health nurse practitioner. c. Reschedule the appointments to another date and time. d. Discuss the option of peritoneal dialysis.

ANS: A The initial action for the nurse is to assess anxiety, coping styles, and the client's acceptance of the required treatment for CKD. The client may be in denial of the diagnosis. While rescheduling hemodialysis appointments may help, and referral to a mental health practitioner and the possibility of peritoneal dialysis are all viable options, assessment of the client's acceptance of the treatment should come first.

What action by the perioperative nursing staff is most important to prevent surgical wound infection in a client having a total joint replacement? a. Administer preoperative antibiotic as ordered. b. Assess the client's white blood cell count. c. Instruct the client to shower the night before. d. Monitor the client's temperature postoperatively.

ANS: A To prevent surgical wound infection, antibiotics are given preoperatively within an hour of surgery. Simply taking a shower will not help prevent infection unless the client is told to use special antimicrobial soap. The other options are processes to monitor for infection, not prevent it.

A family in the emergency department is overwhelmed at the loss of several family members due to a shooting incident in the community. Which intervention should the nurse complete first? a. Provide a calm location for the family to cope and discuss needs. b. Call the hospital chaplain to stay with the family and pray for the deceased. c. Do not allow visiting of the victims until the bodies are prepared. d. Provide privacy for law enforcement to interview the family.

ANS: A The nurse should first provide emotional support by encouraging relaxation, listening to the family's needs, and offering choices when appropriate and possible to give some personal control back to individuals. The family may or may not want the assistance of religious personnel; the nurse should assess for this before calling anyone. Visiting procedures should take into account the needs of the family. The family may want to see the victim immediately and do not want to wait until the body can be prepared. The nurse should assess the family's needs before assuming the body needs to be prepared first. The family may appreciate privacy, but this is not as important as assessing the family's needs.

While assessing a client on a cardiac unit, a nurse identifies the presence of an S3 gallop. Which action should the nurse take next? a. Assess for symptoms of left-sided heart failure. b. Document this as a normal finding. c. Call the health care provider immediately. d. Transfer the client to the intensive care unit.

ANS: A The presence of an S3 gallop is an early diastolic filling sound indicative of increasing left ventricular pressure and left ventricular failure. The other actions are not warranted.

A client has a serum potassium level of 6.5 mmol/L, a serum creatinine level of 2 mg/dL, and a urine output of 350 mL/day. What is the best action by the nurse? a. Place the client on a cardiac monitor immediately. b. Teach the client to limit high-potassium foods. c. Continue to monitor the client's intake and output. d. Ask to have the laboratory redraw the blood specimen.

ANS: A The priority action by the nurse should be to check the cardiac status with a monitor. High potassium levels can lead to dysrhythmias. The other choices are logical nursing interventions for acute kidney injury but not the best immediate action.

3. A male client comes into the emergency department with a serum creatinine of 2.2 mg/dL and a blood urea nitrogen (BUN) of 24 mL/dL. What question should the nurse ask first when taking this client's history? a. "Have you been taking any aspirin, ibuprofen, or naproxen recently?" b. "Do you have anyone in your family with renal failure?" c. "Have you had a diet that is low in protein recently?" d. "Have you had a diet that is low in protein recently?"

ANS: A There are some medications that are nephrotoxic, such as the nonsteroidal anti-inflammatory drugs ibuprofen, aspirin, and naproxen. This would be a good question to initially ask the client since both the serum creatinine and BUN are elevated, indicating some renal problems. A family history of renal failure and kidney transplantation would not be part of the questioning and could cause anxiety in the client. A diet high in protein could be a factor in an increased BUN.

11. A client has a traumatic brain injury. The nurse assesses the following: pulse change from 82 to 60 beats/min, pulse pressure increase from 26 to 40 mm Hg, and respiratory irregularities. What action by the nurse takes priority? a. Call the provider or Rapid Response Team. b. Increase the rate of the IV fluid administration. c. Notify respiratory therapy for a breathing treatment. d. Prepare to give IV pain medication.

ANS: A These manifestations indicate Cushings syndrome, a potentially life-threatening increase in intracranial pressure (ICP), which is an emergency. Immediate medical attention is necessary, so the nurse notifies the provider or the Rapid Response Team. Increasing fluids would increase the ICP. The client does not need a breathing treatment or pain medication.

A client with chronic kidney disease (CKD) is experiencing nausea, vomiting, visual changes, and anorexia. Which action by the nurse is best? a. Check the client's digoxin (Lanoxin) level. b. Administer an anti-nausea medication. c. Ask if the client is able to eat crackers. d. Get a referral to a gastrointestinal provider.

ANS: A These signs and symptoms are indications of digoxin (Lanoxin) toxicity. The nurse should check the level of this medication. Administering antiemetics, asking if the client can eat, and obtaining a referral to a specialist all address the client's symptoms but do not lead to the cause of the symptoms.

A marathon runner comes into the clinic and states "I have not urinated very much in the last few days." The nurse notes a heart rate of 110 beats/min and a blood pressure of 86/58 mm Hg. Which action by the nurse is the priority? a. Give the client a bottle of water immediately. b. Start an intravenous line for fluids. c. Teach the client to drink 2 to 3 liters of water daily. d. Perform an electrocardiogram.

ANS: A This athlete is mildly dehydrated as evidenced by the higher heart rate and lower blood pressure. The nurse can start hydrating the client with a bottle of water first, followed by teaching the client to drink 2 to 3 liters of water each day. An intravenous line may be ordered later, after the client's degree of dehydration is assessed. An electrocardiogram is not necessary at this time.

17. A nurse supervises an unlicensed assistive personnel (UAP) applying electrocardiographic monitoring. Which statement should the nurse provide to the UAP related to this procedure? a. Clean the skin and clip hairs if needed. b. Add gel to the electrodes prior to applying them. c. Place the electrodes on the posterior chest. d. Turn off oxygen prior to monitoring the client.

ANS: A To ensure the best signal transmission, the skin should be clean and hairs clipped. Electrodes should be placed on the anterior chest, and no additional gel is needed. Oxygen has no impact on electrocardiographic

A hospital responds to a local mass casualty event. Which action should the nurse supervisor take to prevent staff post-traumatic stress disorder during a mass casualty event? a. Provide water and healthy snacks for energy throughout the event. b. Schedule 16-hour shifts to allow for greater rest between shifts. c. Encourage counseling upon deactivation of the emergency response plan. d. Assign staff to different roles and units within the medical facility.

ANS: A To prevent staff post-traumatic stress disorder during a mass casualty event, the nurses should use available counseling, encourage and support co-workers, monitor each others stress level and performance, take breaks when needed, talk about feelings with staff and managers, and drink plenty of water and eat healthy snacks for energy. Nurses should also keep in touch with family, friends, and significant others, and not work for more than 12 hours per day. Encouraging counseling upon deactivation of the plan, or after the emergency response is over, does not prevent stress during the casualty event. Assigning staff to unfamiliar roles or units may increase situational stress and is not an approach to prevent post-traumatic stress disorder.

A nurse assesses a client who had a myocardial infarction and is hypotensive. Which additional assessment finding should the nurse expect? a. Heart rate of 120 beats/min b. Cool, clammy skin c. Oxygen saturation of 90% d. Respiratory rate of 8 breaths/min

ANS: A When a client experiences hypotension, baroreceptors in the aortic arch sense a pressure decrease in the vessels. The parasympathetic system responds by lessening the inhibitory effect on the sinoatrial node. This results in an increase in heart rate and respiratory rate. This tachycardia is an early response and is seen even when blood pressure is not critically low. An increased heart rate and respiratory rate will compensate for the low blood pressure and maintain oxygen saturations and perfusion. The client may not be able to compensate for long, and decreased oxygenation and cool, clammy skin will occur later.

4.A nurse provides diabetic education at a public health fair. Which disorders should the nurse include as complications of diabetes mellitus? (Select all that apply.) a. Stroke b. Kidney failure c. Blindness d. Respiratory failure e. Cirrhosis

ANS: A, B, C Complications of diabetes mellitus are caused by macrovascular and microvascular changes. Macrovascular complications include coronary artery disease, cerebrovascular disease, and peripheral vascular disease. Microvascular complications include nephropathy, retinopathy, and neuropathy. Respiratory failure and cirrhosis are not complications of diabetes mellitus.

A home health nurse is visiting a new client who uses oxygen in the home. For which factors does the nurse assess when determining if the client is using the oxygen safely? (Select all that apply.) a. The client does not allow smoking in the house. b. Electrical cords are in good working order. c. Flammable liquids are stored in the garage. d. Household light bulbs are the fluorescent type. e. The client does not have pets inside the home.

ANS: A, B, C Oxygen is an accelerant, which means it enhances combustion, so precautions are needed whenever using it. The nurse should assess if the client allows smoking near the oxygen, whether electrical cords are in good shape or are frayed, and if flammable liquids are stored (and used) in the garage away from the oxygen. Light bulbs and pets are not related to oxygen safety

A nurse teaches a client who has chronic obstructive pulmonary disease. Which statements related to nutrition should the nurse include in this client's teaching? (Select all that apply.) a. "Avoid drinking fluids just before and during meals." b. "Rest before meals if you have dyspnea." c. "Have about six small meals a day." d. "Eat high-fiber foods to promote gastric emptying." e. "Increase carbohydrate intake for energy."

ANS: A, B, C Fluids can make a client feel bloated and should be avoided with meals. Resting before the meal will help a client with dyspnea. Six small meals a day also will help to decrease bloating. Fibrous foods can produce gas, which can cause abdominal bloating and can increase shortness of breath. The client should increase calorie and protein intake to prevent malnourishment. The client should not increase carbohydrate intake as this will increase carbon dioxide production and increase the client's risk of for acidosis.

A nurse assesses a client who is recovering from a heart transplant. Which assessment findings should alert the nurse to the possibility of heart transplant rejection? (Select all that apply.) a. Shortness of breath b. Abdominal bloating c. New-onset bradycardia d. Increased ejection fraction e. Hypertension

ANS: A, B, C Clinical manifestations of heart transplant rejection include shortness of breath, fatigue, fluid gain, abdominal bloating, new-onset bradycardia, hypotension, atrial fibrillation or flutter, decreased activity tolerance, and decreased ejection fraction.

A nurse is caring for a client with a history of renal insufficiency who is scheduled for a cardiac catheterization. Which actions should the nurse take prior to the catheterization? (Select all that apply.) a. Assess for allergies to iodine. b. Administer intravenous fluids. c. Assess blood urea nitrogen (BUN) and creatinine results. d. Insert a Foley catheter. e. Administer a prophylactic antibiotic. f. Insert a central venous catheter.

ANS: A, B, C If the client has kidney disease (as indicated by BUN and creatinine results), fluids and Mucomyst may be given 12 to 24 hours before the procedure for renal protection. The client should be assessed for allergies to iodine, including shellfish; the contrast medium used during the catheterization contains iodine. A Foley catheter and central venous catheter are not required for the procedure and would only increase the client's risk for infection. Prophylactic antibiotics are not administered prior to a cardiac catheterization.

A nurse teaches a client about self-care after experiencing a urinary calculus treated by lithotripsy. Which statements should the nurse include in this client's discharge teaching? (Select all that apply.) a. "Finish the prescribed antibiotic even if you are feeling better." b. "Drink at least 3 liters of fluid each day." c. "The bruising on your back may take several weeks to resolve." d. "Report any blood present in your urine." e. "It is normal to experience pain and difficulty urinating."

ANS: A, B, C The client should be taught to finish the prescribed antibiotic to ensure that he or she does not get a urinary tract infection. The client should drink at least 3 liters of fluid daily to dilute potential stone-forming crystals, prevent dehydration, and promote urine flow. After lithotripsy, the client should expect bruising that may take several weeks to resolve. The client should also experience blood in the urine for several days. The client should report any pain, fever, chills, or difficulty with urination to the provider as these may signal the beginning of an infection or the formation of another stone.

3. A nurse cares for a client with a fracture injury. Twenty minutes after an opioid pain medication is administered, the client reports pain in the site of the fracture. Which actions should the nurse take? (Select all that apply.) a. Administer additional opioids as prescribed. b. Elevate the extremity on pillows. c. Apply ice to the fracture site. d. Place a heating pad at the site of the injury. e. Keep the extremity in a dependent position.

ANS: A, B, C The client with a new fracture likely has edema; elevating the extremity and applying ice probably will help in decreasing pain. Administration of an additional opioid within the dosage guidelines may be ordered. Heat will increase edema and may increase pain. Dependent positioning will also increase edema.

The nurse is caring for five clients on the medical-surgical unit. Which clients would the nurse consider to be at risk for post-renal acute kidney injury (AKI)? (Select all that apply.) a. Man with prostate cancer b. Woman with blood clots in the urinary tract c. Client with ureterolithiasis d. Firefighter with severe burns e. Young woman with lupus

ANS: A, B, C Urine flow obstruction, such as prostate cancer, blood clots in the urinary tract, and kidney stones (ureterolithiasis), causes post-renal AKI. Severe burns would be a pre-renal cause. Lupus would be an intrarenal cause for AKI.

1. A nurse assesses a client who is 6 hours post-surgery for a nasal fracture and has nasal packing in place. Which actions should the nurse take? (Select all that apply.) a. Observe for clear drainage. b. Assess for signs of bleeding. c. Watch the client for frequent swallowing. d. Ask the client to open his or her mouth. e. Administer a nasal steroid to decrease edema. f. Change the nasal packing.

ANS: A, B, C, D The nurse should observe for clear drainage because of the risk for cerebrospinal fluid leakage. The nurse should assess for signs of bleeding by asking the client to open his or her mouth and observing the back of the throat for bleeding. The nurse should also note whether the client is swallowing frequently because this could indicate postnasal bleeding. A nasal steroid would increase the risk for infection. It is too soon to change the packing, which should be changed by the surgeon the first time.

3.A nurse prepares to administer a blood transfusion to a client, and checks the blood label with a second registered nurse using the International Society of Blood Transfusion (ISBT) universal bar-coding system to ensure the right blood for the right client. Which components must be present on the blood label in bar code and in eye-readable format? (Select all that apply.) a. Unique facility identifier b. Lot number related to the donor c. Name of the client receiving blood d. ABO group and Rh type of the donor e. Blood type of the client receiving blood

ANS: A, B, D The ISBT universal bar-coding system includes four components: (1) the unique facility identifier, (2) the lot number relating to the donor, (3) the product code, and (4) the ABO group and Rh type of the donor.

4.A nurse assists with the insertion of a central vascular access device. Which actions should the nurse ensure are completed to prevent a catheter-related bloodstream infection? (Select all that apply.) a. Include a review for the need of the device each day in the clients plan of care. b. Remind the provider to perform hand hygiene prior to starting the procedure. c. Cleanse the preferred site with alcohol and let it dry completely before insertion. d. Ask everyone in the room to wear a surgical mask during the procedure. e. Plan to complete a sterile dressing change on the device every day.

ANS: A, B, D The central vascular access device bundle to prevent catheter-related bloodstream infections includes using a checklist during insertion, performing hand hygiene before inserting the catheter and anytime someone touches the catheter, using chlorhexidine to disinfect the skin at the site of insertion, using preferred sites, and reviewing the need for the catheter every day. The practitioner who inserts the device should wear sterile gloves, gown and mask, and anyone in the room should wear a mask. A sterile dressing change should be completed per organizational policy, usually every 7 days and as needed.

5.A nurse collaborates with the interdisciplinary team to develop a plan of care for a client who is newly diagnosed with diabetes mellitus. Which team members should the nurse include in this interdisciplinary team meeting? (Select all that apply.) a. Registered dietitian b. Clinical pharmacist c. Occupational therapist d. Health care provider e. Speech-language pathologist

ANS: A, B, D When planning care for a client newly diagnosed with diabetes mellitus, the nurse should collaborate with a registered dietitian, clinical pharmacist, and health care provider. The focus of treatment for a newly diagnosed client would be nutrition, medication therapy, and education. The nurse could also consult with a diabetic educator. There is no need for occupational therapy or speech therapy at this time.

A nurse plans care for a client who has chronic obstructive pulmonary disease and thick, tenacious secretions. Which interventions should the nurse include in this client's plan of care? (Select all that apply.) a. Ask the client to drink 2 liters of fluids daily. b. Add humidity to the prescribed oxygen. c. Suction the client every 2 to 3 hours. d. Use a vibrating positive expiratory pressure device. e. Encourage diaphragmatic breathing.

ANS: A, B, D Interventions to decrease thick tenacious secretions include maintaining adequate hydration and providing humidified oxygen. These actions will help to thin secretions, making them easier to remove by coughing. The use of a vibrating positive expiratory pressure device can also help clients remove thick secretions. Although suctioning may assist with the removal of secretions, frequent suctioning can cause airway trauma and does not support the client's ability to successfully remove secretions through normal coughing. Diaphragmatic breathing is not used to improve the removal of thick secretions.

A nurse delegates care for an older adult client to the unlicensed assistive personnel (UAP). Which statements should the nurse include when delegating this client's care? (Select all that apply.) a. "Plan to bathe the client in the evening when the client is most alert." b. "Encourage the client to use a cane when ambulating." c. "Assess the client for symptoms related to pain and discomfort." d. "Remind the client to look at foot placement when walking." e. "Schedule additional time for teaching about prescribed therapies."

ANS: A, B, D The nurse should tell the UAP to schedule activities when the client is normally awake, encourage the client to use a cane when ambulating, and remind the client to look where feet are placed when walking. The nurse should assess the client for symptoms of pain and should provide sufficient time for older adults to process information, including new teaching. These are not items the nurse can delegate. DIF: Applying/Application REF: 836 KEY: Older adult| delegation| unlicensed assistive personnel (UAP) MSC: Integrated Process: Communication and Documentation NOT: Client Needs Category: Safe and Effective Care Environment

3. A nurse is teaching a client with premature ectopic beats. Which education should the nurse include in this clients teaching? (Select all that apply.) a. Smoking cessation b. Stress reduction and management c. Avoiding vagal stimulation d. Adverse effects of medications e. Foods high in potassium

ANS: A, B, D A client who has premature beats or ectopic rhythms should be taught to stop smoking, manage stress, take medications as prescribed, and report adverse effects of medications. Clients with premature beats are not at risk for vasovagal attacks or potassium imbalances.

A nurse assesses clients on a cardiac unit. Which clients should the nurse identify as at greatest risk for the development of acute pericarditis? (Select all that apply.) a. A 36-year-old woman with systemic lupus erythematosus (SLE) b. A 42-year-old man recovering from coronary artery bypass graft surgery c. A 59-year-old woman recovering from a hysterectomy d. An 80-year-old man with a bacterial infection of the respiratory tract e. An 88-year-old woman with a stage III sacral ulcer

ANS: A, B, D Acute pericarditis is most commonly associated acute exacerbations of systemic connective tissue disease, including SLE; with Dressler's syndrome, or inflammation of the cardiac sac after cardiac surgery or a myocardial infarction; and with infective organisms, including bacterial, viral, and fungal infections. Abdominal and reproductive surgeries and pressure ulcers do not increase clients' risk for acute pericarditis.

A client has rheumatoid arthritis (RA) and the visiting nurse is conducting a home assessment. What options can the nurse suggest for the client to maintain independence in activities of daily living (ADLs)? (Select all that apply.) a. Grab bars to reach high items b. Long-handled bath scrub brush c. Soft rocker-recliner chair d. Toothbrush with built-up handle e. Wheelchair cushion for comfort

ANS: A, B, D Grab bars, long-handled bath brushes, and toothbrushes with built-up handles all provide modifications for daily activities, making it easier for the client with RA to complete ADLs independently. The rocker-recliner and wheelchair cushion are comfort measures but do not help increase independence.

A client is undergoing hemodialysis. The client's blood pressure at the beginning of the procedure was 136/88 mm Hg, and now it is 110/54 mm Hg. What actions should the nurse perform to maintain blood pressure? (Select all that apply.) a. Adjust the rate of extracorporeal blood flow. b. Place the client in the Trendelenburg position. c. Stop the hemodialysis treatment. d. Administer a 250-mL bolus of normal saline. e. Contact the health care provider for orders.

ANS: A, B, D Hypotension occurs often during hemodialysis treatments as a result of vasodilation from the warmed dialysate. Modest decreases in blood pressure, as is the case with this client, can be maintained with rate adjustment, Trendelenburg positioning, and a fluid bolus. If the blood pressure drops considerably after two boluses and cooling dialysate, the hemodialysis can be stopped and the health care provider contacted.

4. A nurse plans care for a client who is recovering from open reduction and internal fixation (ORIF) surgery for a right hip fracture. Which interventions should the nurse include in this clients plan of care? (Select all that apply.) a. Elevate heels off the bed with a pillow. b. Ambulate the client on the first postoperative day. c. Push the clients patient-controlled analgesia button. d. Re-position the client every 2 hours. e. Use pillows to encourage subluxation of the hip.

ANS: A, B, D Postoperative care for a client who has ORIF of the hip includes elevating the clients heels off the bed and repositioning every 2 hours to prevent pressure and skin breakdown. It also includes ambulating the client on the first postoperative day, and using pillows or an abduction pillow to prevent subluxation of the hip. The nurse should teach the client to use the patient-controlled analgesia pump, but the nurse should never push the button for the client.

A nurse cares for clients with urinary incontinence. Which types of incontinence are correctly paired with their clinical manifestation? (Select all that apply.) a. Stress incontinence - Urine loss with physical exertion b. Urge incontinence - Large amount of urine with each occurrence c. Functional incontinence - Urine loss results from abnormal detrusor contractions d. Overflow incontinence - Constant dribbling of urine e. Reflex incontinence - Leakage of urine without lower urinary tract disorder

ANS: A, B, D Stress incontinence is a loss of urine with physical exertion, coughing, sneezing, or exercising. Urge incontinence presents with an abrupt and strong urge to void and usually has a large amount of urine released with each occurrence. Overflow incontinence occurs with bladder distention and results in a constant dribbling of urine. Functional incontinence is the leakage of urine caused by factors other than a disorder of the lower urinary tract. Reflex incontinence results from abnormal detrusor contractions from a neurologic abnormality.

A nurse prepares to discharge a client who has heart failure. Based on the Heart Failure Core Measure Set, which actions should the nurse complete prior to discharging this client? (Select all that apply.) a. Teach the client about dietary restrictions. b. Ensure the client is prescribed an angiotensin-converting enzyme (ACE) inhibitor. c. Encourage the client to take a baby aspirin each day. d. Confirm that an echocardiogram has been completed. e. Consult a social worker for additional resources.

ANS: A, B, D The Heart Failure Core Measure Set includes discharge instructions on diet, activity, medications, weight monitoring and plan for worsening symptoms, evaluation of left ventricular systolic function (usually with an echocardiogram), and prescribing an ACE inhibitor or angiotensin receptor blocker. Aspirin is not part of the Heart Failure Core Measure Set and is usually prescribed for clients who experience a myocardial infarction. Although the nurse may consult the social worker or case manager for additional resources, this is not part of the Core Measures.

A home health care nurse is visiting a client discharged home after a hip replacement. The client is still on partial weight bearing and using a walker. What safety precautions can the nurse recommend to the client? (Select all that apply.) a. Buy and install an elevated toilet seat. b. Install grab bars in the shower and by the toilet. c. Step into the bathtub with the affected leg first. d. Remove all throw rugs throughout the house. e. Use a shower chair while taking a shower.

ANS: A, B, D, E Buying and installing an elevated toilet seat, installing grab bars, removing throw rugs, and using a shower chair will all promote safety for this client. The client is still on partial weight bearing, so he or she cannot step into the bathtub leading with the operative side. Stepping into a bathtub may also require the client to bend the hip more than the allowed 90 degrees.

8. A nurse is working with many stroke clients. Which clients would the nurse consider referring to a mental health provider on discharge? (Select all that apply.) a. Client who exhibits extreme emotional lability b. Client with an initial National Institutes of Health (NIH) Stroke Scale score of 38 c. Client with mild forgetfulness and a slight limp d. Client who has a past hospitalization for a suicide attempt e. Client who is unable to walk or eat 3 weeks post-stroke

ANS: A, B, D, E Clients most at risk for post-stroke depression are those with a previous history of depression, severe stroke (NIH Stroke Scale score of 38 is severe), and post-stroke physical or cognitive impairment. The client with mild forgetfulness and a slight limp would be a low priority for this referral.

A nurse is teaching a female client with rheumatoid arthritis (RA) about taking methotrexate (MTX) (Rheumatrex) for disease control. What information does the nurse include? (Select all that apply.) a. "Avoid acetaminophen in over-the-counter medications." b. "It may take several weeks to become effective on pain." c. "Pregnancy and breast-feeding are not affected by MTX." d. "Stay away from large crowds and people who are ill." e. "You may find that folic acid, a B vitamin, reduces side effects."

ANS: A, B, D, E MTX is a disease-modifying antirheumatic drug and is used as a first-line drug for RA. MTX can cause liver toxicity, so the client should be advised to avoid medications that contain acetaminophen. It may take 4 to 6 weeks for effectiveness. MTX can cause immunosuppression, so avoiding sick people and crowds is important. Folic acid helps reduce side effects for some people. Pregnancy and breast-feeding are contraindicated while on this drug.

A nurse teaches a client with a history of calcium phosphate urinary stones. Which statements should the nurse include in this client's dietary teaching? (Select all that apply.) a. "Limit your intake of food high in animal protein." b. "Read food labels to help minimize your sodium intake." c. "Avoid spinach, black tea, and rhubarb." d. "Drink white wine or beer instead of red wine." e. "Reduce your intake of milk and other dairy products."

ANS: A, B, E Clients with calcium phosphate urinary stones should be taught to limit the intake of foods high in animal protein, sodium, and calcium. Clients with calcium oxalate stones should avoid spinach, black tea, and rhubarb. Clients with uric acid stones should avoid red wine.

A nurse is assessing a client with left-sided heart failure. For which clinical manifestations should the nurse assess? (Select all that apply.) a. Pulmonary crackles b. Confusion, restlessness c. Pulmonary hypertension d. Dependent edema e. Cough that worsens at night

ANS: A, B, E Left-sided heart failure occurs with a decrease in contractility of the heart or an increase in afterload. Most of the signs will be noted in the respiratory system. Right-sided heart failure occurs with problems from the pulmonary vasculature onward including pulmonary hypertension. Signs will be noted before the right atrium or ventricle including dependent edema.

An emergency department nurse moves to a new city where heat-related illnesses are common. Which clients does the nurse anticipate being at higher risk for heat-related illnesses? (Select all that apply.) a. Homeless individuals b. Illicit drug users c. White people d. Hockey players e. Older adults

ANS: A, B, E Some of the most vulnerable, at-risk populations for heat-related illness include older adults; blacks (more than whites); people who work outside, such as construction and agricultural workers (more men than women); homeless people; illicit drug users (especially cocaine users); outdoor athletes (recreational and professional); and members of the military who are stationed in countries with hot climates (e.g., Iraq, Afghanistan)

A nurse teaches a female client who has stress incontinence. Which statements should the nurse include about pelvic muscle exercises? (Select all that apply.) a. "When you start and stop your urine stream, you are using your pelvic muscles." b. "Tighten your pelvic muscles for a slow count of 10 and then relax for a slow count of 10." c. "Pelvic muscle exercises should only be performed sitting upright with your feet on the floor." d. "After you have been doing these exercises for a couple days, your control of urine will improve." e. "Like any other muscle in your body, you can make your pelvic muscles stronger by contracting them."

ANS: A, B, E The client should be taught that the muscles used to start and stop urination are pelvic muscles, and that pelvic muscles can be strengthened by contracting and relaxing them. The client should tighten pelvic muscles for a slow count of 10 and then relax the muscles for a slow count of 10, and perform this exercise 15 times while in lying-down, sitting-up, and standing positions. The client should begin to notice improvement in control of urine after several weeks of exercising the pelvic muscles.

2. A nurse teaches a client with a new permanent pacemaker. Which instructions should the nurse include in this clients teaching? (Select all that apply.) a. Until your incision is healed, do not submerge your pacemaker. Only take showers. b. Report any pulse rates lower than your pacemaker settings. c. If you feel weak, apply pressure over your generator. d. Have your pacemaker turned off before having magnetic resonance imaging (MRI). e. Do not lift your left arm above the level of your shoulder for 8 weeks.

ANS: A, B, E The client should not submerge in water until the site has healed; after the incision is healed, the client may take showers or baths without concern for the pacemaker. The client should be instructed to report changes in heart rate or rhythm, such as rates lower than the pacemaker setting or greater than 100 beats/min. The client should be advised of restrictions on physical activity for 8 weeks to allow the pacemaker to settle in place. The client should never apply pressure over the generator and should avoid tight clothing. The client should never have MRI because, whether turned on or off, the pacemaker contains metal. The client should be advised to inform all health care providers that he or she has a pacemaker.

A nurse evaluates laboratory results for a client with heart failure. Which results should the nurse expect? (Select all that apply.) a. Hematocrit: 32.8% b. Serum sodium: 130 mEq/L c. Serum potassium: 4.0 mEq/L d. Serum creatinine: 1.0 mg/dL e. Proteinuria f. Microalbuminuria

ANS: A, B, E, F A hematocrit of 32.8% is low (should be 42.6%), indicating a dilutional ratio of red blood cells to fluid. A serum sodium of 130 mEq/L is low because of hemodilution. Microalbuminuria and proteinuria are present, indicating a decrease in renal filtration. These are early warning signs of decreased compliance of the heart. The potassium level is on the high side of normal and the serum creatinine level is normal.

2.A nurse assesses a client who has a peripherally inserted central catheter (PICC). For which complications should the nurse assess? (Select all that apply.) a. Phlebitis b. Pneumothorax c. Thrombophlebitis d. Excessive bleeding e. Extravasation

ANS: A, C Although the complication rate with PICCs is fairly low, the most common complications are phlebitis, thrombophlebitis, and catheter-related bloodstream infection. Pneumothorax, excessive bleeding, and extravasation are not common complications.

3.A nurse teaches a client with diabetes mellitus about foot care. Which statements should the nurse include in this clients teaching? (Select all that apply.) a. Do not walk around barefoot. b. Soak your feet in a tub each evening. c. Trim toenails straight across with a nail clipper. d. Treat any blisters or sores with Epsom salts. e. Wash your feet every other day.

ANS: A, C Clients who have diabetes mellitus are at high risk for wounds on the feet secondary to peripheral neuropathy and poor arterial circulation. The client should be instructed to not walk around barefoot or wear sandals with open toes. These actions place the client at higher risk for skin breakdown of the feet. The client should be instructed to trim toenails straight across with a nail clipper. Feet should be washed daily with lukewarm water and soap, but feet should not be soaked in the tub. The client should contact the provider immediately if blisters or sores appear and should not use home remedies to treat these wounds.

A nurse is providing pneumonia vaccinations in a community setting. Due to limited finances, the event organizers must limit giving the vaccination to priority groups. What clients would be considered a priority when administering the pneumonia vaccination? (Select all that apply.) a. 22-year-old client with asthma b. Client who had a cholecystectomy last year c. Client with well-controlled diabetes d. Healthy 72-year-old client e. Client who is taking medication for hypertension

ANS: A, C, D, E Clients over 65 years of age and any client (no matter what age) with a chronic health condition would be considered a priority for a pneumonia vaccination. Having a cholecystectomy a year ago does not qualify as a chronic health condition.

2. The nurse working in the emergency department assesses a client who has symptoms of stroke. For what modifiable risk factors should the nurse assess? (Select all that apply.) a. Alcohol intake b. Diabetes c. High-fat diet d. Obesity e. Smoking

ANS: A, C, D, E Alcohol intake, a high-fat diet, obesity, and smoking are all modifiable risk factors for stroke. Diabetes is not modifiable but is a risk factor that can be controlled with medical intervention.

A nurse is providing health education at a community center. Which instructions should the nurse include in teaching about prevention of lightning injuries during a storm? (Select all that apply.) a. Seek shelter inside a building or vehicle. b. Hide under a tall tree. c. Do not take a bath or shower. d. Turn off the television. e. Remove all body piercings. f. Put down golf clubs or gardening tools.

ANS: A, C, D, F When thunder is heard, shelter should be sought in a safe area such as a building or an enclosed vehicle. Electrical equipment such as TVs and stereos should be turned off. Stay away from plumbing, water, and metal objects. Do not stand under an isolated tall tree or a structure such as a flagpole. Body piercings will not increase a persons chances of being struck by lightning.

Emergency medical services (EMS) brings a large number of clients to the emergency department following a mass casualty incident. The nurse identifies the clients with which injuries with yellow tags? (Select all that apply.) a. Partial-thickness burns covering both legs b. Open fractures of both legs with absent pedal pulses c. Neck injury and numbness of both legs d. Small pieces of shrapnel embedded in both eyes e. Head injury and difficult to arouse f. Bruising and pain in the right lower abdomen

ANS: A, C, D, F Clients with burns, spine injuries, eye injuries, and stable abdominal injuries should be treated within 30 minutes to 2 hours, and therefore should be identified with yellow tags. The client with the open fractures and the client with the head injury would be classified as urgent with red tags.

2.A nurse assesses a client who is experiencing diabetic ketoacidosis (DKA). For which manifestations should the nurse monitor the client? (Select all that apply.) a. Deep and fast respirations b. Decreased urine output c. Tachycardia d. Dependent pulmonary crackles e. Orthostatic hypotension

ANS: A, C, E DKA leads to dehydration, which is manifested by tachycardia and orthostatic hypotension. Usually clients have Kussmaul respirations, which are fast and deep. Increased urinary output (polyuria) is severe. Because of diuresis and dehydration, peripheral edema and crackles do not occur.

. A client is hospitalized in the oliguric phase of acute kidney injury (AKI) and is receiving tube feedings. The nurse is teaching the client's spouse about the kidney-specific formulation for the enteral solution compared to standard formulas. What components should be discussed in the teaching plan? (Select all that apply.) a. Lower sodium b. Higher calcium c. Lower potassium d. Higher phosphorus e. Higher calories

ANS: A, C, E Many clients with AKI are too ill to meet caloric goals and require tube feedings with kidney-specific formulas that are lower in sodium, potassium, and phosphorus, and higher in calories than are standard formulas.

An older client returning to the postoperative nursing unit after a hip replacement is disoriented and restless. What actions does the nurse delegate to the unlicensed assistive personnel (UAP)? (Select all that apply.) a. Apply an abduction pillow to the client's legs. b. Assess the skin under the abduction pillow straps. c. Place pillows under the heels to keep them off the bed. d. Monitor cognition to determine when the client can get up. e. Take and record vital signs per unit/facility policy.

ANS: A, C, E The UAP can apply an abduction pillow, elevate the heels on a pillow, and take/record vital signs. Assessing skin is the nurse's responsibility, although if the UAP notices abnormalities, he or she should report them. Determining when the client is able to get out of bed is also a nursing responsibility.

A nurse teaches a client about self-catheterization in the home setting. Which statements should the nurse include in this client's teaching? (Select all that apply.) a. "Wash your hands before and after self-catheterization." b. "Use a large-lumen catheter for each catheterization." c. "Use lubricant on the tip of the catheter before insertion." d. "Self-catheterize at least twice a day or every 12 hours." e. "Use sterile gloves and sterile technique for the procedure." f. "Maintain a specific schedule for catheterization."

ANS: A, C, F The key points in self-catheterization include washing hands, using lubricants, and maintaining a regular schedule to avoid distention and retention of urine that leads to bacterial growth. A smaller rather than a larger lumen catheter is preferred. The client needs to catheterize more often than every 12 hours. Self-catheterization in the home is a clean procedure.

A nurse triages clients arriving at the hospital after a mass casualty. Which clients are correctly classified? (Select all that apply.) a. A 35-year-old female with severe chest pain: red tag b. A 42-year-old male with full-thickness body burns: green tag c. A 55-year-old female with a scalp laceration: black tag d. A 60-year-old male with an open fracture with distal pulses: yellow tag e. An 88-year-old male with shortness of breath and chest bruises: green tag

ANS: A, D Red-tagged clients need immediate care due to life-threatening injuries. A client with severe chest pain would receive a red tag. Yellow-tagged clients have major injuries that should be treated within 30 minutes to 2 hours. A client with an open fracture with distal pulses would receive a yellow tag. The client with full thickness body burns would receive a black tag. The client with a scalp laceration would receive a green tag, and the client with shortness of breath would receive a red tag.

4. A nurse has applied to work at a hospital that has National Stroke Center designation. The nurse realizes the hospital adheres to eight Core Measures for ischemic stroke care. What do these Core Measures include? (Select all that apply.) a. Discharging the client on a statin medication b. Providing the client with comprehensive therapies c. Meeting goals for nutrition within 1 week d. Providing and charting stroke education e. Preventing venous thromboembolism

ANS: A, D, E Core Measures established by The Joint Commission include discharging stroke clients on statins, providing and recording stroke education, and taking measures to prevent venous thromboembolism. The client must be assessed for therapies but may go elsewhere for them. Nutrition goals are not part of the Core Measures.

An emergency department nurse plans care for a client who is admitted with heat stroke. Which interventions should the nurse include in this clients plan of care? (Select all that apply.) a. Administer oxygen via mask or nasal cannula. b. Administer ibuprofen, an antipyretic medication. c. Apply cooling techniques until core body temperature is less than 101 F. d. Infuse 0.9% sodium chloride via a large-bore intravenous cannula. e. Obtain baseline serum electrolytes and cardiac enzymes.

ANS: A, D, E Heat stroke is a medical emergency. Oxygen therapy and intravenous fluids should be provided, and baseline laboratory tests should be performed as quickly as possible. The client should be cooled until core body temperature is reduced to 102 F. Antipyretics should not be administered.

30.A nurse develops a dietary plan for a client with diabetes mellitus and new-onset microalbuminuria. Which component of the clients diet should the nurse decrease? a. Carbohydrates b. Proteins c. Fats d. Total calories

ANS: B Restriction of dietary protein to 0.8 g/kg of body weight per day is recommended for clients with microalbuminuria to delay progression to renal failure. The clients diet does not need to be decreased in carbohydrates, fats, or total calories.

A nurse is teaching a wilderness survival class. Which statements should the nurse include about the prevention of hypothermia and frostbite? (Select all that apply.) a. Wear synthetic clothing instead of cotton to keep your skin dry. b. Drink plenty of fluids. Brandy can be used to keep your body warm. c. Remove your hat when exercising to prevent the loss of heat. d. Wear sunglasses to protect skin and eyes from harmful rays. e. Know your physical limits. Come in out of the cold when limits are reached.

ANS: A, D, E To prevent hypothermia and frostbite, the nurse should teach clients to wear synthetic clothing (which moves moisture away from the body and dries quickly), layer clothing, and wear a hat, facemask, sunscreen, and sunglasses. The client should also be taught to drink plenty of fluids, but to avoid alcohol when participating in winter activities. Clients should know their physical limits and come in out of the cold when these limits have been reached.

A nurse cares for a client with arthritis who reports frequent asthma attacks. Which action should the nurse take first? a. Review the client's pulmonary function test results. b. Ask about medications the client is currently taking. c. Assess how frequently the client uses a bronchodilator. d. Consult the provider and request arterial blood gases.

ANS: B Aspirin and other nonsteroidal anti-inflammatory drugs (NSAIDs) can trigger asthma in some people. This results from increased production of leukotriene when aspirin or NSAIDs suppress other inflammatory pathways and is a high priority given the client's history. Reviewing pulmonary function test results will not address the immediate problem of frequent asthma attacks. This is a good intervention for reviewing response to bronchodilators. Questioning the client about the use of bronchodilators will address interventions for the attacks but not their cause. Reviewing arterial blood gas results would not be of use in a client between attacks because many clients are asymptomatic when not having attacks

A nurse administers medications to a client who has asthma. Which medication classification is paired correctly with its physiologic response to the medication? a. Bronchodilator - Stabilizes the membranes of mast cells and prevents the release of inflammatory mediators b. Cholinergic antagonist - Causes bronchodilation by inhibiting the parasympathetic nervous system c. Corticosteroid - Relaxes bronchiolar smooth muscles by binding to and activating pulmonary beta2 receptors d. Cromone - Disrupts the production of pathways of inflammatory mediators

ANS: B Cholinergic antagonist drugs cause bronchodilation by inhibiting the parasympathetic nervous system. This allows the sympathetic nervous system to dominate and release norepinephrine that actives beta2 receptors. Bronchodilators relax bronchiolar smooth muscles by binding to and activating pulmonary beta2 receptors. Corticosteroids disrupt the production of pathways of inflammatory mediators. Cromones stabilize the membranes of mast cells and prevent the release of inflammatory mediators.

A pulmonary nurse cares for clients who have chronic obstructive pulmonary disease (COPD). Which client should the nurse assess first? a. A 46-year-old with a 30-pack-year history of smoking b. A 52-year-old in a tripod position using accessory muscles to breathe c. A 68-year-old who has dependent edema and clubbed fingers d. A 74-year-old with a chronic cough and thick, tenacious secretions

ANS: B The client who is in a tripod position and using accessory muscles is working to breathe. This client must be assessed first to establish how well the client is breathing and provide interventions to minimize respiratory failure. The other clients are not in acute distress.

After teaching a client how to perform diaphragmatic breathing, the nurse assesses the client's understanding. Which action demonstrates that the client correctly understands the teaching? a. The client lays on his or her side with his or her knees bent. b. The client places his or her hands on his or her abdomen. c. The client lays in a prone position with his or her legs straight. d. The client places his or her hands above his or her head.

ANS: B To perform diaphragmatic breathing correctly, the client should place his or her hands on his or her abdomen to create resistance. This type of breathing cannot be performed effectively while lying on the side or with hands over the head. This type of breathing would not be as effective lying prone.

1. A nurse is caring for a client who has just had a central venous access line inserted. Which action should the nurse take next? a. Begin the prescribed infusion via the new access. b. Ensure an x-ray is completed to confirm placement. c. Check medication calculations with a second RN. d. Make sure the solution is appropriate for a central line.

ANS: B A central venous access device, once placed, needs an x-ray confirmation of proper placement before it is used. The bedside nurse would be responsible for beginning the infusion once placement has been verified. Any IV solution can be given through a central line.

43.A nurse prepares to administer prescribed regular and NPH insulin. Place the nurses actions in the correct order to administer these medications. 1. Inspect bottles for expiration dates. 2. Gently roll the bottle of NPH between the hands. 3. Wash your hands. 4. Inject air into the regular insulin. 5. Withdraw the NPH insulin. 6. Withdraw the regular insulin. 7. Inject air into the NPH bottle. 8. Clean rubber stoppers with an alcohol swab. a. 1, 3, 8, 2, 4, 6, 7, 5 b. 3, 1, 2, 8, 7, 4, 6, 5 c. 8, 1, 3, 2, 4, 6, 7, 5 d. 2, 3, 1, 8, 7, 5, 4, 6

ANS: B After washing hands, it is important to inspect the bottles and then to roll the NPH to mix the insulin. Rubber stoppers should be cleaned with alcohol after rolling the NPH and before sticking a needle into either bottle. It is important to inject air into the NPH bottle before placing the needle in a regular insulin bottle to avoid mixing of regular and NPH insulin. The shorter-acting insulin is always drawn up first.

23.A nurse cares for a client who is diagnosed with acute rejection 2 months after receiving a simultaneous pancreas-kidney transplant. The client states, I was doing so well with my new organs, and the thought of having to go back to living on hemodialysis and taking insulin is so depressing. How should the nurse respond? a. Following the drug regimen more closely would have prevented this. b. One acute rejection episode does not mean that you will lose the new organs. c. Dialysis is a viable treatment option for you and may save your life. d. Since you are on the national registry, you can receive a second transplantation.

ANS: B An episode of acute rejection does not automatically mean that the client will lose the transplant. Pharmacologic manipulation of host immune responses at this time can limit damage to the organ and allow the graft to be maintained. The other statements either belittle the client or downplay his or her concerns. The client may not be a candidate for additional organ transplantation.

4.A nurse is caring for a client who is having a subclavian central venous catheter inserted. The client begins to report chest pain and difficulty breathing. After administering oxygen, which action should the nurse take next? a. Administer a sublingual nitroglycerin tablet. b. Prepare to assist with chest tube insertion. c. Place a sterile dressing over the IV site. d. Re-position the client into the Trendelenburg position.

ANS: B An insertion-related complication of central venous catheters is a pneumothorax. Signs and symptoms of a pneumothorax include chest pain and dyspnea. Treatment includes removing the catheter, administering oxygen, and placing a chest tube. Pain is caused by the pneumothorax, which must be taken care of with a chest tube insertion. Use of a sterile dressing and placement of the client in a Trendelenburg position are not indicated for the primary problem of a pneumothorax.

14.A nurse is caring for a client with a peripheral vascular access device who is experiencing pain, redness, and swelling at the site. After removing the device, which action should the nurse take to relieve pain? a. Administer topical lidocaine to the site. b. Place warm compresses on the site. c. Administer prescribed oral pain medication. d. Massage the site with scented oils.

ANS: B At the first sign of phlebitis, the catheter should be removed and warm compresses used to relieve pain. The other options are not appropriate for this type of pain.

36.After teaching a client with type 2 diabetes mellitus, the nurse assesses the clients understanding. Which statement made by the client indicates a need for additional teaching? a. I need to have an annual appointment even if my glucose levels are in good control. b. Since my diabetes is controlled with diet and exercise, I must be seen only if I am sick. c. I can still develop complications even though I do not have to take insulin at this time. d. If I have surgery or get very ill, I may have to receive insulin injections for a short time.

ANS: B Clients with diabetes need to be seen at least annually to monitor for long-term complications, including visual changes, microalbuminuria, and lipid analysis. The client may develop complications and may need insulin in the future.

5.A nurse is caring for a client who is receiving an epidural infusion for pain management. Which assessment finding requires immediate intervention from the nurse? a. Redness at the catheter insertion site b. Report of headache and stiff neck c. Temperature of 100.1 F (37.8 C) d. Pain rating of 8 on a scale of 0 to 10

ANS: B Complications of epidural therapy include infection, bleeding, leakage of cerebrospinal fluid, occlusion of the catheter lumen, and catheter migration. Headache, neck stiffness, and a temperature higher than 101 F are signs of meningitis and should be reported to the provider immediately. The other findings are important but do not require immediate intervention.

The nurse is teaching the main principles of hemodialysis to a client with chronic kidney disease. Which statement by the client indicates a need for further teaching by the nurse? a. "My sodium level changes by movement from the blood into the dialysate." b. "Dialysis works by movement of wastes from lower to higher concentration." c. "Extra fluid can be pulled from the blood by osmosis." d. "The dialysate is similar to blood but without any toxins."

ANS: B Dialysis works using the passive transfer of toxins by diffusion. Diffusion is the movement of molecules from an area of higher concentration to an area of lower concentration. The other statements show a correct understanding about hemodialysis.

28.A nurse reviews the medication list of a client with a 20-year history of diabetes mellitus. The client holds up the bottle of prescribed duloxetine (Cymbalta) and states, My cousin has depression and is taking this drug. Do you think Im depressed? How should the nurse respond? a. Many people with long-term diabetes become depressed after a while. b. Its for peripheral neuropathy. Do you have burning pain in your feet or hands? c. This antidepressant also has anti-inflammatory properties for diabetic pain. d. No. Many medications can be used for several different disorders.

ANS: B Damage along nerves causes peripheral neuropathy and leads to burning pain along the nerves. Many drugs, including duloxetine (Cymbalta), can be used to treat peripheral neuropathy. The nurse should assess the client for this condition and then should provide an explanation of why this drug is being used. This medication, although it is used for depression, is not being used for that reason in this case. Duloxetine does not have anti-inflammatory properties. Telling the client that many medications are used for different disorders does not provide the client with enough information to be useful.

A nurse cares for a client with diabetes mellitus who asks, Why do I need to administer more than one injection of insulin each day? How should the nurse respond? a. You need to start with multiple injections until you become more proficient at self-injection. b. A single dose of insulin each day would not match your blood insulin levels and your food intake patterns. c. A regimen of a single dose of insulin injected each day would require that you eat fewer carbohydrates. d. A single dose of insulin would be too large to be absorbed, predictably putting you at risk for insulin shock.

ANS: B Even when a single injection of insulin contains a combined dose of different-acting insulin types, the timing of the actions and the timing of food intake may not match well enough to prevent wide variations in blood glucose levels. One dose of insulin would not be appropriate even if the client decreased carbohydrate intake. Additional injections are not required to allow the client practice with injections, nor will one dose increase the clients risk of insulin shock.

35.A nurse cares for a client who has diabetes mellitus. The nurse administers 6 units of regular insulin and 10 units of NPH insulin at 0700. At which time should the nurse assess the client for potential problems related to the NPH insulin? a. 0800 b. 1600 c. 2000 d. 2300

ANS: B Neutral protamine Hagedorn (NPH) is an intermediate-acting insulin with an onset of 1.5 hours, peak of 4 to 12 hours, and duration of action of 22 hours. Checking the client at 0800 would be too soon. Checking the client at 2000 and 2300 would be too late. The nurse should check the client at 1600.

A client is wearing a Venturi mask to deliver oxygen and the dinner tray has arrived. What action by the nurse is best? a. Assess the clients oxygen saturation and, if normal, turn off the oxygen. b. Determine if the client can switch to a nasal cannula during the meal. c. Have the client lift the mask off the face when taking bites of food. d. Turn the oxygen off while the client eats the meal and then restart it.

ANS: B Oxygen is a drug that needs to be delivered constantly. The nurse should determine if the provider has approved switching to a nasal cannula during meals. If not, the nurse should consult with the provider about this issue. The oxygen should not be turned off. Lifting the mask to eat will alter the FiO2 delivered.

A nurse is caring for a client using oxygen while in the hospital. What assessment finding indicates that goals for a priority diagnosis are being met? a. 100% of meals being eaten by the client b. Intact skin behind the ears c. The client understanding the need for oxygen d. Unchanged weight for the past 3 days

ANS: B Oxygen tubing can cause pressure ulcers, so clients using oxygen have the nursing diagnosis of Risk for Impaired Skin Integrity. Intact skin behind the ears indicates that goals for this diagnosis are being met. Nutrition and weight are not related to using oxygen. Understanding the need for oxygen is important but would not take priority over a physical problem.

The charge nurse on a medical unit is preparing to admit several "clients" who have possible pandemic flu during a preparedness drill. What action by the nurse is best? a. Admit the "clients" on Contact Precautions. b. Cohort the "clients" in the same area of the unit. c. Do not allow pregnant caregivers to care for these "clients." d. Place the "clients" on enhanced Droplet Precautions.

ANS: B Preventing the spread of pandemic flu is equally important as caring for the clients who have it. Clients can be cohorted together in the same set of rooms on one part of the unit to use distancing to help prevent the spread of the disease. The other actions are not appropriate.

A nurse assesses a client with diabetes mellitus. Which clinical manifestation should alert the nurse to decreased kidney function in this client? a. Urine specific gravity of 1.033 b. Presence of protein in the urine c. Elevated capillary blood glucose level d. Presence of ketone bodies in the urine

ANS: B Renal dysfunction often occurs in the client with diabetes. Proteinuria is a result of renal dysfunction. Specific gravity is elevated with dehydration. Elevated capillary blood glucose levels and ketones in the urine are consistent with diabetes mellitus but are not specific to renal function.

44.A nurse reviews the chart and new prescriptions for a client with diabetic ketoacidosis: Vital Signs and Assessment Laboratory Results Medications Blood pressure: 90/62 mm Hg Pulse: 120 beats/min Respiratory rate: 28 breaths/min Urine output: 20 mL/hr via catheter Serum potassium: 2.6 mEq/L Potassium chloride 40 mEq IV bolus STAT Increase IV fluid to 100 mL/hr Which action should the nurse take? a. Administer the potassium and then consult with the provider about the fluid order. b. Increase the intravenous rate and then consult with the provider about the potassium prescription. c. Administer the potassium first before increasing the infusion flow rate. d. Increase the intravenous flow rate before administering the potassium.

ANS: B The client is acutely ill and is severely dehydrated and hypokalemic. The client requires more IV fluids and potassium. However, potassium should not be infused unless the urine output is at least 30 mL/hr. The nurse should first increase the IV rate and then consult with the provider about the potassium.

41.A nurse reviews laboratory results for a client with diabetes mellitus who is prescribed an intensified insulin regimen: Fasting blood glucose: 75 mg/dL Postprandial blood glucose: 200 mg/dL Hemoglobin A1c level: 5.5% How should the nurse interpret these laboratory findings? a. Increased risk for developing ketoacidosis b. Good control of blood glucose c. Increased risk for developing hyperglycemia d. Signs of insulin resistance

ANS: B The client is maintaining blood glucose levels within the defined ranges for goals in an intensified regimen. Because the clients glycemic control is good, he or she is not at higher risk for ketoacidosis or hyperglycemia and is not showing signs of insulin resistance.

A nurse cares for a client who is prescribed pioglitazone (Actos). After 6 months of therapy, the client reports that his urine has become darker since starting the medication. Which action should the nurse take? a. Assess for pain or burning with urination. b. Review the clients liver function study results. c. Instruct the client to increase water intake. d. Test a sample of urine for occult blood.

ANS: B Thiazolidinediones (including pioglitazone) can affect liver function; liver function should be assessed at the start of therapy and at regular intervals while the client continues to take these drugs. Dark urine is one indicator of liver impairment because bilirubin is increased in the blood and is excreted in the urine. The nurse should check the clients most recent liver function studies. The nurse does not need to assess for pain or burning with urination and does not need to check the urine for occult blood. The client does not need to be told to increase water intake.

33.A nurse teaches a client with diabetes mellitus about sick day management. Which statement should the nurse include in this clients teaching? a. When ill, avoid eating or drinking to reduce vomiting and diarrhea. b. Monitor your blood glucose levels at least every 4 hours while sick. c. If vomiting, do not use insulin or take your oral antidiabetic agent. d. Try to continue your prescribed exercise regimen even if you are sick.

ANS: B When ill, the client should monitor his or her blood glucose at least every 4 hours. The client should continue taking the medication regimen while ill. The client should continue to eat and drink as tolerated but should not exercise while sick

19.A nurse assesses a client who has diabetes mellitus. Which arterial blood gas values should the nurse identify as potential ketoacidosis in this client? a. pH 7.38, HCO3 22 mEq/L, PCO2 38 mm Hg, PO2 98 mm Hg b. pH 7.28, HCO3 18 mEq/L, PCO2 28 mm Hg, PO2 98 mm Hg c. pH 7.48, HCO3 28 mEq/L, PCO2 38 mm Hg, PO2 98 mm Hg d. pH 7.32, HCO3 22 mEq/L, PCO2 58 mm Hg, PO2 88 mm Hg

ANS: B When the lungs can no longer offset acidosis, the pH decreases to below normal. A client who has diabetic ketoacidosis would present with arterial blood gas values that show primary metabolic acidosis with decreased bicarbonate levels and a compensatory respiratory alkalosis with decreased carbon dioxide levels.

A nurse obtains a focused health history for a client who is scheduled for magnetic resonance angiography. Which priority question should the nurse ask before the test? a. "Have you had a recent blood transfusion?" b. "Do you have allergies to iodine or shellfish?" c. "Are you taking any cardiac medications?" d. "Do you currently use oral contraceptives?"

ANS: B Allergies to iodine and/or shellfish need to be explored because the client may have a similar reaction to the dye used in the procedure. In some cases, the client may need to be medicated with antihistamines or steroids before the test is given. A recent blood transfusion or current use of cardiac medications or oral contraceptives would not affect the angiography. DIF: Applying/Application REF: 845 KEY: Assessment/diagnostic examination| allergies MSC: Integrated Process: Nursing Process: Assessment NOT: Client Needs Category: Safe and Effective Care Environment: Safety and Infection Control

A nurse cares for a client who has elevated levels of antidiuretic hormone (ADH). Which disorder should the nurse identify as a trigger for the release of this hormone? a. Pneumonia b. Dehydration c. Renal failure d. Edema

ANS: B ADH increases tubular permeability to water, leading to absorption of more water into the capillaries. ADH is triggered by a rising extracellular fluid osmolarity, as occurs in dehydration. Pneumonia, renal failure, and edema would not trigger the release of ADH.

After administering newly prescribed captopril (Capoten) to a client with heart failure, the nurse implements interventions to decrease complications. Which priority intervention should the nurse implement for this client? a. Provide food to decrease nausea and aid in absorption. b. Instruct the client to ask for assistance when rising from bed. c. Collaborate with unlicensed assistive personnel to bathe the client. d. Monitor potassium levels and check for symptoms of hypokalemia.

ANS: B Administration of the first dose of angiotensin-converting enzyme (ACE) inhibitors is often associated with hypotension, usually termed first-dose effect. The nurse should instruct the client to seek assistance before arising from bed to prevent injury from postural hypotension. ACE inhibitors do not need to be taken with food. Collaboration with unlicensed assistive personnel to provide hygiene is not a priority. The client should be encouraged to complete activities of daily living as independently as possible. The nurse should monitor for hyperkalemia, not hypokalemia, especially if the client has renal insufficiency secondary to heart failure.

A nurse works in the rheumatology clinic and sees clients with rheumatoid arthritis (RA). Which client should the nurse see first? a. Client who reports jaw pain when eating b. Client with a red, hot, swollen right wrist c. Client who has a puffy-looking area behind the knee d. Client with a worse joint deformity since the last visit

ANS: B All of the options are possible manifestations of RA. However, the presence of one joint that is much redder, hotter, or more swollen that the other joints may indicate infection. The nurse needs to see this client first.

The clinic nurse assesses a client with diabetes during a checkup. The client also has osteoarthritis (OA). The nurse notes the client's blood glucose readings have been elevated. What question by the nurse is most appropriate? a. "Are you compliant with following the diabetic diet?" b. "Have you been taking glucosamine supplements?" c. "How much exercise do you really get each week?" d. "You're still taking your diabetic medication, right?"

ANS: B All of the topics are appropriate for a client whose blood glucose readings have been higher than usual. However, since this client also has OA, and glucosamine can increase blood glucose levels, the nurse should ask about its use. The other questions all have an element of nontherapeutic communication in them. "Compliant" is a word associated with negative images, and the client may deny being "noncompliant." Asking how much exercise the client "really" gets is accusatory. Asking if the client takes his or her medications "right?" is patronizing.

A nurse reviews the laboratory findings of a client with a urinary tract infection. The laboratory report notes a "shift to the left" in a client's white blood cell count. Which action should the nurse take? a. Request that the laboratory perform a differential analysis on the white blood cells. b. Notify the provider and start an intravenous line for parenteral antibiotics. c. Collaborate with the unlicensed assistive personnel (UAP) to strain the client's urine for renal calculi. d. Assess the client for a potential allergic reaction and anaphylactic shock.

ANS: B An increase in band cells creates a "shift to the left." A left shift most commonly occurs with urosepsis and is seen rarely with uncomplicated urinary tract infections. The nurse will be administering antibiotics, most likely via IV, so he or she should notify the provider and prepare to give the antibiotics. The shift to the left is part of a differential white blood cell count. The nurse would not need to strain urine for stones. Allergic reactions are associated with elevated eosinophil cells, not band cells.

A client is having a peritoneal dialysis treatment. The nurse notes an opaque color to the effluent. What is the priority action by the nurse? a. Warm the dialysate solution in a microwave before instillation. b. Take a sample of the effluent and send to the laboratory. c. Flush the tubing with normal saline to maintain patency of the catheter. d. Check the peritoneal catheter for kinking and curling.

ANS: B An opaque or cloudy effluent is the first sign of peritonitis. A sample of the effluent would need to be sent to the laboratory for culture and sensitivity in order to administer the correct antibiotic. Warming the dialysate in a microwave and flushing the tubing are not safe actions by the nurse. Checking the catheter for obstruction is a viable option but will not treat the peritonitis.

A nurse assesses a client who has mitral valve regurgitation. For which cardiac dysrhythmia should the nurse assess? a. Preventricular contractions b. Atrial fibrillation c. Symptomatic bradycardia d. Sinus tachycardia

ANS: B Atrial fibrillation is a clinical manifestation of mitral valve regurgitation and stenosis. Preventricular contractions and bradycardia are not associated with valvular problems. These are usually identified in clients with electrolyte imbalances, myocardial infarction, and sinus node problems. Sinus tachycardia is a manifestation of aortic regurgitation due to a decrease in cardiac output.

5. A nurse evaluates prescriptions for a client with chronic atrial fibrillation. Which medication should the nurse expect to find on this clients medication administration record to prevent a common complication of this condition? a. Sotalol (Betapace) b. Warfarin (Coumadin) c. Atropine (Sal-Tropine) d. Lidocaine (Xylocaine)

ANS: B Atrial fibrillation puts clients at risk for developing emboli. Clients at risk for emboli are treated with anticoagulants, such as heparin, enoxaparin, or warfarin. Sotalol, atropine, and lidocaine are not appropriate for this complication.

2. A nurse cares for a client who has a heart rate averaging 56 beats/min with no adverse symptoms. Which activity modification should the nurse suggest to avoid further slowing of the heart rate? a. Make certain that your bath water is warm. b. Avoid straining while having a bowel movement. c. Limit your intake of caffeinated drinks to one a day. d. Avoid strenuous exercise such as running.

ANS: B Bearing down strenuously during a bowel movement is one type of Valsalva maneuver, which stimulates the vagus nerve and results in slowing of the heart rate. Such a response is not desirable in a person who has bradycardia. The other instructions are not appropriate for this condition.

After a hospitals emergency department (ED) has efficiently triaged, treated, and transferred clients from a community disaster to appropriate units, the hospital incident command officer wants to stand down from the emergency plan. Which question should the nursing supervisor ask at this time? a. Are you sure no more victims are coming into the ED? b. Do all areas of the hospital have the supplies and personnel they need? c. Have all ED staff had the chance to eat and rest recently? d. Does the Chief Medical Officer agree this disaster is under control?

ANS: B Before standing down, the incident command officer ensures that the needs of the other hospital departments have been taken care of because they may still be stressed and may need continued support to keep functioning. Many more walking wounded victims may present to the ED; that number may not be predictable. Giving staff the chance to eat and rest is important, but all areas of the facility need that too. Although the Chief Medical Officer (CMO) may be involved in the incident, the CMO does not determine when the hospital can stand down.

A nurse teaches a client with diabetes mellitus and a body mass index of 42 who is at high risk for coronary artery disease. Which statement related to nutrition should the nurse include in this client's teaching? a. "The best way to lose weight is a high-protein, low-carbohydrate diet." b. "You should balance weight loss with consuming necessary nutrients." c. "A nutritionist will provide you with information about your new diet." d. "If you exercise more frequently, you won't need to change your diet."

ANS: B Clients at risk for cardiovascular diseases should follow the American Heart Association guidelines to combat obesity and improve cardiac health. The nurse should encourage the client to eat vegetables, fruits, unrefined whole-grain products, and fat-free dairy products while losing weight. High-protein food items are often high in fat and calories. Although the nutritionist can assist with client education, the nurse should include nutrition education and assist the client to make healthy decisions. Exercising and eating nutrient-rich foods are both important components in reducing cardiovascular risk.

After teaching a client who is being discharged home after mitral valve replacement surgery, the nurse assesses the client's understanding. Which client statement indicates a need for additional teaching? a. "I'll be able to carry heavy loads after 6 months of rest." b. "I will have my teeth cleaned by my dentist in 2 weeks." c. "I must avoid eating foods high in vitamin K, like spinach." d. "I must use an electric razor instead of a straight razor to shave."

ANS: B Clients who have defective or repaired valves are at high risk for endocarditis. The client who has had valve surgery should avoid dental procedures for 6 months because of the risk for endocarditis. When undergoing a mitral valve replacement surgery, the client needs to be placed on anticoagulant therapy to prevent vegetation forming on the new valve. Clients on anticoagulant therapy should be instructed on bleeding precautions, including using an electric razor. If the client is prescribed warfarin, the client should avoid foods high in vitamin K. Clients recovering from open heart valve replacements should not carry anything heavy for 6 months while the chest incision and muscle heal.

4. A nurse assesses a client with atrial fibrillation. Which manifestation should alert the nurse to the possibility of a serious complication from this condition? a. Sinus tachycardia b. Speech alterations c. Fatigue d. Dyspnea with activity

ANS: B Clients with atrial fibrillation are at risk for embolic stroke. Evidence of embolic events includes changes in mentation, speech, sensory function, and motor function. Clients with atrial fibrillation often have a rapid ventricular response as a result. Fatigue is a nonspecific complaint. Clients with atrial fibrillation often have dyspnea as a result of the decreased cardiac output caused by the rhythm disturbance.

8. An emergency department nurse assesses a client with kidney trauma and notes that the client's abdomen is tender and distended and blood is visible at the urinary meatus. Which prescription should the nurse consult the provider about before implementation? a. Assessing vital signs every 15 minutes b. Inserting an indwelling urinary catheter c. Administering intravenous fluids at 125 mL/hr d. Typing and crossmatching for blood products

ANS: B Clients with blood at the urinary meatus should not have a urinary catheter inserted via the urethra before additional diagnostic studies are done. The urethra could be torn. The nurse should question the provider about the need for a catheter; if one is needed, the provider can insert a suprapubic catheter. The nurse should monitor the client's vital signs closely, send blood for type and crossmatch in case the client needs blood products, and administer intravenous fluids.

22. After assessing a client who is receiving an amiodarone intravenous infusion for unstable ventricular tachycardia, the nurse documents the findings and compares these with the previous assessment findings: Vital Signs Nursing Assessment Time: 0800 Temperature: 98 F Heart rate: 68 beats/min Blood pressure: 135/60 mm Hg Respiratory rate: 14 breaths/min Oxygen saturation: 96% Oxygen therapy: 2 L nasal cannula Time: 1000 Temperature: 98.2 F Heart rate: 50 beats/min Blood pressure: 132/57 mm Hg Respiratory rate: 16 breaths/min Oxygen saturation: 95% Oxygen therapy: 2 L nasal cannula Time: 0800 Client alert and oriented. Cardiac rhythm: normal sinus rhythm. Skin: warm, dry, and appropriate for race. Respirations equal and unlabored. Client denies shortness of breath and chest pain. Time: 1000 Client alert and oriented. Cardiac rhythm: sinus bradycardia. Skin: warm, dry, and appropriate for race. Respirations equal and unlabored. Client denies shortness of breath and chest pain. Client voids 420 mL of clear yellow urine. Based on the assessments, which action should the nurse take? a. Stop the infusion and flush the IV. b. Slow the amiodarone infusion rate. c. Administer IV normal saline. d. Ask the client to cough and deep breathe.

ANS: B IV administration of amiodarone may cause bradycardia and atrioventricular (AV) block. The correct action for the nurse to take at this time is to slow the infusion, because the client is asymptomatic and no evidence reveals AV block that might require pacing. Abruptly ceasing the medication could allow fatal dysrhythmias to occur. The administration of IV fluids and encouragement of coughing and deep breathing exercises are not indicated, and will not increase the clients heart rate.

A nurse reviews a female client's laboratory results. Which results from the client's urinalysis should the nurse recognize as abnormal? a. pH 5.6 b. Ketone bodies present c. Specific gravity of 1.020 d. Clear and yellow color

ANS: B Ketone bodies are by-products of incomplete metabolism of fatty acids. Normally no ketones are present in urine. Ketone bodies are produced when fat sources are used instead of glucose to provide cellular energy. A pH between 4.6 and 8, specific gravity between 1.005 and 1.030, and clear yellow urine are normal findings for a female client's urinalysis.

The nurse is caring for four clients with chronic kidney disease. Which client should the nurse assess first upon initial rounding? a. Woman with a blood pressure of 158/90 mm Hg b. Client with Kussmaul respirations c. Man with skin itching from head to toe d. Client with halitosis and stomatitis

ANS: B Kussmaul respirations indicate a worsening of chronic kidney disease (CKD). The client is increasing the rate and depth of breathing to excrete carbon dioxide through the lungs. Hypertension is common in most clients with CKD, and skin itching increases with calcium-phosphate imbalances, another common finding in CKD. Uremia from CKD causes ammonia to be formed, resulting in the common findings of halitosis and stomatitis

A nurse cares for a postmenopausal client who has had two episodes of bacterial urethritis in the last 6 months. The client asks, "I never have urinary tract infections. Why is this happening now?" How should the nurse respond? a. "Your immune system becomes less effective as you age." b. "Low estrogen levels can make the tissue more susceptible to infection." c. "You should be more careful with your personal hygiene in this area." d. "It is likely that you have an untreated sexually transmitted disease."

ANS: B Low estrogen levels decrease moisture and secretions in the perineal area and cause other tissue changes, predisposing it to the development of infection. Urethritis is most common in postmenopausal women for this reason. Although immune function does decrease with aging and sexually transmitted diseases are a known cause of urethritis, the most likely reason in this client is low estrogen levels. Personal hygiene usually does not contribute to this disease process.

A nurse reviews the health history of a client with an oversecretion of renin. Which disorder should the nurse correlate with this assessment finding? a. Alzheimer's disease b. Hypertension c. Diabetes mellitus d. Viral hepatitis

ANS: B Renin is secreted when special cells in the distal convoluted tubule, called the macula densa, sense changes in blood volume and pressure. When the macula densa cells sense that blood volume, blood pressure, or blood sodium levels are low, renin is secreted. Renin then converts angiotensinogen into angiotensin I. This leads to a series of reactions that cause secretion of the hormone aldosterone. This hormone increases kidney reabsorption of sodium and water, increasing blood pressure, blood volume, and blood sodium levels. Inappropriate or excessive renin secretion is a major cause of persistent hypertension. Renin has no impact on Alzheimer's disease, diabetes mellitus, or viral hepatitis.

A client with systemic lupus erythematosus (SLE) was recently discharged from the hospital after an acute exacerbation. The client is in the clinic for a follow-up visit and is distraught about the possibility of another hospitalization disrupting the family. What action by the nurse is best? a. Explain to the client that SLE is an unpredictable disease. b. Help the client create backup plans to minimize disruption. c. Offer to talk to the family and educate them about SLE. d. Tell the client to remain compliant with treatment plans.

ANS: B SLE is an unpredictable disease and acute exacerbations can occur without warning, creating chaos in the family. Helping the client make backup plans for this event not only will decrease the disruption but will give the client a sense of having more control. Explaining facts about the disease is helpful as well but does not engage the client in problem solving. The family may need education, but again this does not help the client to problem-solve. Remaining compliant may help decrease exacerbations, but is not as powerful an intervention as helping the client plan for such events.

A nurse assesses a client admitted to the cardiac unit. Which statement by the client alerts the nurse to the possibility of right-sided heart failure? a. "I sleep with four pillows at night." b. "My shoes fit really tight lately." c. "I wake up coughing every night." d. "I have trouble catching my breath."

ANS: B Signs of systemic congestion occur with right-sided heart failure. Fluid is retained, pressure builds in the venous system, and peripheral edema develops. Left-sided heart failure symptoms include respiratory symptoms. Orthopnea, coughing, and difficulty breathing all could be results of left-sided heart failure.

An emergency department nurse assesses a client with a history of urinary incontinence who presents with extreme dry mouth, constipation, and an inability to void. Which question should the nurse ask first? a. "Are you drinking plenty of water?" b. "What medications are you taking?" c. "Have you tried laxatives or enemas?" d. "Has this type of thing ever happened before?"

ANS: B Some types of incontinence are treated with anticholinergic medications such as propantheline (Pro-Banthine). Anticholinergic side effects include dry mouth, constipation, and urinary retention. The nurse needs to assess the client's medication list to determine whether the client is taking an anticholinergic medication. If he or she is taking anticholinergics, the nurse should further assess the client's manifestations to determine if they are related to a simple side effect or an overdose. The other questions are not as helpful to understanding the current situation.

7. A student nurse is preparing morning medications for a client who had a stroke. The student plans to hold the docusate sodium (Colace) because the client had a large stool earlier. What action by the supervising nurse is best? a. Have the student ask the client if it is desired or not. b. Inform the student that the docusate should be given. c. Tell the student to document the rationale. d. Tell the student to give it unless the client refuses.

ANS: B Stool softeners should be given to clients with neurologic disorders in order to prevent an elevation in intracranial pressure that accompanies the Valsalva maneuver when constipated. The supervising nurse should instruct the student to administer the docusate. The other options are not appropriate. The medication could be held for diarrhea.

A nurse cares for a client recovering from prosthetic valve replacement surgery. The client asks, "Why will I need to take anticoagulants for the rest of my life?" How should the nurse respond? a. "The prosthetic valve places you at greater risk for a heart attack." b. "Blood clots form more easily in artificial replacement valves." c. "The vein taken from your leg reduces circulation in the leg." d. "The surgery left a lot of small clots in your heart and lungs."

ANS: B Synthetic valve prostheses and scar tissue provide surfaces on which platelets can aggregate easily and initiate the formation of blood clots. The other responses are inaccurate.

A nurse cares for a client who has an 80% blockage of the right coronary artery (RCA) and is scheduled for bypass surgery. Which intervention should the nurse be prepared to implement while this client waits for surgery? a. Administration of IV furosemide (Lasix) b. Initiation of an external pacemaker c. Assistance with endotracheal intubation d. Placement of central venous access

ANS: B The RCA supplies the right atrium, the right ventricle, the inferior portion of the left ventricle, and the atrioventricular (AV) node. It also supplies the sinoatrial node in 50% of people. If the client totally occludes the RCA, the AV node would not function and the client would go into heart block, so emergency pacing should be available for the client. Furosemide, intubation, and central venous access will not address the primary complication of RCA occlusion, which is AV node malfunction.

10. After teaching a client who has an implantable cardioverter-defibrillator (ICD), a nurse assesses the clients understanding. Which statement by the client indicates a correct understanding of the teaching? a. I should wear a snug-fitting shirt over the ICD. b. I will avoid sources of strong electromagnetic fields. c. I should participate in a strenuous exercise program. d. Now I can discontinue my antidysrhythmic medication.

ANS: B The client being discharged with an ICD is instructed to avoid strong sources of electromagnetic fields. Clients should avoid tight clothing, which could cause irritation over the ICD generator. The client should be encouraged to exercise but should not engage in strenuous activities that cause the heart rate to meet or exceed the ICD cutoff point because the ICD can discharge inappropriately. The client should continue all prescribed medications.

9. After teaching a client with hypertension secondary to renal disease, the nurse assesses the client's understanding. Which statement made by the client indicates a need for additional teaching? a. "I can prevent more damage to my kidneys by managing my blood pressure." b. "If I have increased urination at night, I need to drink less fluid during the day." c. "I need to see the registered dietitian to discuss limiting my protein intake." d. "It is important that I take my antihypertensive medications as directed."

ANS: B The client should not restrict fluids during the day due to increased urination at night. Clients with renal disease may be prescribed fluid restrictions. These clients should be assessed thoroughly for potential dehydration. Increased nocturnal voiding can be decreased by consuming fluids earlier in the day. Blood pressure control is needed to slow the progression of renal dysfunction. When dietary protein is restricted, refer the client to the registered dietitian as needed.

A nurse assesses a client with pericarditis. Which assessment finding should the nurse expect to find? a. Heart rate that speeds up and slows down b. Friction rub at the left lower sternal border c. Presence of a regular gallop rhythm d. Coarse crackles in bilateral lung bases

ANS: B The client with pericarditis may present with a pericardial friction rub at the left lower sternal border. This sound is the result of friction from inflamed pericardial layers when they rub together. The other assessments are not related.

A nurse cares for a client who is prescribed magnetic resonance imaging (MRI) of the heart. The client's health history includes a previous myocardial infarction and pacemaker implantation. Which action should the nurse take? a. Schedule an electrocardiogram just before the MRI. b. Notify the health care provider before scheduling the MRI. c. Call the physician and request a laboratory draw for cardiac enzymes. d. Instruct the client to increase fluid intake the day before the MRI.

ANS: B The magnetic fields of the MRI can deactivate the pacemaker. The nurse should call the health care provider and report that the client has a pacemaker so the provider can order other diagnostic tests. The client does not need an electrocardiogram, cardiac enzymes, or increased fluids.

22. A nurse cares for an older adult client with multiple fractures. Which action should the nurse take to manage this clients pain? a. Meperidine (Demerol) injections every 4 hours around the clock b. Patient-controlled analgesia (PCA) pump with morphine c. Ibuprofen (Motrin) 600 mg orally every 4 hours PRN for pain d. Morphine 4 mg intravenous push every 2 hours PRN for pain

ANS: B The older adult client should never be treated with meperidine because toxic metabolites can cause seizures. The client should be managed with a PCA pump to control pain best. Motrin most likely would not provide complete pain relief with multiple fractures. IV morphine PRN would not control pain as well as a pump that the client can control.

A client is scheduled to have a hip replacement. Preoperatively, the client is found to be mildly anemic and the surgeon states the client may need a blood transfusion during or after the surgery. What action by the preoperative nurse is most important? a. Administer preoperative medications as prescribed. b. Ensure that a consent for transfusion is on the chart. c. Explain to the client how anemia affects healing. d. Teach the client about foods high in protein and iron.

ANS: B The preoperative nurse should ensure that all valid consents are on the chart, including one for blood transfusions if this may be needed. Administering preoperative medications is important for all preoperative clients and is not specific to this client. Teaching in the preoperative area should focus on immediate concerns.

A nurse auscultated heart tones on an older adult client. Which action should the nurse take based on heart tones heard? (Click the media button to hear the audio clip.) a. Administer a diuretic. b. Document the finding. c. Decrease the IV flow rate. d. Evaluate the client's medications.

ANS: B The sound heard is an atrial gallop S4. An atrial gallop may be heard in older clients because of a stiffened ventricle. The nurse should document the finding, but no other intervention is needed at this time.

A nurse teaches a client recovering from a heart transplant who is prescribed cyclosporine (Sandimmune). Which statement should the nurse include in this client's discharge teaching? a. "Use a soft-bristled toothbrush and avoid flossing." b. "Avoid large crowds and people who are sick." c. "Change positions slowly to avoid hypotension." d. "Check your heart rate before taking the medication."

ANS: B These agents cause immune suppression, leaving the client more vulnerable to infection. The medication does not place the client at risk for bleeding, orthostatic hypotension, or a change in heart rate.

A nurse is giving discharge instructions to a client recently diagnosed with chronic kidney disease (CKD). Which statements made by the client indicate a correct understanding of the teaching? (Select all that apply.) a. "I can continue to take antacids to relieve heartburn." b. "I need to ask for an antibiotic when scheduling a dental appointment." c. "I'll need to check my blood sugar often to prevent hypoglycemia." d. "The dose of my pain medication may have to be adjusted." e. "I should watch for bleeding when taking my anticoagulants."

ANS: B, C, D, E In discharge teaching, the nurse must emphasize that the client needs to have an antibiotic prophylactically before dental procedures to prevent infection. There may be a need for dose reduction in medications if the kidney is not excreting them properly (antacids with magnesium, antibiotics, antidiabetic drugs, insulin, opioids, and anticoagulants).

A nurse assesses a client with chronic obstructive pulmonary disease. Which questions should the nurse ask to determine the client's activity tolerance? (Select all that apply.) a. "What color is your sputum?" b. "Do you have any difficulty sleeping?" c. "How long does it take to perform your morning routine?" d. "Do you walk upstairs every day?" e. "Have you lost any weight lately?"

ANS: B, C, E Difficulty sleeping could indicate worsening breathlessness, as could taking longer to perform activities of daily living. Weight loss could mean increased dyspnea as the client becomes too fatigued to eat. The color of the client's sputum would not assist in determining activity tolerance. Asking whether the client walks upstairs every day is not as pertinent as determining if the client becomes short of breath on walking upstairs, or if the client goes upstairs less often than previously.

A nurse assesses a client who is diagnosed with infective endocarditis. Which assessment findings should the nurse expect? (Select all that apply.) a. Weight gain b. Night sweats c. Cardiac murmur d. Abdominal bloating e. Osler's nodes

ANS: B, C, E Clinical manifestations of infective endocarditis include fever with chills, night sweats, malaise and fatigue, anorexia and weight loss, cardiac murmur, and Osler's nodes on palms of the hands and soles of the feet. Abdominal bloating is a manifestation of heart transplantation rejection.

2. An emergency nurse assesses a client who is admitted with a pelvic fracture. Which assessments should the nurse monitor to prevent a complication of this injury? (Select all that apply.) a. Temperature b. Urinary output c. Blood pressure d. Pupil reaction e. Skin color

ANS: B, C, E With a pelvic fracture, internal organ damage may result in bleeding and hypovolemic shock. The nurse monitors the clients heart rate, blood pressure, urine output, skin color, and level of consciousness frequently to determine whether shock is manifesting. It is important to monitor the urine for blood to assess whether the urinary system has been damaged with the pelvic fracture. Changes in temperature and pupil reactions are not directly associated with hypovolemic shock. Temperature changes are usually associated with hypo- or hyperthermia or infectious processes. Pupillary changes occur with brain injuries, bleeds, or neurovascular accidents.

An emergency room nurse assesses a female client. Which assessment findings should alert the nurse to request a prescription for an electrocardiogram? (Select all that apply.) a. Hypertension b. Fatigue despite adequate rest c. Indigestion d. Abdominal pain e. Shortness of breath

ANS: B, C, E Women may not have chest pain with myocardial infarction, but may feel discomfort or indigestion. They often present with a triad of symptoms—indigestion or feeling of abdominal fullness, feeling of chronic fatigue despite adequate rest, and feeling unable to catch their breath. Frequently, women are not diagnosed and therefore are not treated adequately. Hypertension and abdominal pain are not associated with acute coronary syndrome.

A hospital prepares for a mass casualty event. Which functions are correctly paired with the personnel role? (Select all that apply.) a. Paramedic Decides the number, acuity, and resource needs of clients b. Hospital incident commander Assumes overall leadership for implementing the emergency plan c. Public information officer Provides advanced life support during transportation to the hospital d. Triage officer Rapidly evaluates each client to determine priorities for treatment e. Medical command physician Serves as a liaison between the health care facility and the media

ANS: B, D The hospital incident commander assumes overall leadership for implementing the emergency plan. The triage officer rapidly evaluates each client to determine priorities for treatment. The paramedic provides advanced life support during transportation to the hospital. The public information officer serves as a liaison between the health care facility and the media. The medical command physician decides the number, acuity, and resource needs of clients.

A nurse prepares a client for a pharmacologic stress echocardiogram. Which actions should the nurse take when preparing this client for the procedure? (Select all that apply.) a. Assist the provider to place a central venous access device. b. Prepare for continuous blood pressure and pulse monitoring. c. Administer the client's prescribed beta blocker. d. Give the client nothing by mouth 3 to 6 hours before the procedure. e. Explain to the client that dobutamine will simulate exercise for this examination.

ANS: B, D, E Clients receiving a pharmacologic stress echocardiogram will need peripheral venous access and continuous blood pressure and pulse monitoring. The client must be NPO 3 to 6 hours prior to the procedure. Education about dobutamine, which will be administered during the procedure, should be performed. Beta blockers are often held prior to the procedure.

The nurse is teaching a client with diabetes mellitus how to prevent or delay chronic kidney disease (CKD). Which client statements indicate a lack of understanding of the teaching? (Select all that apply.) a. "I need to decrease sodium, cholesterol, and protein in my diet." b. "My weight should be maintained at a body mass index of 30." c. "Smoking should be stopped as soon as I possibly can." d. "I can continue to take an aspirin every 4 to 8 hours for my pain." e. "I really only need to drink a couple of glasses of water each day."

ANS: B, D, E Weight should be maintained at a body mass index (BMI) of 22 to 25. A BMI of 30 indicates obesity. The use of nonsteroidal anti-inflammatory drugs such as aspirin should be limited to the lowest time at the lowest dose due to interference with kidney blood flow. The client should drink at least 2 liters of water daily. Diet adjustments should be made by restricting sodium, cholesterol, and protein. Smoking causes constriction of blood vessels and decreases kidney perfusion, so the client should stop smoking.

A nurse assesses a client with a fungal urinary tract infection (UTI). Which assessments should the nurse complete? (Select all that apply.) a. Palpate the kidneys and bladder. b. Assess the medical history and current medical problems. c. Perform a bladder scan to assess post-void residual. d. Inquire about recent travel to foreign countries. e. Obtain a current list of medications.

ANS: B, E Clients who are severely immunocompromised or who have diabetes mellitus are more prone to fungal UTIs. The nurse should assess for these factors by asking about medical history, current medical problems, and the current medication list. A physical examination and a post-void residual may be needed, but not until further information is obtained indicating that these examinations are necessary. Travel to foreign countries probably would not be important because, even if exposed, the client needs some degree of compromised immunity to develop a fungal UTI.

A nurse teaches clients about the difference between urge incontinence and stress incontinence. Which statements should the nurse include in this education? (Select all that apply.) a. "Urge incontinence involves a post-void residual volume less than 50 mL." b. "Stress incontinence occurs due to weak pelvic floor muscles." c. "Stress incontinence usually occurs in people with dementia." d. "Urge incontinence can be managed by increasing fluid intake." e. "Urge incontinence occurs due to abnormal bladder contractions."

ANS: B, E Clients who suffer from stress incontinence have weak pelvic floor muscles or urethral sphincter and cannot tighten their urethra sufficiently to overcome the increased detrusor pressure. Stress incontinence is common after childbirth, when the pelvic muscles are stretched and weakened from pregnancy and delivery. Urge incontinence occurs in people who cannot suppress the contraction signal from the detrusor muscle. Abnormal detrusor contractions may be a result of neurologic abnormalities including dementia, or may occur with no known abnormality. Post-void residual is associated with reflex incontinence, not with urge incontinence or stress incontinence. Management of urge incontinence includes decreasing fluid intake, especially in the evening hours.

3. A nurse is caring for a client after a stroke. What actions may the nurse delegate to the unlicensed assistive personnel (UAP)? (Select all that apply.) a. Assess neurologic status with the Glasgow Coma Scale. b. Check and document oxygen saturation every 1 to 2 hours. c. Cluster client care to allow periods of uninterrupted rest. d. Elevate the head of the bed to 45 degrees to prevent aspiration. e. Position the client supine with the head in a neutral midline position.

ANS: B, E The UAP can take and document vital signs, including oxygen saturation, and keep the clients head in a neutral, midline position with correct direction from the nurse. The nurse assesses the Glasgow Coma Scale score. The nursing staff should not cluster care because this can cause an increase in the intracranial pressure. The head of the bed should be minimally elevated, up to 30 degrees.

A hospital prepares to receive large numbers of casualties from a community disaster. Which clients should the nurse identify as appropriate for discharge or transfer to another facility? (Select all that apply.) a. Older adult in the medical decision unit for evaluation of chest pain b. Client who had open reduction and internal fixation of a femur fracture 3 days ago c. Client admitted last night with community-acquired pneumonia d. Infant who has a fever of unknown origin e. Client on the medical unit for wound care

ANS: B, E The client with the femur fracture could be transferred to a rehabilitation facility, and the client on the medical unit for wound care should be transferred home with home health or to a long-term care facility for ongoing wound care. The client in the medical decision unit should be identified for dismissal if diagnostic testing reveals a noncardiac source of chest pain. The newly admitted client with pneumonia would not be a good choice because culture results are not yet available and antibiotics have not been administered long enough. The infant does not have a definitive diagnosis.

A nurse cares for a client with chronic obstructive pulmonary disease (COPD) who appears thin and disheveled. Which question should the nurse ask first? a. "Do you have a strong support system?" b. "What do you understand about your disease?" c. "Do you experience shortness of breath with basic activities?" d. "What medications are you prescribed to take each day?"

ANS: C Clients with severe COPD may not be able to perform daily activities, including bathing and eating, because of excessive shortness of breath. The nurse should ask the client if shortness of breath is interfering with basic activities. Although the nurse should know about the client's support systems, current knowledge, and medications, these questions do not address the client's appearance.

The nurse is teaching a client with chronic obstructive pulmonary disease who has been prescribed continuous oxygen therapy at home. Which statement indicates the client correctly understands the teaching? a. "I plan to wear my oxygen when I exercise and feel short of breath." b. "I will use my portable oxygen when grilling burgers in the backyard." c. "I plan to use cotton balls to cushion the oxygen tubing on my ears." d. "I will only smoke while I am wearing my oxygen via nasal cannula."

ANS: C Cotton balls can decrease pressure ulcers from the oxygen tubing. Continuous oxygen orders mean the client should wear the oxygen at all times. Oxygen fuels a fire. Wearing oxygen while grilling and smoking increases the risk for fire.

After teaching a client who is prescribed a long-acting beta2 agonist medication, a nurse assesses the client's understanding. Which statement indicates the client comprehends the teaching? a. "I will carry this medication with me at all times in case I need it." b. "I will take this medication when I start to experience an asthma attack." c. "I will take this medication every morning to help prevent an acute attack." d. "I will be weaned off this medication when I no longer need it."

ANS: C Long-acting beta2 agonist medications will help prevent an acute asthma attack because they are long acting. The client will take this medication every day for best effect. The client does not have to always keep this medication with him or her because it is not used as a rescue medication. This is not the medication the client will use during an acute asthma attack because it does not have an immediate onset of action. The client will not be weaned off this medication because this is likely to be one of his or her daily medications.

A nurse cares for a client with chronic obstructive pulmonary disease (COPD). The client states that he no longer enjoys going out with his friends. How should the nurse respond? a. "There are a variety of support groups for people who have COPD." b. "I will ask your provider to prescribe you with an antianxiety agent." c. "Share any thoughts and feelings that cause you to limit social activities." d. "Friends can be a good support system for clients with chronic disorders."

ANS: C Many clients with moderate to severe COPD become socially isolated because they are embarrassed by frequent coughing and mucus production. They also can experience fatigue, which limits their activities. The nurse needs to encourage the client to verbalize thoughts and feelings so that appropriate interventions can be selected. Joining a support group would not decrease feelings of social isolation if the client does not verbalize feelings. Antianxiety agents will not help the client with social isolation. Encouraging a client to participate in activities without verbalizing concerns also would not be an effective strategy for decreasing social isolation.

After teaching a client who is prescribed salmeterol (Serevent), the nurse assesses the client's understanding. Which statement by the client indicates a need for additional teaching? a. "I will be certain to shake the inhaler well before I use it." b. "It may take a while before I notice a change in my asthma." c. "I will use the drug when I have an asthma attack." d. "I will be careful not to let the drug escape out of my nose and mouth."

ANS: C Salmeterol is designed to prevent an asthma attack; it does not relieve or reverse symptoms. Salmeterol has a slow onset of action; therefore, it should not be used as a rescue drug. The drug must be shaken well because it has a tendency to separate easily. Poor technique on the client's part allows the drug to escape through the nose and mouth.

A nurse cares for a client who has developed esophagitis after undergoing radiation therapy for lung cancer. Which diet selection should the nurse provide for this client? a. Spaghetti with meat sauce, ice cream b. Chicken soup, grilled cheese sandwich c. Omelet, soft whole wheat bread d. Pasta salad, custard, orange juice

ANS: C Side effects of radiation therapy may include inflammation of the esophagus. Clients should be taught that bland, soft, high-calorie foods are best, along with liquid nutritional supplements. Tomato sauce may prove too spicy for a client with esophagitis. A grilled cheese sandwich is too difficult to swallow with this condition, and orange juice and other foods with citric acid are too caustic.

The nurse instructs a client on how to correctly use an inhaler with a spacer. In which order should these steps occur? 1. "Press down firmly on the canister to release one dose of medication." 2. "Breathe in slowly and deeply." 3. "Shake the whole unit vigorously three or four times." 4. "Insert the mouthpiece of the inhaler into the nonmouthpiece end of the spacer." 5. "Place the mouthpiece into your mouth, over the tongue, and seal your lips tightly around the mouthpiece." 6. "Remove the mouthpiece from your mouth, keep your lips closed, and hold your breath for at least 10 seconds." a. 2, 3, 4, 5, 6, 1 b. 3, 4, 5, 1, 6, 2 c. 4, 3, 5, 1, 2, 6 d. 5, 3, 6, 1, 2, 4

ANS: C The proper order for correctly using an inhaler with a spacer is as follows. Insert the mouthpiece of the inhaler into the nonmouthpiece end of the spacer. Shake the whole unit vigorously three or four times. Place the mouthpiece into the mouth, over the tongue, and seal the lips tightly around it. Press down firmly on the canister of the inhaler to release one dose of medication into the spacer. Breathe in slowly and deeply. Remove the mouthpiece from the mouth, and, keeping the lips closed, hold the breath for at least 10 seconds. Then breathe out slowly. Wait at least 1 minute between puffs.

34.A nurse assesses a client who is being treated for hyperglycemic-hyperosmolar state (HHS). Which clinical manifestation indicates to the nurse that the therapy needs to be adjusted? a. Serum potassium level has increased. b. Blood osmolarity has decreased. c. Glasgow Coma Scale score is unchanged. d. Urine remains negative for ketone bodies.

ANS: C A slow but steady improvement in central nervous system functioning is the best indicator of therapy effectiveness for HHS. Lack of improvement in the level of consciousness may indicate inadequate rates of fluid replacement. The Glasgow Coma Scale assesses the clients state of consciousness against criteria of a scale including best eye, verbal, and motor responses. An increase in serum potassium, decreased blood osmolality, and urine negative for ketone bodies do not indicate adequacy of treatment.

A nurse assesses a client who has a 15-year history of diabetes and notes decreased tactile sensation in both feet. Which action should the nurse take first? a. Document the finding in the clients chart. b. Assess tactile sensation in the clients hands. c. Examine the clients feet for signs of injury. d. Notify the health care provider.

ANS: C Diabetic neuropathy is common when the disease is of long duration. The client is at great risk for injury in any area with decreased sensation because he or she is less able to feel injurious events. Feet are common locations for neuropathy and injury, so the nurse should inspect them for any signs of injury. After assessment, the nurse should document findings in the clients chart. Testing sensory perception in the hands may or may not be needed. The health care provider can be notified after assessment and documentation have been completed.

A nurse assesses a client who has a 15-year history of diabetes and notes decreased tactile sensation in both feet. Which action should the nurse take first? a. Document the finding in the clients chart. b. Assess tactile sensation in the clientshands. c. Examine the clients feet for signs of injury. d. Notify the health care provider.

ANS: C Diabetic neuropathy is common when the disease is of long duration. The client is at great risk for injury in any area with decreased sensation because he or she is less able to feel injurious events. Feet are common locations for neuropathy and injury, so the nurse should inspect them for any signs of injury. After assessment, the nurse should document findings in the clients chart. Testing sensory perception in the hands may or may not be needed. The health care provider can be notified after assessment and documentation have been completed.

A nurse has educated a client on isoniazid (INH). What statement by the client indicates teaching has been effective? a. "I need to take extra vitamin C while on INH." b. "I should take this medicine with milk or juice." c. "I will take this medication on an empty stomach." d. "My contact lenses will be permanently stained."

ANS: C INH needs to be taken on an empty stomach, either 1 hour before or 2 hours after meals. Extra vitamin B needs to be taken while on the drug. Staining of contact lenses commonly occurs while taking rifampin (Rifadin).

12. A nurse is caring for a client who has sleep apnea and is prescribed modafinil (Provigil). The client asks, How will this medication help me? How should the nurse respond? a. This medication will treat your sleep apnea. b. This sedative will help you to sleep at night. c. This medication will promote daytime wakefulness. d. This analgesic will increase comfort while you sleep.

ANS: C Modafinil is helpful for clients who have narcolepsy (uncontrollable daytime sleep) related to sleep apnea. This medication promotes daytime wakefulness.

A nurse is caring for several older clients in the hospital that the nurse identifies as being at high risk for healthcare-associated pneumonia. To reduce this risk, what activity should the nurse delegate to the unlicensed assistive personnel (UAP)? a. Encourage between-meal snacks. b. Monitor temperature every 4 hours. c. Provide oral care every 4 hours. d. Report any new onset of cough.

ANS: C Oral colonization by gram-negative bacteria is a risk factor for healthcare-associated pneumonia. Good, frequent oral care can help prevent this from developing and is a task that can be delegated to the UAP. Encouraging good nutrition is important, but this will not prevent pneumonia. Monitoring temperature and reporting new cough in clients is important to detect the onset of possible pneumonia but do not prevent it.

5. A nurse assesses clients on the medical-surgical unit. Which client is at greatest risk for development of obstructive sleep apnea? a. A 26-year-old woman who is 8 months pregnant b. A 42-year-old man with gastroesophageal reflux disease c. A 55-year-old woman who is 50 pounds overweight d. A 73-year-old man with type 2 diabetes mellitus

ANS: C The client at highest risk would be the one who is extremely overweight. None of the other clients have risk factors for sleep apnea.

40.After teaching a client who is newly diagnosed with type 2 diabetes mellitus, the nurse assesses the clients understanding. Which statement made by the client indicates a need for additional teaching? a. I should increase my intake of vegetables with higher amounts of dietary fiber. b. My intake of saturated fats should be no more than 10% of my total calorie intake. c. I should decrease my intake of protein and eliminate carbohydrates from my diet. d. My intake of water is not restricted by my treatment plan or medication regimen.

ANS: C The client should not completely eliminate carbohydrates from the diet, and should reduce protein if microalbuminuria is present. The client should increase dietary intake of complex carbohydrates, including vegetables, and decrease intake of fat. Water does not need to be restricted unless kidney failure is present.

45.At 4:45 p.m., a nurse assesses a client with diabetes mellitus who is recovering from an abdominal hysterectomy 2 days ago. The nurse notes that the client is confused and diaphoretic. The nurse reviews the assessment data provided in the chart below: Capillary Blood Glucose Testing (AC/HS) Dietary Intake At 0630: 95 At 1130: 70 At 1630: 47 Breakfast: 10% eaten client states she is not hungry Lunch: 5% eaten client is nauseous; vomits once After reviewing the clients assessment data, which action is appropriate at this time? a. Assess the clients oxygen saturation level and administer oxygen. b. Reorient the client and apply a cool washcloth to the clients forehead. c. Administer dextrose 50% intravenously and reassess the client. d. Provide a glass of orange juice and encourage the client to eat dinner.

ANS: C The clients symptoms are related to hypoglycemia. Since the client has not been tolerating food, the nurse should administer dextrose intravenously. The clients oxygen level could be checked, but based on the information provided, this is not the priority. The client will not be reoriented until the glucose level rises.

A client is admitted with suspected pneumonia from the emergency department. The client went to the primary care provider a "few days ago" and shows the nurse the results of what the client calls "an allergy test," as shown below: What action by the nurse takes priority? a. Assess the client for possible items to which he or she is allergic. b. Call the primary care provider's office to request records. c. Immediately place the client on Airborne Precautions. d. Prepare to begin administration of intravenous antibiotics.

ANS: C This "allergy test" is actually a positive tuberculosis test. The client should be placed on Airborne Precautions immediately. The other options do not take priority over preventing the spread of the disease.TB

A nurse plans care for an 83-year-old client who is experiencing age-related sensory perception changes. Which intervention should the nurse include in this client's plan of care? a. Provide a call button that requires only minimal pressure to activate. b. Write the date on the client's white board to promote orientation. c. Ensure that the path to the bathroom is free from equipment. d. Encourage the client to season food to stimulate nutritional intake.

ANS: C Dementia and confusion are not common phenomena in older adults. However, physical impairment related to illness can be expected. Providing opportunities for hazard-free ambulation will maintain strength and mobility (and ensure safety). Providing a call button, providing the date, and seasoning food do not address the client's impaired sensory perception. DIF: Applying/Application REF: 840 KEY: Patient safety| fall prevention| older adult MSC: Integrated Process: Nursing Process: Implementation NOT: Client Needs Category: Physiological Integrity: Basic Care and Comfort

7. A telemetry nurse assesses a client with third-degree heart block who has wide QRS complexes and a heart rate of 35 beats/min on the cardiac monitor. Which assessment should the nurse complete next? a. Pulmonary auscultation b. Pulse strength and amplitude c. Level of consciousness d. Mobility and gait stability

ANS: C A heart rate of 40 beats/min or less with widened QRS complexes could have hemodynamic consequences. The client is at risk for inadequate cerebral perfusion. The nurse should assess for level of consciousness, light- headedness, confusion, syncope, and seizure activity.

A nurse assesses a client who is recovering after a left-sided cardiac catheterization. Which assessment finding requires immediate intervention? a. Urinary output less than intake b. Bruising at the insertion site c. Slurred speech and confusion d. Discomfort in the left leg

ANS: C A left-sided cardiac catheterization specifically increases the risk for a cerebral vascular accident. A change in neurologic status needs to be acted on immediately. Discomfort and bruising are expected at the site. If intake decreases, a client can become dehydrated because of dye excretion. The second intervention would be to increase the client's fluid status. Neurologic changes would take priority.

A confused client with pneumonia is admitted with an indwelling catheter in place. During interdisciplinary rounds the following day, which question should the nurse ask the primary health care provider? a. "Do you want daily weights on this client?" b. "Will the client be able to return home?" c. "Can we discontinue the indwelling catheter?" d. "Should we get another chest x-ray today?"

ANS: C An indwelling catheter dramatically increases the risks of urinary tract infection and urosepsis. Nursing staff should ensure that catheters are left in place only as long as they are medically needed. The nurse should inquire about removing the catheter. All other questions might be appropriate, but because of client safety, this question takes priority.

A client has a continuous passive motion (CPM) device after a total knee replacement. What action does the nurse delegate to the unlicensed assistive personnel (UAP) after the affected leg is placed in the machine while the client is in bed? a. Assess the distal circulation in 30 minutes. b. Change the settings based on range of motion. c. Raise the lower siderail on the affected side. d. Remind the client to do quad-setting exercises.

ANS: C Because the client's leg is strapped into the CPM, if it falls off the bed due to movement, the client's leg (and new joint) can be injured. The nurse should instruct the UAP to raise the siderail to prevent this from occurring. Assessment is a nursing responsibility. Only the surgeon, physical therapist, or specially trained technician adjusts the CPM settings. Quad-setting exercises are not related to the CPM machine.

After teaching a client who is recovering from a heart transplant to change positions slowly, the client asks, "Why is this important?" How should the nurse respond? a. "Rapid position changes can create shear and friction forces, which can tear out your internal vascular sutures." b. "Your new vascular connections are more sensitive to position changes, leading to increased intravascular pressure and dizziness." c. "Your new heart is not connected to the nervous system and is unable to respond to decreases in blood pressure caused by position changes." d. "While your heart is recovering, blood flow is diverted away from the brain, increasing the risk for stroke when you stand up."

ANS: C Because the new heart is denervated, the baroreceptor and other mechanisms that compensate for blood pressure drops caused by position changes do not function. This allows orthostatic hypotension to persist in the postoperative period. The other options are false statements and do not correctly address the client's question.

11. A nurse cares for a client with atrial fibrillation who reports fatigue when completing activities of daily living. What interventions should the nurse implement to address this clients concerns? a. Administer oxygen therapy at 2 liters per nasal cannula. b. Provide the client with a sleeping pill to stimulate rest. c. Schedule periods of exercise and rest during the day. d. Ask unlicensed assistive personnel to help bathe the client.

ANS: C Clients who have atrial fibrillation are at risk for decreased cardiac output and fatigue when completing activities of daily living. The nurse should schedule periods of exercise and rest during the day to decrease fatigue. The other interventions will not assist the client with self-care activities.

16. The nurse asks a client who has experienced ventricular dysrhythmias about substance abuse. The client asks, Why do you want to know if I use cocaine? How should the nurse respond? a. Substance abuse puts clients at risk for many health issues. b. The hospital requires that I ask you about cocaine use. c. Clients who use cocaine are at risk for fatal dysrhythmias. d. We can provide services for cessation of substance abuse.

ANS: C Clients who use cocaine or illicit inhalants are particularly at risk for potentially fatal dysrhythmias. The other responses do not adequately address the clients question.

A nurse assesses a client in an outpatient clinic. Which statement alerts the nurse to the possibility of left-sided heart failure? a. "I have been drinking more water than usual." b. "I am awakened by the need to urinate at night." c. "I must stop halfway up the stairs to catch my breath." d. "I have experienced blurred vision on several occasions."

ANS: C Clients with left-sided heart failure report weakness or fatigue while performing normal activities of daily living, as well as difficulty breathing, or "catching their breath." This occurs as fluid moves into the alveoli. Nocturia is often seen with right-sided heart failure. Thirst and blurred vision are not related to heart failure.

4. A nurse cares for a middle-aged female client with diabetes mellitus who is being treated for the third episode of acute pyelonephritis in the past year. The client asks, "What can I do to help prevent these infections?" How should the nurse respond? a. "Test your urine daily for the presence of ketone bodies and proteins." b. "Use tampons rather than sanitary napkins during your menstrual period." c. "Drink more water and empty your bladder more frequently during the day." d. "Keep your hemoglobin A1c under 9% by keeping your blood sugar controlled."

ANS: C Clients with long-standing diabetes mellitus are at risk for pyelonephritis for many reasons. Chronically elevated blood glucose levels spill glucose into the urine, changing the pH and providing a favorable climate for bacterial growth. The neuropathy associated with diabetes reduces bladder tone and reduces the client's sensation of bladder fullness. Thus, even with large amounts of urine, the client voids less frequently, allowing stasis and overgrowth of microorganisms. Increasing fluid intake (specifically water) and voiding frequently prevent stasis and bacterial overgrowth. Testing urine and using tampons will not help prevent pyelonephritis. A hemoglobin A1c of 9% is too high.

The nurse is taking the vital signs of a client after hemodialysis. Blood pressure is 110/58 mm Hg, pulse 66 beats/min, and temperature is 99.8° F (37.6° C). What is the most appropriate action by the nurse? a. Administer fluid to increase blood pressure. b. Check the white blood cell count. c. Monitor the client's temperature. d. Connect the client to an electrocardiographic (ECG) monitor.

ANS: C During hemodialysis, the dialysate is warmed to increase diffusion and prevent hypothermia. The client's temperature could reflect the temperature of the dialysate. There is no indication to check the white blood cell count or connect the client to an ECG monitor. The other vital signs are within normal limits.

A nurse assesses a client with renal insufficiency and a low red blood cell count. The client asks, "Is my anemia related to the renal insufficiency?" How should the nurse respond? a. "Red blood cells produce erythropoietin, which increases blood flow to the kidneys." b. "Your anemia and renal insufficiency are related to inadequate vitamin D and a loss of bone density." c. "Erythropoietin is usually released from the kidneys and stimulates red blood cell production in the bone marrow." d. "Kidney insufficiency inhibits active transportation of red blood cells throughout the blood."

ANS: C Erythropoietin is produced in the kidney and is released in response to decreased oxygen tension in the renal blood supply. Erythropoietin stimulates red blood cell production in the bone marrow. Anemia and renal insufficiency are not manifestations of vitamin D deficiency. The kidneys do not play a role in the transportation of red blood cells or any other cells in the blood.

A client has newly diagnosed systemic lupus erythematosus (SLE). What instruction by the nurse is most important? a. "Be sure you get enough sleep at night." b. "Eat plenty of high-protein, high-iron foods." c. "Notify your provider at once if you get a fever." d. "Weigh yourself every day on the same scale."

ANS: C Fever is the classic sign of a lupus flare and should be reported immediately. Rest and nutrition are important but do not take priority over teaching the client what to do if he or she develops an elevated temperature. Daily weights may or may not be important depending on renal involvement.

8. A client experiences impaired swallowing after a stroke and has worked with speech-language pathology on eating. What nursing assessment best indicates that a priority goal for this problem has been met? a. Chooses preferred items from the menu b. Eats 75% to 100% of all meals and snacks c. Has clear lung sounds on auscultation d. Gains 2 pounds after 1 week

ANS: C Impaired swallowing can lead to aspiration, so the priority goal for this problem is no aspiration. Clear lung sounds is the best indicator that aspiration has not occurred. Choosing menu items is not related to this problem. Eating meals does not indicate the client is not still aspirating. A weight gain indicates improved nutrition but still does not show a lack of aspiration.

A nurse cares for clients during a community-wide disaster drill. Once of the clients asks, Why are the individuals with black tags not receiving any care? How should the nurse respond? a. To do the greatest good for the greatest number of people, it is necessary to sacrifice some. b. Not everyone will survive a disaster, so it is best to identify those people early and move on. c. In a disaster, extensive resources are not used for one person at the expense of many others. d. With black tags, volunteers can identify those who are dying and can give them comfort care.

ANS: C In a disaster, military-style triage is used; this approach identifies the dead or expectant dead with black tags. This practice helps to maintain the goal of triage, which is doing the most good for the most people. Precious resources are not used for those with overwhelming critical injury or illness, so that they can be allocated to others who have a reasonable expectation of survival. Clients are not sacrificed. Telling students to move on after identifying the expectant dead belittles their feelings and does not provide an adequate explanation. Clients are not black-tagged to allow volunteers to give comfort care.

A nurse assesses an older adult client who is experiencing a myocardial infarction. Which clinical manifestation should the nurse expect? a. Excruciating pain on inspiration b. Left lateral chest wall pain c. Disorientation and confusion d. Numbness and tingling of the arm

ANS: C In older adults, disorientation or confusion may be the major manifestation of myocardial infarction caused by poor cardiac output. Pain manifestations and numbness and tingling of the arm could also be related to the myocardial infarction. However, the nurse should be more concerned about the new onset of disorientation or confusion caused by decreased perfusion.

The nurse is teaching a client how to increase the flow of dialysate into the peritoneal cavity during dialysis. Which statement by the client demonstrates a correct understanding of the teaching? a. "I should leave the drainage bag above the level of my abdomen." b. "I could flush the tubing with normal saline if the flow stops." c. "I should take a stool softener every morning to avoid constipation." d. "My diet should have low fiber in it to prevent any irritation."

ANS: C Inflow and outflow problems of the dialysate are best controlled by preventing constipation. A daily stool softener is the best option for the client. The drainage bag should be below the level of the abdomen. Flushing the tubing will not help with the flow. A diet high in fiber will also help with a constipation problem.

A nurse obtains a sterile urine specimen from a client's Foley catheter. After applying a clamp to the drainage tubing distal to the injection port, which action should the nurse take next? a. Clamp another section of the tube to create a fixed sample section for retrieval. b. Insert a syringe into the injection port and aspirate the quantity of urine required. c. Clean the injection port cap of the drainage tubing with povidone-iodine solution. d. Withdraw 10 mL of urine and discard it; then withdraw a fresh sample of urine.

ANS: C It is important to clean the injection port cap of the catheter drainage tubing with an appropriate antiseptic, such as povidone-iodine solution or alcohol. This will help prevent surface contamination before injection of the syringe. The urine sample should be collected directly from the catheter; therefore, a second clamp to create a sample section would not be appropriate. Every sample from the catheter is usable; there is the need to discard the first sample.

A nurse assesses a client 2 hours after a cardiac angiography via the left femoral artery. The nurse notes that the left pedal pulse is weak. Which action should the nurse take? a. Elevate the leg and apply a sandbag to the entrance site. b. Increase the flow rate of intravenous fluids. c. Assess the color and temperature of the left leg. d. Document the finding as "left pedal pulse of +1/4."

ANS: C Loss of a pulse distal to an angiography entry site is serious, indicating a possible arterial obstruction. The pulse may be faint because of edema. The left pulse should be compared with the right, and pulses should be compared with previous assessments, especially before the procedure. Assessing color (pale, cyanosis) and temperature (cool, cold) will identify a decrease in circulation. Once all peripheral and vascular assessment data are acquired, the primary health care provider should be notified. Simply documenting the findings is inappropriate. The leg should be positioned below the level of the heart or dangling to increase blood flow to the distal portion of the leg. Increasing intravenous fluids will not address the client's problem.

The nurse working in the orthopedic clinic knows that a client with which factor has an absolute contraindication for having a total joint replacement? a. Needs multiple dental fillings b. Over age 85 c. Severe osteoporosis d. Urinary tract infection

ANS: C Osteoporosis is a contraindication to joint replacement because the bones have a high risk of shattering as the new prosthesis is implanted. The client who needs fillings should have them done prior to the surgery. Age greater than 85 is not an absolute contraindication. A urinary tract infection can be treated prior to surgery.

After teaching a client with bacterial cystitis who is prescribed phenazopyridine (Pyridium), the nurse assesses the client's understanding. Which statement made by the client indicates a correct understanding of the teaching? a. "I will not take this drug with food or milk." b. "If I think I am pregnant, I will stop the drug." c. "An orange color in my urine should not alarm me." d. "I will drink two glasses of cranberry juice daily."

ANS: C Phenazopyridine discolors urine, most commonly to a deep reddish orange. Many clients think they have blood in their urine when they see this. In addition, the urine can permanently stain clothing. Phenazopyridine is safe to take if the client is pregnant. There are no dietary restrictions or needs while taking this medication.

A client is placed on fluid restrictions because of chronic kidney disease (CKD). Which assessment finding would alert the nurse that the client's fluid balance is stable at this time? a. Decreased calcium levels b. Increased phosphorus levels c. No adventitious sounds in the lungs d. Increased edema in the legs

ANS: C The absence of adventitious sounds upon auscultation of the lungs indicates a lack of fluid overload and fluid balance in the client's body. Decreased calcium levels and increased phosphorus levels are common findings with CKD. Edema would indicate a fluid imbalance.

The charge nurse of the medical-surgical unit is making staff assignments. Which staff member should be assigned to a client with chronic kidney disease who is exhibiting a low-grade fever and a pericardial friction rub? a. Registered nurse who just floated from the surgical unit b. Registered nurse who just floated from the dialysis unit c. Registered nurse who was assigned the same client yesterday d. Licensed practical nurse with 5 years' experience on this floor

ANS: C The client is exhibiting symptoms of pericarditis, which can occur with chronic kidney disease. Continuity of care is important to assess subtle differences in clients. Therefore, the registered nurse (RN) who was assigned to this client previously should again give care to this client. The float nurses would not be as knowledgeable about the unit and its clients. The licensed practical nurse may not have the education level of the RN to assess for pericarditis.

26. A nurse is caring for four clients in the neurologic/neurosurgical intensive care unit. Which client should the nurse assess first? a. Client who has been diagnosed with meningitis with a fever of 101 F (38.3 C) b. Client who had a transient ischemic attack and is waiting for teaching on clopidogrel (Plavix) c. Client receiving tissue plasminogen activator (t-PA) who has a change in respiratory pattern and rate d. Client who is waiting for subarachnoid bolt insertion with the consent form already signed

ANS: C The client receiving t-PA has a change in neurologic status while receiving this fibrinolytic therapy. The nurse assesses this client first as he or she may have an intracerebral bleed. The client with meningitis has expected manifestations. The client waiting for discharge teaching is a lower priority. The client waiting for surgery can be assessed quickly after the nurse sees the client who is receiving t-PA, or the nurse could delegate checking on this client to another nurse.

A nurse assesses clients on a medical-surgical unit. Which client should the nurse identify as having the greatest risk for cardiovascular disease? a. An 86-year-old man with a history of asthma b. A 32-year-old Asian-American man with colorectal cancer c. A 45-year-old American Indian woman with diabetes mellitus d. A 53-year-old postmenopausal woman who is on hormone therapy

ANS: C The incidence of coronary artery disease and hypertension is higher in American Indians than in whites or Asian Americans. Diabetes mellitus increases the risk for hypertension and coronary artery disease in people of any race or ethnicity. Asthma, colorectal cancer, and hormone therapy do not increase risk for cardiovascular disease.

A nurse prepares a client for coronary artery bypass graft surgery. The client states, "I am afraid I might die." How should the nurse respond? a. "This is a routine surgery and the risk of death is very low." b. "Would you like to speak with a chaplain prior to surgery?" c. "Tell me more about your concerns about the surgery." d. "What support systems do you have to assist you?"

ANS: C The nurse should discuss the client's feelings and concerns related to the surgery. The nurse should not provide false hope or push the client's concerns off on the chaplain. The nurse should address support systems after addressing the client's current issue.

The charge nurse is orienting a float nurse to an assigned client with an arteriovenous (AV) fistula for hemodialysis in her left arm. Which action by the float nurse would be considered unsafe? a. Palpating the access site for a bruit or thrill b. Using the right arm for a blood pressure reading c. Administering intravenous fluids through the AV fistula d. Checking distal pulses in the left arm

ANS: C The nurse should not use the arm with the AV fistula for intravenous infusion, blood pressure readings, or venipuncture. Compression and infection can result in the loss of the AV fistula. The AV fistula should be monitored by auscultating or palpating the access site. Checking the distal pulse would be an appropriate assessment.

An emergency department charge nurse notes an increase in sick calls and bickering among the staff after a week with multiple trauma incidents. Which action should the nurse take? a. Organize a pizza party for each shift. b. Remind the staff of the facilitys sick-leave policy. c. Arrange for critical incident stress debriefing. d. Talk individually with staff members.

ANS: C The staff may be suffering from critical incident stress and needs to have a debriefing by the critical incident stress management team to prevent the consequences of long-term, unabated stress. Speaking with staff members individually does not provide the same level of support as a group debriefing. Organizing a party and revisiting the sick-leave policy may be helpful, but are not as important and beneficial as a debriefing.

A nurse is caring for a client whose wife died in a recent mass casualty accident. The client says, I cant believe that my wife is gone and I am left to raise my children all by myself. How should the nurse respond? a. Please accept my sympathies for your loss. b. I can call the hospital chaplain if you wish. c. You sound anxious about being a single parent. d. At least your children still have you in their lives.

ANS: C Therapeutic communication includes active listening and honesty. This statement demonstrates that the nurse recognizes the clients distress and has provided an opening for discussion. Extending sympathy and offering to call the chaplain do not give the client the opportunity to discuss feelings. Stating that the children still have one parent discounts the clients feelings and situation.

A client is diagnosed with chronic kidney disease (CKD). What is an ideal goal of treatment set by the nurse in the care plan to reduce the risk of pulmonary edema? a. Maintaining oxygen saturation of 89% b. Minimal crackles and wheezes in lung sounds c. Maintaining a balanced intake and output d. Limited shortness of breath upon exertion

ANS: C With an optimal fluid balance, the client will be more able to eject blood from the left ventricle without increased pressure in the left ventricle and pulmonary vessels. Other ideal goals are oxygen saturations greater than 92%, no auscultated crackles or wheezes, and no demonstrated shortness of breath.

After a total knee replacement, a client is on the postoperative nursing unit with a continuous femoral nerve blockade. On assessment, the nurse notes the client's pulses are 2+/4+ bilaterally; the skin is pale pink, warm, and dry; and the client is unable to dorsiflex or plantarflex the affected foot. What action does the nurse perform next? a. Document the findings and monitor as prescribed. b. Increase the frequency of monitoring the client. c. Notify the surgeon or anesthesia provider immediately. d. Palpate the client's bladder or perform a bladder scan.

ANS: C With the femoral nerve block, the client should still be able to dorsiflex and plantarflex the affected foot. Since this client has an abnormal finding, the nurse should notify either the surgeon or the anesthesia provider immediately. Documentation is the last priority. Increasing the frequency of assessment may be a good idea, but first the nurse must notify the appropriate person. Palpating the bladder is not related.

A nurse assesses a client with asthma and notes bilateral wheezing, decreased pulse oxygen saturation, and suprasternal retraction on inhalation. Which actions should the nurse take? (Select all that apply.) a. Administer prescribed salmeterol (Serevent) inhaler. b. Assess the client for a tracheal deviation. c. Administer oxygen to keep saturations greater than 94%. d. Perform peak expiratory flow readings. e. Administer prescribed albuterol (Proventil) inhaler.

ANS: C, E Suprasternal retraction caused by inhalation usually indicates that the client is using accessory muscles and is having difficulty moving air into the respiratory passages because of airway narrowing. Wheezing indicates a narrowed airway; a decreased pulse oxygen saturation also supports this finding. The asthma is not responding to the medication, and intervention is needed. Administration of a rescue inhaler is indicated, probably along with administration of oxygen. The nurse would not do a peak flow reading at this time, nor would a code be called. Midline trachea is a normal and expected finding.

A nurse cares for a client who had a chest tube placed 6 hours ago and refuses to take deep breaths because of the pain. Which action should the nurse take? a. Ambulate the client in the hallway to promote deep breathing. b. Auscultate the client's anterior and posterior lung fields. c. Encourage the client to take shallow breaths to help with the pain. d. Administer pain medication and encourage the client to take deep breaths.

ANS: D A chest tube is placed in the pleural space and may be uncomfortable for a client. The nurse should provide pain medication to minimize discomfort and encourage the client to take deep breaths. The other responses do not address the client's discomfort and need to take deep breaths to prevent complications.

A nurse cares for a client who has a chest tube. When would this client be at highest risk for developing a pneumothorax? a. When the insertion site becomes red and warm to the touch b. When the tube drainage decreases and becomes sanguineous c. When the client experiences pain at the insertion site d. When the tube becomes disconnected from the drainage system

ANS: D Intrathoracic pressures are less than atmospheric pressures; therefore, if the chest tube becomes disconnected from the drainage system, air can be sucked into the pleural space and cause a pneumothorax. A red, warm, and painful insertion site does not increase the client's risk for a pneumothorax. Tube drainage should decrease and become serous as the client heals. Sanguineous drainage is a sign of bleeding but does not increase the client's risk for a pneumothorax.

The nurse is caring for a client who is prescribed a long-acting beta2 agonist. The client states, "The medication is too expensive to use every day. I only use my inhaler when I have an attack." How should the nurse respond? a. "You are using the inhaler incorrectly. This medication should be taken daily." b. "If you decrease environmental stimuli, it will be okay for you to use the inhaler only for asthma attacks." c. "Tell me more about your fears related to feelings of breathlessness." d. "It is important to use this type of inhaler every day. Let's identify potential community services to help you."

ANS: D Long-acting beta2 agonists should be used every day to prevent asthma attacks. This medication should not be taken when an attack starts. Asthma medications can be expensive. Telling the client that he or she is using the inhaler incorrectly does not address the client's financial situation, which is the main issue here. Clients with limited incomes should be provided with community resources. Asking the client about fears related to breathlessness does not address the client's immediate concerns.

A nurse evaluates the following arterial blood gas and vital sign results for a client with chronic obstructive pulmonary disease (COPD): Arterial Blood Gas Results Vital Signs pH = 7.32 PaCO2 = 62 mm Hg PaO2 = 46 mm Hg HCO3- = 28 mEq/L Heart rate = 110 beats/min Respiratory rate = 12 breaths/min Blood pressure = 145/65 mm Hg Oxygen saturation = 76% Which action should the nurse take first? a. Administer a short-acting beta2 agonist inhaler. b. Document the findings as normal for a client with COPD. c. Teach the client diaphragmatic breathing techniques. d. Initiate oxygenation therapy to increase saturation to 92%.

ANS: D Oxygen should be administered to a client who is hypoxic even if the client has COPD and is a carbon dioxide retainer. The other interventions do not address the client's hypoxia, which is the priority.

A nurse assesses several clients who have a history of asthma. Which client should the nurse assess first? a. A 66-year-old client with a barrel chest and clubbed fingernails b. A 48-year-old client with an oxygen saturation level of 92% at rest c. A 35-year-old client who has a longer expiratory phase than inspiratory phase d. A 27-year-old client with a heart rate of 120 beats/min

ANS: D Tachycardia can indicate hypoxemia as the body tries to circulate the oxygen that is available. A barrel chest is not an emergency finding. Likewise, a pulse oximetry level of 92% is not considered an acute finding. The expiratory phase is expected to be longer than the inspiratory phase in someone with airflow limitation.

2.A nurse assesses a client who has a radial artery catheter. Which assessment should the nurse complete first? a. Amount of pressure in fluid container b. Date of catheter tubing change c. Percent of heparin in infusion container d. Presence of an ulnar pulse

ANS: D An intra-arterial catheter may cause arterial occlusion, which can lead to absent or decreased perfusion to the extremity. Assessment of an ulnar pulse is one way to assess circulation to the arm in which the catheter is located. The nurse would note that there is enough pressure in the fluid container to keep the system flushed, and would check to see whether the catheter tubing needs to be changed. However, these are not assessments of greatest concern. Because of heparin-induced thrombocytopenia, heparin is not used in most institutions for an arterial catheter.

3. A nurse assesses a client who reports waking up feeling very tired, even after 8 hours of good sleep. Which action should the nurse take first? a. Contact the provider for a prescription for sleep medication. b. Tell the client not to drink beverages with caffeine before bed. c. Educate the client to sleep upright in a reclining chair. d. Ask the client if he or she has ever been evaluated for sleep apnea.

ANS: D Clients are usually unaware that they have sleep apnea, but it should be suspected in people who have persistent daytime sleepiness and report waking up tired. Causes of the problem should be assessed before the client is offered suggestions for treatment.

27.A nurse teaches a client with diabetes mellitus who is experiencing numbness and reduced sensation. Which statement should the nurse include in this clients teaching to prevent injury? a. Examine your feet using a mirror every day. b. Rotate your insulin injection sites every week. c. Check your blood glucose level before each meal. d. Use a bath thermometer to test the water temperature.

ANS: D Clients with diminished sensory perception can easily experience a burn injury when bathwater is too hot. Instead of checking the temperature of the water by feeling it, they should use a thermometer. Examining the feet daily does not prevent injury, although daily foot examinations are important to find problems so they can be addressed. Rotating insulin and checking blood glucose levels will not prevent injury.

6. A nurse assesses a client who had an intraosseous catheter placed in the left leg. Which assessment finding is of greatest concern? a. The catheter has been in place for 20 hours. b. The client has poor vascular access in the upper extremities. c. The catheter is placed in the proximal tibia. d. The clients left lower extremity is cool to the touch.

ANS: D Compartment syndrome is a condition in which increased tissue perfusion in a confined anatomic space causes decreased blood flow to the area. A cool extremity can signal the possibility of this syndrome. All other findings are important; however, the possible development of compartment syndrome requires immediate intervention because the client could require amputation of the limb if the nurse does not correctly assess this perfusion problem.

37.When teaching a client recently diagnosed with type 1 diabetes mellitus, the client states, I will never be able to stick myself with a needle. How should the nurse respond? a. I can give your injections to you while you are here in the hospital. b. Everyone gets used to giving themselves injections. It really does not hurt. c. Your disease will not be managed properly if you refuse to administer the shots. d. Tell me what it is about the injections that are concerning you.

ANS: D Devote as much teaching time as possible to insulin injection and blood glucose monitoring. Clients with newly diagnosed diabetes are often fearful of giving themselves injections. If the client is worried about giving the injections, it is best to try to find out what specifically is causing the concern, so it can be addressed. Giving the injections for the client does not promote self-care ability. Telling the client that others give themselves injections may cause the client to feel bad. Stating that you dont know another way to manage the disease is dismissive of the clients concerns.

A nurse assesses clients who are at risk for diabetes mellitus. Which client is at greatest risk? a. A 29-year-old Caucasian b. A 32-year-old African-American c. A 44-year-old Asian d. A 48-year-old American Indian

ANS: D Diabetes is a particular problem among African Americans, Hispanics, and American Indians. The incidence of diabetes increases in all races and ethnic groups with age. Being both an American Indian and middle-aged places this client at highest risk.

After teaching a young adult client who is newly diagnosed with type 1 diabetes mellitus, the nurse assesses the clients understanding. Which statement made by the client indicates a correct understanding of the need for eye examinations? a. At my age, I should continue seeing the ophthalmologist as I usually do. b. I will see the eye doctor when I have a vision problem and yearly after age 40. c. My vision will change quickly. I should see the ophthalmologist twice a year. d. Diabetes can cause blindness, so I should see the ophthalmologist yearly.

ANS: D Diabetic retinopathy is a leading cause of blindness in North America. All clients with diabetes, regardless of age, should be examined by an ophthalmologist (rather than an optometrist or optician) at diagnosis and at least yearly thereafter.

17.After teaching a client who has diabetes mellitus and proliferative retinopathy, nephropathy, and peripheral neuropathy, the nurse assesses the clients understanding. Which statement made by the client indicates a correct understanding of the teaching? a. I have so many complications; exercising is not recommended. b. I will exercise more frequently because I have so many complications. c. I used to run for exercise; I will start training for a marathon. d. I should look into swimming or water aerobics to get my exercise.

ANS: D Exercise is not contraindicated for this client, although modifications based on existing pathology are necessary to prevent further injury. Swimming or water aerobics will give the client exercise without the worry of having the correct shoes or developing a foot injury. The client should not exercise too vigorously.

39.A nurse reviews the medication list of a client recovering from a computed tomography (CT) scan with IV contrast to rule out small bowel obstruction. Which medication should alert the nurse to contact the provider and withhold the prescribed dose? a. Pioglitazone (Actos) b. Glimepiride (Amaryl) c. Glipizide (Glucotrol) d. Metformin (Glucophage)

ANS: D Glucophage should not be administered when the kidneys are attempting to excrete IV contrast from the body. This combination would place the client at high risk for kidney failure. The nurse should hold the metformin dose and contact the provider. The other medications are safe to administer after receiving IV contrast.

16.A nurse teaches a client who is prescribed an insulin pump. Which statement should the nurse include in this clients discharge education? a. Test your urine daily for ketones. b. Use only buffered insulin in your pump. c. Store the insulin in the freezer until you need it. d. Change the needle every 3 days.

ANS: D Having the same needle remain in place through the skin for longer than 3 days drastically increases the risk for infection in or through the delivery system. Having an insulin pump does not require the client to test for ketones in the urine. Insulin should not be frozen. Insulin is not buffered.

A nurse reviews laboratory results for a client with diabetes mellitus who presents with polyuria, lethargy, and a blood glucose of 560 mg/dL. Which laboratory result should the nurse correlate with the clients polyuria? a. Serum sodium: 163 mEq/L b. Serum creatinine: 1.6 mg/dL c. Presence of urine ketone bodies d. Serum osmolarity: 375 mOsm/kg

ANS: D Hyperglycemia causes hyperosmolarity of extracellular fluid. This leads to polyuria from an osmotic diuresis. The clients serum osmolarity is high. The clients sodium would be expected to be high owing to dehydration. Serum creatinine and urine ketone bodies are not related to the polyuria.

8.A nurse assesses a clients peripheral IV site, and notices edema and tenderness above the site. Which action should the nurse take next? a. Apply cold compresses to the IV site. b. Elevate the extremity on a pillow. c. Flush the catheter with normal saline. d. Stop the infusion of intravenous fluids.

ANS: D Infiltration occurs when the needle dislodges partially or completely from the vein. Signs of infiltration include edema and tenderness above the site. The nurse should stop the infusion and remove the catheter. Cold compresses and elevation of the extremity can be done after the catheter is discontinued to increase client comfort. Alternatively, warm compresses may be prescribed per institutional policy and may help speed circulation to the area.

32.A nurse reviews the laboratory results of a client who is receiving intravenous insulin. Which should alert the nurse to intervene immediately? a. Serum chloride level of 98 mmol/L b. Serum calcium level of 8.8 mg/dL c. Serum sodium level of 132 mmol/L d. Serum potassium level of 2.5 mmol/L

ANS: D Insulin activates the sodium-potassium ATPase pump, increasing the movement of potassium from the extracellular fluid into the intracellular fluid, resulting in hypokalemia. In hyperglycemia, hypokalemia can also result from excessive urine loss of potassium. The chloride level is normal. The calcium and sodium levels are slightly low, but this would not be related to hyperglycemia and insulin administration.

22.A nurse teaches a client with type 1 diabetes mellitus. Which statement should the nurse include in this clients teaching to decrease the clients insulin needs? a. Limit your fluid intake to 2 liters a day. b. Animal organ meat is high in insulin. c. Limit your carbohydrate intake to 80 grams a day. d. Walk at a moderate pace for 1 mile daily.

ANS: D Moderate exercise such as walking helps regulate blood glucose levels on a daily basis and results in lowered insulin requirements for clients with type 1 diabetes mellitus. Restricting fluids and eating organ meats will not reduce insulin needs. People with diabetes need at least 130 grams of carbohydrates each day.

After teaching a client with type 2 diabetes mellitus who is prescribed nateglinide (Starlix), the nurse assesses the clients understanding. Which statement made by the client indicates a correct understanding of the prescribed therapy? a. Ill take this medicine during each of my meals. b. I must take this medicine in the morning when I wake. c. I will take this medicine before I go to bed. d. I will take this medicine immediately before I eat.

ANS: D Nateglinide is an insulin secretagogue that is designed to increase meal-related insulin secretion. It should be taken immediately before each meal. The medication should not be taken without eating as it will decrease the clients blood glucose levels. The medication should be taken before meals instead of during meals.

38.A nurse assesses a client with diabetes mellitus who self-administers subcutaneous insulin. The nurse notes a spongy, swelling area at the site the client uses most frequently for insulin injection. Which action should the nurse take? a. Apply ice to the site to reduce inflammation. b. Consult the provider for a new administration route. c. Assess the client for other signs of cellulitis. d. Instruct the client to rotate sites for insulin injection.

ANS: D The clients tissue has been damaged from continuous use of the same site. The client should be educated to rotate sites. The damaged tissue is not caused by cellulitis or any type infection, and applying ice may cause more damage to the tissue. Insulin can only be administered subcutaneously and intravenously. It would not be appropriate or practical to change the administration route.

A nurse cares for a client experiencing diabetic ketoacidosis who presents with Kussmaul respirations. Which action should the nurse take? a. Administration of oxygen via face mask b. Intravenous administration of 10% glucose c. Implementation of seizure precautions d. Administration of intravenous insulin

ANS: D The rapid, deep respiratory efforts of Kussmaul respirations are the bodys attempt to reduce the acids produced by using fat rather than glucose for fuel. Only the administration of insulin will reduce this type of respiration by assisting glucose to move into cells and to be used for fuel instead of fat. The client who is in ketoacidosis may not experience any respiratory impairment and therefore does not need additional oxygen. Giving the client glucose would be contraindicated. The client does not require seizure precautions.

13. A nurse cares for a client who has packing inserted for posterior nasal bleeding. Which action should the nurse take first? a. Assess the clients pain level. b. Keep the clients head elevated. c. Teach the client about the causes of nasal bleeding. d. Make sure the string is taped to the clients cheek.

ANS: D The string should be attached to the clients cheek to hold the packing in place. The nurse needs to make sure that this does not move because it can occlude the clients airway. The other options are good interventions, but ensuring that the airway is patent is the priority objective.

11.A nurse prepares to insert a peripheral venous catheter in an older adult client. Which action should the nurse take to protect the clients skin during this procedure? a. Lower the extremity below the level of the heart. b. Apply warm compresses to the extremity. c. Tap the skin lightly and avoid slapping. d. Place a washcloth between the skin and tourniquet.

ANS: D To protect the clients skin, the nurse should place a washcloth or the clients gown between the skin and tourniquet. The other interventions are methods to distend the vein but will not protect the clients skin.

21.A nurse cares for a client who has type 1 diabetes mellitus. The client asks, Is it okay for me to have an occasional glass of wine? How should the nurse respond? a. Drinking any wine or alcohol will increase your insulin requirements. b. Because of poor kidney function, people with diabetes should avoid alcohol. c. You should not drink alcohol because it will make you hungry and overeat. d. One glass of wine is okay with a meal and is counted as two fat exchanges.

ANS: D Under normal circumstances, blood glucose levels will not be affected by moderate use of alcohol when diabetes is well controlled. Because alcohol can induce hypoglycemia, it should be ingested with or shortly after a meal. One alcoholic beverage is substituted for two fat exchanges when caloric intake is calculated. Kidney function is not impacted by alcohol intake. Alcohol is not associated with increased hunger or overeating.

A client has a deep vein thrombosis ( DVT ). What comfort measure does the nurse delegate to the unlicensed assistive personnel? a. Ambulate the client. b. Apply a warm moist pack. c. Massage the client's leg. d. Provide an ice pack.

Apply a warm moist pack Warm moist packs will help with the pain of a DVT. Ambulation is not a comfort measure. Massaging the client's legs is contraindicated to prevent complications such as pulmonary embolism. Ice packs are not recommended for DVT

7.A nurse is assessing clients who have intravenous therapy prescribed. Which assessment finding for a client with a peripherally inserted central catheter (PICC) requires immediate attention? a. The initial site dressing is 3 days old. b. The PICC was inserted 4 weeks ago. c. A securement device is absent. d. Upper extremity swelling is noted.

ANS: D Upper extremity swelling could indicate infiltration, and the PICC will need to be removed. The initial dressing over the PICC site should be changed within 24 hours. This does not require immediate attention, but the swelling does. The dwell time for PICCs can be months or even years. Securement devices are being used more often now to secure the catheter in place and prevent complications such as phlebitis and infiltration. The IV should have one, but this does not take priority over the client whose arm is swollen.

A nurse assesses a client's recent memory. Which client statement confirms that the client's remote memory is intact? a. "A young girl wrapped in a shroud fell asleep on a bed of clouds." b. "I was born on April 3, 1967, in Johnstown Community Hospital." c. "Apple, chair, and pencil are the words you just stated." d. "I ate oatmeal with wheat toast and orange juice for breakfast."

ANS: D Asking clients about recent events that can be verified, such as what the client ate for breakfast, assesses the client's recent memory. The client's ability to make up a rhyme tests not memory, but rather a higher level of cognition. Asking clients about certain facts from the past that can be verified assesses remote or long-term memory. Asking the client to repeat words assesses the client's immediate memory. DIF: Applying/Application REF: 839 KEY: Memory| assessment/diagnostic examination MSC: Integrated Process: Nursing Process: Assessment NOT: Client Needs Category: Health Promotion and Maintenance

A client has a long history of hypertension. Which category of medications would the nurse expect to be ordered to avoid chronic kidney disease (CKD)? a. Antibiotic b. Histamine blocker c. Bronchodilator d. Angiotensin-converting enzyme (ACE) inhibitor

ANS: D ACE inhibitors stop the conversion of angiotensin I to the vasoconstrictor angiotensin II. This category of medication also blocks bradykinin and prostaglandin, increases renin, and decreases aldosterone, which promotes vasodilation and perfusion to the kidney. Antibiotics fight infection, histamine blockers decrease inflammation, and bronchodilators increase the size of the bronchi; none of these medications helps slow the progression of CKD in clients with hypertension.

An emergency department nurse triages clients who present with chest discomfort. Which client should the nurse plan to assess first? a. A 42-year-old female who describes her pain as a dull ache with numbness in her fingers b. A 49-year-old male who reports moderate pain that is worse on inspiration c. A 53-year-old female who reports substernal pain that radiates to her abdomen d. A 58-year-old male who describes his pain as intense stabbing that spreads across his chest

ANS: D All clients who have chest pain should be assessed more thoroughly. To determine which client should be seen first, the nurse must understand common differences in pain descriptions. Intense stabbing, vise-like substernal pain that spreads through the client's chest, arms, jaw, back, or neck is indicative of a myocardial infarction. The nurse should plan to see this client first to prevent cardiac cell death. A dull ache with numbness in the fingers is consistent with anxiety. Pain that gets worse with inspiration is usually related to a pleuropulmonary problem. Pain that spreads to the abdomen is often associated with an esophageal-gastric problem, especially when this pain is experienced by a male client. Female clients may experience abdominal discomfort with a myocardial event. Although clients with anxiety, pleuropulmonary, and esophageal-gastric problems should be seen, they are not a higher priority than myocardial infarction.

A nurse assesses a client who is scheduled for a cardiac catheterization. Which assessment should the nurse complete prior to this procedure? a. Client's level of anxiety b. Ability to turn self in bed c. Cardiac rhythm and heart rate d. Allergies to iodine-based agents

ANS: D Before the procedure, the nurse should ascertain whether the client has an allergy to iodine-containing preparations, such as seafood or local anesthetics. The contrast medium used during the procedure is iodine based. This allergy can cause a life-threatening reaction, so it is a high priority. Second, it is important for the nurse to assess anxiety, mobility, and baseline cardiac status.

A nurse assesses a client after administering a prescribed beta blocker. Which assessment should the nurse expect to find? a. Blood pressure increased from 98/42 mm Hg to 132/60 mm Hg b. Respiratory rate decreased from 25 breaths/min to 14 breaths/min c. Oxygen saturation increased from 88% to 96% d. Pulse decreased from 100 beats/min to 80 beats/min

ANS: D Beta blockers block the stimulation of beta1-adrenergic receptors. They block the sympathetic (fight-or-flight) response and decrease the heart rate (HR). The beta blocker will decrease HR and blood pressure, increasing ventricular filling time. It usually does not have effects on beta2-adrenergic receptor sites. Cardiac output will drop because of decreased HR.

23. A nurse is caring for a client who is scheduled for a dose of cefazolin and vitamins at this time. Hemodialysis for this client is also scheduled in 60 minutes. Which action by the nurse is best? a. Administer cefazolin since the level of the antibiotic must be maintained. b. Hold the vitamins but administer the cefazolin. c. Hold the cefazolin but administer the vitamins. d. Hold all medications since both cefazolin and vitamins are dialyzable.

ANS: D Both the cefazolin and the vitamins should be held until after the hemodialysis is completed because they would otherwise be removed by the dialysis process.

2. A nurse cares for a client with autosomal dominant polycystic kidney disease (ADPKD). The client asks, "Will my children develop this disease?" How should the nurse respond? a. "No genetic link is known, so your children are not at increased risk." b. "Your sons will develop this disease because it has a sex-linked gene." c. "Only if both you and your spouse are carriers of this disease." d. "Each of your children has a 50% risk of having ADPKD."

ANS: D Children whose parent has the autosomal dominant form of PKD have a 50% chance of inheriting the gene that causes the disease. ADPKD is transmitted as an autosomal dominant trait and therefore is not gender specific. Both parents do not need to have this disorder.

A nurse is field-triaging clients after an industrial accident. Which client condition should the nurse triage with a red tag? a. Dislocated right hip and an open fracture of the right lower leg b. Large contusion to the forehead and a bloody nose c. Closed fracture of the right clavicle and arm numbness d. Multiple fractured ribs and shortness of breath

ANS: D Clients who have an immediate threat to life are given the highest priority, are placed in the emergent or class I category, and are given a red triage tag. The client with multiple rib fractures and shortness of breath most likely has developed a pneumothorax, which may be fatal if not treated immediately. The client with the hip and leg problem and the client with the clavicle fracture would be classified as class II; these major but stable injuries can wait 30 minutes to 2 hours for definitive care. The client with facial wounds would be considered the walking wounded and classified as nonurgent.

A client with rheumatoid arthritis (RA) is on the postoperative nursing unit after having elective surgery. The client reports that one arm feels like "pins and needles" and that the neck is very painful since returning from surgery. What action by the nurse is best? a. Assist the client to change positions. b. Document the findings in the client's chart. c. Encourage range of motion of the neck. d. Notify the provider immediately.

ANS: D Clients with RA can have cervical joint involvement. This can lead to an emergent situation in which the phrenic nerve is compressed, causing respiratory insufficiency. The client can also suffer a permanent spinal cord injury. The nurse needs to notify the provider immediately. Changing positions and doing range of motion may actually worsen the situation. The nurse should document findings after notifying the provider.

A nurse teaches a client who has a history of heart failure. Which statement should the nurse include in this client's discharge teaching? a. "Avoid drinking more than 3 quarts of liquids each day." b. "Eat six small meals daily instead of three larger meals." c. "When you feel short of breath, take an additional diuretic." d. "Weigh yourself daily while wearing the same amount of clothing."

ANS: D Clients with heart failure are instructed to weigh themselves daily to detect worsening heart failure early, and thus avoid complications. Other signs of worsening heart failure include increasing dyspnea, exercise intolerance, cold symptoms, and nocturia. Fluid overload increases symptoms of heart failure. The client should be taught to eat a heart-healthy diet, balance intake and output to prevent dehydration and overload, and take medications as prescribed. The most important discharge teaching is daily weights as this provides the best data related to fluid retention.

6. A trauma nurse cares for several clients with fractures. Which client should the nurse identify as at highest risk for developing deep vein thrombosis? a. An 18-year-old male athlete with a fractured clavicle b. A 36-year old female with type 2 diabetes and fractured ribs c. A 55-year-old woman prescribed aspirin for rheumatoid arthritis d. A 74-year-old man who smokes and has a fractured pelvis

ANS: D Deep vein thrombosis (DVT) as a complication with bone fractures occurs more often when fractures are sustained in the lower extremities and the client has additional risk factors for thrombus formation. Other risk factors include obesity, smoking, oral contraceptives, previous thrombus events, advanced age, venous stasis, and heart disease. The other clients do not have risk factors for DVT.

A nurse teaches a client who is prescribed digoxin (Lanoxin) therapy. Which statement should the nurse include in this client's teaching? a. "Avoid taking aspirin or aspirin-containing products." b. "Increase your intake of foods that are high in potassium." c. "Hold this medication if your pulse rate is below 80 beats/min." d. "Do not take this medication within 1 hour of taking an antacid."

ANS: D Gastrointestinal absorption of digoxin is erratic. Many medications, especially antacids, interfere with its absorption. Clients are taught to hold their digoxin for bradycardia; a heart rate of 80 beats/min is too high for this cutoff. Potassium and aspirin have no impact on digoxin absorption, nor do these statements decrease complications of digoxin therapy.

A nurse reviews the urinalysis of a client and notes the presence of glucose. Which action should the nurse take? a. Document findings and continue to monitor the client. b. Contact the provider and recommend a 24-hour urine test. c. Review the client's recent dietary selections. d. Perform a capillary artery glucose assessment.

ANS: D Glucose normally is not found in the urine. The normal renal threshold for glucose is about 220 mg/dL, which means that a person whose blood glucose is less than 220 mg/dL will not have glucose in the urine. A positive finding for glucose on urinalysis indicates high blood sugar. The most appropriate action would be to perform a capillary artery glucose assessment. The client needs further evaluation for this abnormal result; therefore, documenting and continuing to monitor is not appropriate. Requesting a 24-hour urine test or reviewing the client's dietary selections will not assist the nurse to make a clinical decision related to this abnormality.

A client is assessed by the nurse after a hemodialysis session. The nurse notes bleeding from the client's nose and around the intravenous catheter. What action by the nurse is the priority? a. Hold pressure over the client's nose for 10 minutes. b. Take the client's pulse, blood pressure, and temperature. c. Assess for a bruit or thrill over the arteriovenous fistula. d. Prepare protamine sulfate for administration.

ANS: D Heparin is used with hemodialysis treatments. The bleeding alerts the nurse that too much anticoagulant is in the client's system and protamine sulfate should be administered. Pressure, taking vital signs, and assessing for a bruit or thrill are not as important as medication administration.

A nurse reviews the urinalysis results of a client and notes a urine osmolality of 1200 mOsm/L. Which action should the nurse take? a. Contact the provider and recommend a low-sodium diet. b. Prepare to administer an intravenous diuretic. c. Obtain a suction device and implement seizure precautions. d. Encourage the client to drink more fluids.

ANS: D Normal urine osmolality ranges from 300 to 900 mOsm/L. This client's urine is more concentrated, indicating dehydration. The nurse should encourage the client to drink more water. Dehydration can be associated with elevated serum sodium levels. Although a low-sodium diet may be appropriate for this client, this diet change will not have a significant impact on urine osmolality. A diuretic would increase urine output and decrease urine osmolality further. Low serum sodium levels, not elevated serum levels, place the client at risk for seizure activity. These options would further contribute to the client's dehydration or elevate the osmolality.

An older client has returned to the surgical unit after a total hip replacement. The client is confused and restless. What intervention by the nurse is most important to prevent injury? a. Administer mild sedation. b. Keep all four siderails up. c. Restrain the client's hands. d. Use an abduction pillow.

ANS: D Older clients often have trouble metabolizing anesthetics and pain medication, leading to confusion or restlessness postoperatively. To prevent the hip from dislocating, the nurse should use an abduction pillow since the client cannot follow directions at this time. Sedation may worsen the client's mental status and should be avoided. Using all four siderails may be considered a restraint. Hand restraints are not necessary in this situation.

3. A nurse obtains the health history of a client with a fractured femur. Which factor identified in the clients history should the nurse recognize as an aspect that may impede healing of the fracture? a. Sedentary lifestyle b. A 30pack-year smoking history c. Prescribed oral contraceptives d. Pagets disease

ANS: D Pagets disease and bone cancer can cause pathologic fractures such as a fractured femur that do not achieve total healing. The other factors do not impede healing but may cause other health risks.

A nurse is caring for a client with acute pericarditis who reports substernal precordial pain that radiates to the left side of the neck. Which nonpharmacologic comfort measure should the nurse implement? a. Apply an ice pack to the client's chest. b. Provide a neck rub, especially on the left side. c. Allow the client to lie in bed with the lights down. d. Sit the client up with a pillow to lean forward on.

ANS: D Pain from acute pericarditis may worsen when the client lays supine. The nurse should position the client in a comfortable position, which usually is upright and leaning slightly forward. Pain is decreased by using gravity to take pressure off the heart muscle. An ice pack and neck rub will not relieve this pain.

An emergency department (ED) charge nurse prepares to receive clients from a mass casualty within the community. What is the role of this nurse during the event? a. Ask ED staff to discharge clients from the medical-surgical units in order to make room for critically injured victims. b. Call additional medical-surgical and critical care nursing staff to come to the hospital to assist when victims are brought in. c. Inform the incident commander at the mass casualty scene about how many victims may be handled by the ED. d. Direct medical-surgical and critical care nurses to assist with clients currently in the ED while emergency staff prepare to receive the mass casualty victims.

ANS: D The ED charge nurse should direct additional nursing staff to help care for current ED clients while the ED staff prepares to receive mass casualty victims; however, they should not be assigned to the most critically ill or injured clients. The house supervisor and unit directors would collaborate to discharge stable clients. The hospital incident commander is responsible for mobilizing resources and would have the responsibility for calling in staff. The medical command physician would be the person best able to communicate with on-scene personnel regarding the ability to take more clients.

A client has just had a central line catheter placed that is specific for hemodialysis. What is the most appropriate action by the nurse? a. Use the catheter for the next laboratory blood draw. b. Monitor the central venous pressure through this line. c. Access the line for the next intravenous medication. d. Place a heparin or heparin/saline dwell after hemodialysis.

ANS: D The central line should have a heparin or heparin/saline dwell after hemodialysis treatment. The central line catheter used for dialysis should not be used for blood sampling, monitoring central venous pressures, or giving drugs or fluids.

A nurse cares for a client who has advanced cardiac disease and states, "I am having trouble sleeping at night." How should the nurse respond? a. "I will consult the provider to prescribe a sleep study to determine the problem." b. "You become hypoxic while sleeping; oxygen therapy via nasal cannula will help." c. "A continuous positive airway pressure, or CPAP, breathing mask will help you breathe at night." d. "Use pillows to elevate your head and chest while you are sleeping."

ANS: D The client is experiencing orthopnea (shortness of breath while lying flat). The nurse should teach the client to elevate the head and chest with pillows or sleep in a recliner. A sleep study is not necessary to diagnose this client. Oxygen and CPAP will not help a client with orthopnea.

The nurse is caring for an older patient post hip-repair surgery. What postoperative care must the nurse perform for this patient? 1 Re-position the patient every 4 hours 2 Inspect the patient's heels once a day 3 Monitor the patient frequently to prevent falls 4 Instruct the patient to maintain bedrest for a week

Answer: 3 After hip repair, the older patient is at risk to experience acute confusion and delirium. The patient must be monitored frequently to prevent falls, especially when trying to get out of bed. The patient is encouraged to begin ambulating the day after surgery to prevent complications associated with immobility. The patient must be re-positioned every 1 to 2 hours to prevent pressure ulcers. The heels of the patient must be kept off the bed at all times to prevent ulcer formation. The heels and other bony prominences should be inspected every 8 to 12 hours.

A nurse cares for a client with end-stage heart failure who is awaiting a transplant. The client appears depressed and states, "I know a transplant is my last chance, but I don't want to become a vegetable." How should the nurse respond? a. "Would you like to speak with a priest or chaplain?" b. "I will arrange for a psychiatrist to speak with you." c. "Do you want to come off the transplant list?" d. "Would you like information about advance directives?"

ANS: D The client is verbalizing a real concern or fear about negative outcomes of the surgery. This anxiety itself can have a negative effect on the outcome of the surgery because of sympathetic stimulation. The best action is to allow the client to verbalize the concern and work toward a positive outcome without making the client feel as though he or she is crazy. The client needs to feel that he or she has some control over the future. The nurse personally provides care to address the client's concerns instead of pushing the client's issues off on a chaplain or psychiatrist. The nurse should not jump to conclusions and suggest taking the client off the transplant list, which is the best treatment option.

A client with chronic kidney disease states, "I feel chained to the hemodialysis machine." What is the nurse's best response to the client's statement? a. "That feeling will gradually go away as you get used to the treatment." b. "You probably need to see a psychiatrist to see if you are depressed." c. "Do you need help from social services to discuss financial aid?" d. "Tell me more about your feelings regarding hemodialysis treatment."

ANS: D The nurse needs to explore the client's feelings in order to help the client cope and enter a phase of acceptance or resignation. It is common for clients to be discouraged because of the dependency of the treatment, especially during the first year. Referrals to a mental health provider or social services are possibilities, but only after exploring the client's feelings first. Telling the client his or her feelings will go away is dismissive of the client's concerns.

A client with acute kidney injury has a blood pressure of 76/55 mm Hg. The health care provider ordered 1000 mL of normal saline to be infused over 1 hour to maintain perfusion. The client is starting to develop shortness of breath. What is the nurse's priority action? a. Calculate the mean arterial pressure (MAP). b. Ask for insertion of a pulmonary artery catheter. c. Take the client's pulse. d. Slow down the normal saline infusion.

ANS: D The nurse should assess that the client could be developing fluid overload and respiratory distress and slow down the normal saline infusion. The calculation of the MAP also reflects perfusion. The insertion of a pulmonary artery catheter would evaluate the client's hemodynamic status, but this should not be the initial action by the nurse. Vital signs are also important after adjusting the intravenous infusion.

A nurse cares for a client who is having trouble voiding. The client states, "I cannot urinate in public places." How should the nurse respond? a. "I will turn on the faucet in the bathroom to help stimulate your urination." b. "I can recommend a prescription for a diuretic to improve your urine output." c. "I'll move you to a room with a private bathroom to increase your comfort." d. "I will close the curtain to provide you with as much privacy as possible."

ANS: D The nurse should provide privacy to clients who may be uncomfortable or have issues related to elimination or the urogenital area. Turning on the faucet and administering a diuretic will not address the client's concern. Although moving the client to a private room with a private bathroom would be nice, this is not realistic. The nurse needs to provide as much privacy as possible within the client's current room.

A nurse assesses a client after administering isosorbide mononitrate (Imdur). The client reports a headache. Which action should the nurse take? a. Initiate oxygen therapy. b. Hold the next dose of Imdur. c. Instruct the client to drink water. d. Administer PRN acetaminophen.

ANS: D The vasodilating effects of isosorbide mononitrate frequently cause clients to have headaches during the initial period of therapy. Clients should be told about this side effect and encouraged to take the medication with food. Some clients obtain relief with mild analgesics, such as acetaminophen. The client's headache is not related to hypoxia or dehydration; therefore, these interventions would not help. The client needs to take the medication as prescribed to prevent angina; the medication should not be held.

3. A nurse receives a report on a client who had a left-sided stroke and has homonymous hemianopsia. What action by the nurse is most appropriate for this client? a. Assess for bladder retention and/or incontinence. b. Listen to the clients lungs after eating or drinking. c. Prop the clients right side up when sitting in a chair. d. Rotate the clients meal tray when the client stops eating.

ANS: D This condition is blindness on the same side of both eyes. The client must turn his or her head to see the entire visual field. The client may not see all the food on the tray, so the nurse rotates it so uneaten food is now within the visual field. This condition is not related to bladder function, difficulty swallowing, or lack of trunk control.

9. A nurse prepares to defibrillate a client who is in ventricular fibrillation. Which priority intervention should the nurse perform prior to defibrillating this client? a. Make sure the defibrillator is set to the synchronous mode. b. Administer 1 mg of intravenous epinephrine. c. Test the equipment by delivering a smaller shock at 100 joules. d. Ensure that everyone is clear of contact with the client and the bed.

ANS: D To avoid injury, the rescuer commands that all personnel clear contact with the client or the bed and ensures their compliance before delivery of the shock. A precordial thump can be delivered when no defibrillator is available. Defibrillation is done in asynchronous mode. Equipment should not be tested before a client is defibrillated because this is an emergency procedure; equipment should be checked on a routine basis. Epinephrine should be administered after defibrillation.

The nurse admits an older adult patient who sustained a left hip fracture and is in considerable pain. The nurse anticipates that the patient will be placed in which type of traction? 1 Buck's traction 2 Plaster traction 3 Overhead traction 4 Balanced skin traction

Answer: 1 Buck's traction Buck's traction may be applied before surgery to help decrease pain associated with muscle spasm. Balanced skin traction is indicated for fracture of the femur or pelvis. Overhead traction is indicated for fracture of the humerus with or without involvement of the shoulder and clavicle. Plaster traction is indicated for wrist fracture.

The nurse prepares to perform a neurovascular assessment on a patient with closed multiple fractures of the right humerus. Which technique does the nurse use? 1 Auscultate lung sounds 2 Assess sensation of the right upper extremity 3 Inspect the abdomen for tenderness and bowel sounds 4 Assess the level of consciousness and ability to follow commands

Answer: 2 Assessing sensation of the right upper extremity is part of a focused neurovascular assessment for the patient with multiple fractures of the right humerus. Inspecting the abdomen and auscultating lung sounds of the patient with multiple fractures are not part of a focused neurovascular assessment. Because the patient does not have a head injury, assessing the patient's level of consciousness and ability to follow commands is not part of a focused neurovascular assessment.

Which intervention is most appropriate for the patient who was prescribed hydroxychloroquine and developed retinal damage? 1 Decreasing the dose of the drug 2 Discontinuing the drug completely 3 Discontinuing the drug for a few days 4 Continuing the treatment with no change

Answer: 2 Retinal damage is a serious and rare complication of hydroxychloroquine use; therefore, the drug should be completely discontinued. Decreasing the dose of the drug might not help in reducing or preventing further retinal damage.

The nurse provides discharge planning to a patient who has been prescribed methotrexate (MTX). Which vitamin is included to help decrease some of the side effects of MTX? 1 Niacin 2 Folic acid 3 Vitamin D 4 Vitamin K

Answer: 2 Folic Acid Folic acid, one of the B vitamins, is often given to patients who are taking MTX to help decrease some of the drug's side effects including GI problems, such as nausea and vomiting, and oral problems, such as mouth sores. Niacin and vitamins D and K do not have any significant impact on the side effects of methotrexate.

Which drug is contraindicated for a patient with rheumatoid arthritis and multiple sclerosis? 1 Celecoxib 2 Infliximab 3 Methotrexate 4 Hydroxychloroquine

Answer: 2 Infliximab Remind patients with multiple sclerosis (MS), tuberculosis (TB), or a positive TB test that they should not receive TNF inhibitors because they make them more susceptible to flare-ups of these diseases. Infliximab is a tumor necrosis factor—alpha (TNFA) inhibitor, which may be used for rheumatoid arthritis. All TNFA inhibitor drugs should be avoided by a patient with multiple sclerosis.

A patient has symptoms of rheumatoid arthritis (RA). Which laboratory finding indicates to the nurse that the patient may have RA? 1 Total serum complement, 75 units/mL 2 Positive total antinuclear antibody (ANA) 3 Erythrocyte sedimentation rate (ESR), 20 mm/hr 4 Beta-globulin level, 1.0 g/dL

Answer: 2 Positive total antinuclear antibody (ANA) Elevation of total ANA is common in systemic lupus erythematosus (SLE), systemic sclerosis (SSc), and RA.

The nurse is teaching safety measures to a patient with rheumatoid arthritis who is on methotrexate therapy. Which statements should the nurse include? Select all that apply. 1 "You should eat a calcium-rich diet." 2 "You should avoid crowded places." 3 "You should avoid alcoholic beverages." 4 "You should take folic acid supplements." 5 "You should have your eyes examined for every 6 months."

Answer: 2, 3, 4 Patients taking methotrexate may experience folic acid deficiency so folic acid supplementation is recommended. A patient on methotrexate may have immunosuppression and be more prone to infections. Therefore, the patient should avoid crowded places. Patients on methotrexate should not drink alcohol in order to prevent liver toxicity.

The nurse is caring for a female patient diagnosed with rheumatoid arthritis who is prescribed methotrexate. Which instruction should the nurse provide for safe and effective care? 1 "You should report hair loss." 2 "You should report chest pain." 3 "You should strictly use birth control." 4 "You should discontinue the therapy if the symptoms are suppressed."

Answer: 3 Methotrexate therapy may cause birth defects. Therefore, a woman of child-bearing age prescribed methotrexate for rheumatoid arthritis should strictly use birth control.

To which resource should the nurse refer the patient to for upper extremity rehabilitation after trauma? 1 Physical therapy 2 Cognitive retraining 3 Occupational therapy 4 Behavior modification

Answer: 3 Occupational therapy is typically the best form of rehabilitation for trauma of the upper extremities, while physical therapy is more appropriate for lower-extremity rehabilitation. Cognitive retraining and behavior modification are not indicated for this type of trauma.

Which type of trauma increases a patient's risk for hospital-acquired infection? 1 Malunion 2 Osteonecrosis 3 Open fractures 4 Compartment syndrome

Answer: 3 Open fractures increase the risk of osteomyelitis, which also increases the risk for hospital-acquired infections. Malunion is incorrect bone healing. Osteonecrosis and compartment syndrome are complications of closed fractures.

What may be the medications provided to prevent osteoporosis in the patient with rheumatoid arthritis who is prescribed long-term steroid therapy? 1 Calcium 750 mg and vitamin D 100 mg daily 2 Calcium 1000 mg and vitamin D 200 mg daily 3 Calcium 1500 mg and vitamin D 400 mg daily 4 Calcium 1800 mg and vitamin D 500 mg daily

Answer: 3 Osteoporosis is a complication of glucocorticoid therapy; therefore, adequate calcium and vitamin D should be taken to prevent osteoporosis. Calcium 1500 mg and vitamin D 400 mg are recommended for patients receiving glucocorticoid therapy.

Which type of fracture does not divide the bone into two portions because the break is through only part of the bone? 1 Fragility 2 Complete 3 Incomplete 4 Compression

Answer: 3 Incomplete An incomplete fracture does not divide the bone into two portions; it is only broken through a part of the bone. A fragility fracture occurs after minimal trauma to a bone that has been weakened by disease. A complete fracture refers to a break that is across the entire width of the bone in such a way that the bone is divided into two distinct sections. Compression fractures are produced by a loading force applied to the long axis of cancellous bone.

The nurse is reviewing the medication history for a patient diagnosed with rheumatoid arthritis (RA) who has been ordered to start sulfasalazine therapy. The nurse plans to contact the health care provider if the patient has which condition? 1 Glaucoma 2 Hypertension 3 Hypothyroidism 4 Sulfa allergy

Answer: 4

The nurse is caring for a patient with a tibial fracture. For which complication common in a fracture of this type should the nurse monitor the patient? 1 Fat embolism 2 Crush syndrome 3 Hypovolemic shock 4 Acute compartment syndrome (ACS)

Answer: 4 ACS is a complication typically seen in lower limb fractures, such as those of the tibia. The nurse should monitor for circulation and infection, contracture, and persistent motor weakness, which are associated with ACS. Crush syndrome is typically seen with crushing injuries. Fat embolism and hypovolemic shock are not specific to the lower limbs.

When differentiating between fat embolism and blood clot embolism, which statement is true? 1 Blood clot embolism usually results within 48 hours of injury. 2 Blood clot embolism is more likely to result in the development of petechiae. 3 Fat embolism is more likely to result in the need for a pulmonary embolectomy. 4 Fat embolism is more likely to occur with fractures of the long bones.

Answer: 4 Fat embolism is more likely to occur with fractures of the long bones. Blood clot embolism can occur anytime, but fat embolism usually occurs within 48 hours of injury. Fat embolism and blood clot embolism have many identical assessment findings, except fat embolism results in petechiae and blood clot embolism does not. Blood clot embolism can necessitate surgery, including pulmonary embolectomy or vena cava umbrella, while fat embolism generally does not require surgical intervention.

An older patient with rheumatoid arthritis reports a burning and tingling sensation in the legs. Further assessment reveals foot drop in the patient. Which condition does the nurse suspect as a clinical manifestation of vasculitis present in rheumatoid arthritis? 1 Quadriplegia 2 Osteoporosis 3 Caplan's syndrome 4 Peripheral neuropathy

Answer: 4 Peripheral neuropathy associated with decreased blood circulation may cause foot drop and paresthesias in patients with rheumatoid arthritis. Quadriplegia is paralysis and may occur in patients with rheumatoid arthritis who are also suffering from cervical disease.

A patient is brought to the emergency department (ED) via ambulance after a motor vehicle crash. What condition does the nurse assess for first? 1 Pain 2 Bleeding 3 Head injury 4 Respiratory distress

Answer: 4 The patient should first be assessed for respiratory distress, and any oxygen interventions instituted accordingly. Bleeding is the second assessment priority, head injury is the third assessment priority, and pain is the fourth assessment priority in this case.

Which biologic is approved for administration once a month for patients with rheumatoid arthritis and psoriatic arthritis? 1 Infliximab 2 Abatacept 3 Etanercept 4 Golimumab

Answer: 4 Golimumab Golimumab is the biologic administered ONLY ONCE PER MONTH for patients with rheumatoid arthritis OR psoriatic arthritis. A dosage of infliximab is repeated every 2nd and 6th week after the initial infusion. Abatacept requires IV infusions every 2 weeks. Etanercept is given subcutaneously twice weekly in most patients with rheumatoid arthritis.

A patient with ankylosing spondylitis is being treated with a first-line disease-modifying antirheumatic drug (DMARD). Which DMARD drug does the nurse anticipate will be prescribed for this patient? 1 Abatacept 2 Infliximab 3 Tocilizumab 4 Methotrexate

Answer: 4 Methotrexate Methotrexate is a first-line disease-modifying antirheumatic drug, which is an immunosuppressive medication. Abatacept, infliximab, and tocilizumab are biological response modifiers (BRMs) (also called biologics)

Which assessment will the nurse perform to detect side effects after administering the drug golimumab? 1 Blood pressure 2 Pedal pulses 3 Serum electrolytes 4 Temperature

Answer: 4 Temperature Golimumab is the first biologic that is administered only once each month for psoriatic arthritis, which can cause opportunistic pathogens. Therefore, the temperature is taken because if it is elevated, it indicates the person has a fever, which is an early sign of infection.

What nonpharmacologic comfort measures should the nurse include in the plan for a client with severe varicose veins? a. Administering mild analgesics for pain b. Applying elastic compression stockings c. Elevating the legs when sitting or lying d. Reminding the client to do leg exercises e. Teaching the client about surgical options

Applying elastic compression stockings Elevating the legs when sitting or lying Reminding the client to do leg exercises ANS: B, C, D The three E's of care for varicose veins include elastic compression hose, exercise, and elevation. Mild analgesics are not a nonpharmacologic measure. Teaching about surgical options is not a comfort measure

A client had a femoropopliteal bypass graft with a synthetic graft. What action by the nurse is most important to prevent wound infection? a. Appropriate hand hygiene before giving care b. Assessing the client's temperature every 4 hours c. Clean technique when changing dressings d. Monitoring the client's daily white blood cell count

Appropriate hand hygiene before giving care Hand hygiene is the best way to prevent infections in hospitalized clients. Dressing changes should be done with sterile technique. Assessing vital signs and white blood cell count will not prevent infection.

The nurse is working with a client who takes atorvastatin ( Lipitor ). The client's recent laboratory results include a blood urea nitrogen ( BUN ) of 33 mg/dL and creatinine of 2.8 mg/dL. What action by the nurse is best? a. Ask if the client eats grapefruit. b. Assess the client for dehydration. c. Facilitate admission to the hospital. d. Obtain a random urinalysis.

Ask if the client eats grapefruit There is a drug-food interaction between statins and grapefruit that can lead to acute kidney failure. This client has elevated renal laboratory results, indicating some degree of kidney involvement. The nurse should assess if the client eats grapefruit or drinks grapefruit juice.

A nurse is caring for a client on IV infusion of heparin. What action does this nurse include in the client's plan of care? a. Assess the client for bleeding. b. Monitor the daily activated partial thromboplastin time (aPTT) results. c. Stop the IV for aPTT above baseline. d. Use an IV pump for the infusion. e. Weigh the client daily on the same scale.

Assess the client for bleeding Monitor the daily activated partial thromboplastin time ( aPTT ) results Use an IV pump for the infusion Assessing for bleeding, monitoring aPTT, and using an IV pump for the infusion are all important safety measures for heparin to prevent injury from bleeding. The aPTT needs to be 1.5 to 2 times normal in order to demonstrate that the heparin is therapeutic. Weighing the client is not related.

The nurse is assessing a client on admission to the hospital. The client's leg appears as shown below: What action by the nurse is best? a. Assess the client's ankle-brachial index. b. Elevate the client's leg above the heart. c. Obtain an ice pack to provide comfort. d. Prepare to teach about heparin sodium.

Assess the client's ankle-brachial index This client has dependent rubor, a classic finding in peripheral arterial disease. The nurse should measure the client's ankle-brachial index. Elevating the leg above the heart will further impede arterial blood flow. Ice will cause vasoconstriction, also impeding circulation and perhaps causing tissue injury. Heparin sodium is not the drug of choice for this condition.

A client with a history of heart failure and hypertension is in the clinic for a follow up visit. The client is on Lisinopril ( Prinvil ) and warfarin ( Coumadin ). The client reports new onset cough. What action by the nurse is most appropriate? a. Assess the client's lung sounds and oxygenation. b. Instruct the client on another antihypertensive. c. Obtain a set of vital signs and document them. d. Remind the client that cough is a side effect of Prinivil.

Assess the client's lung sounds and oxygenation ANS: A This client could be having an exacerbation of heart failure or be experiencing a side effect of lisinopril (and other angiotensin-converting enzyme inhibitors). The nurse should assess the client's lung sounds and other signs of oxygenation first. The client may or may not need to switch antihypertensive medications. Vital signs and documentation are important, but the nurse should assess the respiratory system first. If the cough turns out to be a side effect, reminding the client is appropriate, but then more action needs to be taken.

A client has been diagnosed with a deep vein thrombosis and is to be discharged on warfarin ( Comadin ). The client is adamant about refusing the drug because " it's dangerous ". What action by the nurse is best? aa. Assess the reason behind the client's fear. b. Remind the client about laboratory monitoring. c. Tell the client drugs are safer today than before. d. Warn the client about consequences of noncompliance.

Assess the reason behind the client's fear

A nurse cares for a client with hepatitis C. The clients brother states, I do not want to contract this infection, so I will not go into his hospital room. How should the nurse respond? a. If you wear a gown and gloves, you will not get this virus. b. Viral hepatitis is not spread through casual contact. c. This virus is only transmitted through a fecal specimen. d. I can give you an update on your brothers status from here.

B

The nursing assistant reports that the client with metabolic acidosis due to kidney failure is breathing rapidly and deeply. The nurse explains this to the nursing assistant in which of these manners? A. "The client is acting out and we should pay him no mind" B. "Rapid breathing is a way to compensate for acidosis caused by his condition" C. "Normally a client with this disorder will breathe slowly, I will go assess him" D. "Deep breathing is a symptom of diabetes, I will check his blood glucose"

B (The nurse explains that kussmaul or rapid and deep breathing helps the body compensate for metabolic acidosis by blowing off the CO2 or respiratory acid through the lungs. This will also increase the body's pH level.The client would not be judged for acting out without a clear understanding of the underlying client's cause. Slow respirations are not consistent with metabolic acidosis, however, may cause respiratory acidosis. Deep breathing (Kussmaul breathing) is a compensatory mechanism for metabolic acidosis ocurring with DKA or kidney disease.)

A nurse is planning care for a client who is hyperventilating. The client's arterial blood gas values are pH 7.30, PaO2 94 mm Hg, PaCO2 31 mm Hg, and HCO3- 26 mEq/L. Which question should the nurse ask when developing this client's plan of care? a. "Do you take any over-the-counter medications?" b. "You appear anxious. What is causing your distress?" c. "Do you have a history of anxiety attacks?" d. "You are breathing fast. Is this causing you to feel light-headed?

B (The nurse should assist the client who is experiencing anxiety-induced respiratory alkalosis to identify causes of the anxiety. The other questions will not identify the cause of the acid-base imbalance.)

A nurse is planning care for a client who is anxious and irritable. The client's arterial blood gas values are pH 7.30, PaO2 96 mm Hg, PaCO2 43 mm Hg, and HCO3- 19 mEq/L. Which questions should the nurse ask the client and spouse when developing the plan of care? (Select all that apply.) a. "Are you taking any antacid medications?" b. "Is your spouse's current behavior typical?" c. "Do you drink any alcoholic beverages?" d. "Have you been experiencing any vomiting?" e. "Are you experiencing any shortness of breath?"

B, C (This client's symptoms of anxiety and irritability are related to a state of metabolic acidosis. The nurse should ask the client's spouse or family members if the client's behavior is typical for him or her, and establish a baseline for comparison with later assessment findings. The nurse should also assess for alcohol intake because alcohol can change a client's personality and cause metabolic acidosis. The other options are not causes of metabolic acidosis.)

The nurse and nursing student are caring for a client with a new diagnosis of diabetes whose blood glucose is 974 mg/dL (54.1 mmol/L). Which of these statements indicates the student understands the relationship between blood glucose and acid base balance? A."The excess glucose in the blood causes the client to hypoventilate and retain carbon dioxide resulting in respiratory acidosis" B."The hyperglycemia is caused by inability of glucose to enter the cell causing a starvation state and break down of fats" C. "The client has a hyperosmolar condition causing polyuria and polyphagia, but the acid base balance is normal" D. "The client is retaining carbon dioxide which led to respiratory acidosis and somnolence"

B (The nursing student understands the relationship between blood glucose and acid base balance when the student states that hyperglycemia is caused by inability of glucose to enter the cell causing a starvation state and break down of fats. Glucose cannot enter the cell to provide energy without the presence of insulin. The body begins to break down fat for energy which produces ketones and causes ketoacidosis.The client with ketoacidosis will hyperventilate, breathing more rapidly and deeply to rid the body of respiratory acids such as CO2. This process buffers the acidosis. A hyperosmolar state does occur, however the acid base balance is still affected. CO2 is retained when the client's inability to ventilate or remove CO2 effectively occurs. Hypercarbia, CO2 retention, is generally caused by problems affecting the pulmonary system.)

A nurse is assessing a client who has acute pancreatitis and is at risk for an acid-base imbalance. For which manifestation of this acid-base imbalance should the nurse assess? a. Agitation b. Kussmaul respirations c. Seizures d. Positive Chvostek's sign

B (The pancreas is a major site of bicarbonate production. Pancreatitis can cause a relative metabolic acidosis through underproduction of bicarbonate ions. Manifestations of acidosis include lethargy and Kussmaul respirations. Agitation, seizures, and a positive Chvostek's sign are manifestations of the electrolyte imbalances that accompany alkalosis.)

A nurse is caring for a client who is experiencing excessive diarrhea. The client's arterial blood gas values are pH 7.28, PaO2 98 mm Hg, PaCO2 45 mm Hg, and HCO3- 16 mEq/L. Which provider order should the nurse expect to receive? a. Furosemide (Lasix) 40 mg intravenous push b. Sodium bicarbonate 100 mEq diluted in 1 L of D5W c. Mechanical ventilation d. Indwelling urinary catheter

B (This client's arterial blood gas values represent metabolic acidosis related to a loss of bicarbonate ions from diarrhea. The bicarbonate should be replaced to help restore this client's acid-base balance. Furosemide would cause an increase in acid fluid and acid elimination via the urinary tract; although this may improve the client's pH, the client has excessive diarrhea and cannot afford to lose more fluid. Mechanical ventilation is used to treat respiratory acidosis for clients who cannot keep their oxygen saturation at 90%, or who have respirator muscle fatigue. Mechanical ventilation and an indwelling urinary catheter would not be prescribed for this client.)

A nurse is caring for an older adult client who has a pulmonary infection. Which action should the nurse take first? Encourage the client to increase fluid intake b. Assess the clients level of consciousness. c. Raise the head of the bed to at least 45 degrees d. Provide the client with humidified oxygen.

B Assessing the clients level of consciousness will be most important because it will show how the client is responding to the presence of the infection. Although it will be important for the nurse to encourage the client to turn, cough, and frequently breathe deeply; raise the head of the bed; increase oral fluid intake; and humidify the oxygen administered, none of these actions will be as important as assessing the level of consciousness. Also, the client who has a pulmonary infection may not be able to cough effectively if an area of abscess is present.

A nurse is assessing clients who are at risk for acid-base imbalance. Which clients are correctly paired with the acid-base imbalance? (Select all that apply.) a. Metabolic alkalosis - Young adult who is prescribed intravenous morphine sulfate for pain b. Metabolic acidosis - Older adult who is following a carbohydrate-free diet c. Respiratory alkalosis - Client on mechanical ventilation at a rate of 28 breaths/min d. Respiratory acidosis - Postoperative client who received 6 units of packed red blood cells e. Metabolic alkalosis - Older client prescribed antacids for gastroesophageal reflux disease

B, C, E (Respiratory acidosis often occurs as the result of underventilation. The client who is taking opioids, especially IV opioids, is at risk for respiratory depression and respiratory acidosis. One cause of metabolic acidosis is a strict low-calorie diet or one that is low in carbohydrate content. Such a diet increases the rate of fat catabolism and results in the formation of excessive ketoacids. A ventilator set at a high respiratory rate or tidal volume will cause the client to lose too much carbon dioxide, leading to an acid deficit and respiratory alkalosis. Citrate is a substance used as a preservative in blood products. It is not only a base, it is also a precursor for bicarbonate. Multiple units of packed red blood cells could cause metabolic alkalosis. Sodium bicarbonate antacids may increase the risk of metabolic alkalosis.)

The nurse is evaluating a 3-day diet history with a client who has an elevated lipid panel. What meal selection indicates the client is managing this condition well with diet? a. A 4-ounce steak, French fries, iceberg lettuce b. Baked chicken breast, broccoli, tomatoes c. Fried catfish, cornbread, peas d. Spaghetti with meat sauce, garlic bread

Baked chicken breast, broccoli, tomatoes The diet recommended for this client would be low in saturated fats and red meat, high in vegetables and whole grains (fiber), low in salt, and low in trans fat. The best choice is the chicken with broccoli and tomatoes. The French fries have too much fat and the iceberg lettuce has little fiber. The catfish is fried. The spaghetti dinner has too much red meat and no vegetables.

A nurse teaches a client who is prescribed nicotine replacement therapy. Which statement should the nurse include in this clients teaching? a. Make a list of reasons why smoking is a bad habit. b. Rise slowly when getting out of bed in the morning. c. Smoking while taking this medication will increase your risk of a stroke. d. Stopping this medication suddenly increases your risk for a heart attack.

C Clients who smoke while using drugs for nicotine replacement therapy increase the risk of stroke and heart attack. Nurses should teach clients not to smoke while taking this drug. The other responses are inappropriate.

A nurse obtains the health history of a client who is recently diagnosed with lung cancer and identifies that the client has a 60 pack-year smoking history. Which action is most important for the nurse to take when interviewing this client? a. Tell the client that he needs to quit smoking to stop further cancer development. b. Encourage the client to be completely honest about both tobacco and marijuana use. c. Maintain a nonjudgmental attitude to avoid causing the client to feel guilty. d. Avoid giving the client false hope regarding cancer treatment and prognosis.

C Smoking history includes the use of cigarettes, cigars, pipe tobacco, marijuana, and other controlled substances. Because the client may have guilt or denial about this habit, assume a nonjudgmental attitude during the interview. This will encourage the client to be honest about the exposure. Ask the client whether any of these substances are used now or were used in the past. Assess whether the client has passive exposure to smoke in the home or workplace. If the client smokes, ask for how long, how many packs per day, and whether he or she has quit smoking (and how long ago). Document the smoking history in pack-years (number of packs smoked daily multiplied by the number of years the client has smoked). Quitting smoking may not stop further cancer development. This statement would be giving the client false hope, which should be avoided, but is not as important as maintaining a nonjudgmental attitude.

A nurse cares for a client who had a bronchoscopy 2 hours ago. The client asks for a drink of water. Which action should the nurse take next? a. Call the physician and request a prescription for food and water. b. Provide the client with ice chips instead of a drink of water c. Assess the clients gag reflex before giving any food or water d. Let the client have a small sip to see whether he or she can swallow

C The topical anesthetic used during the procedure will have affected the clients gag reflex. Before allowing the client anything to eat or drink, the nurse must check for the return of this reflex.

A nurse is providing care after auscultating clients breath sounds. Which assessment finding is correctly matched to the nurses primary intervention? a. Hollow sounds are heard over the trachea. The nurse increases the oxygen flow rate. b. Crackles are heard in bases. The nurse c. Wheezes are heard in central areas. The nurse administers an inhaled bronchodilator. d. Vesicular sounds are heard over the periphery. administers an inhaled bronchodilator. The nurse has the client breathe deeply.

C Wheezes are indicative of narrowed airways, and bronchodilators help to open the air passages. Hollow sounds are typically heard over the trachea, and no intervention is necessary. If crackles are heard, the client may need a diuretic. Crackles represent a deep interstitial process, and coughing forcefully will not help the client expectorate secretions. Vesicular sounds heard in the periphery are normal and require no intervention.

Which finding in the physical assessment may be associated with osteoarthritis? 1 Depression 2 Impaired mobility 3 Altered body image 4 Decreased quality of life

Correct = 2 Impaired mobility Impaired mobility is a clinical manifestation in a patient with osteoarthritis. It occurs due to pain and inflammation in the joints. Depression, altered body image, and decreased quality of life are psychosocial changes associated with osteoarthritis.

A nurse is caring for four clients. Which one should the nurse see first? a. Client who needs a beta blocker, and has a blood pressure of 92/58 mm Hg b. Client who had a first dose of captopril (Capoten) and needs to use the bathroom c. Hypertensive client with a blood pressure of 188/92 mm Hg d. Client who needs pain medication prior to a dressing change of a surgical wound

Client who had a first does of captopril ( Capoten ) and needs to use the bathroom Angiotensin-converting enzyme inhibitors such as captopril can cause hypotension, especially after the first dose. The nurse should see this client first to prevent falling if the client decides to get up without assistance. The two blood pressure readings are abnormal but not critical. The nurse should check on the client with higher blood pressure next to assess for problems related to the reading. The nurse can administer the beta blocker as standards state to hold it if the systolic blood pressure is below 90 mm Hg. The client who needs pain medication prior to the dressing change is not a priority over client safety and assisting the other client to the bathroom.

A nurse is caring for a client with a nonhealing arterial lower leg ulcer. What action by the nurse is best? a. Consult with the Wound Ostomy Care Nurse. b. Give pain medication prior to dressing changes. c. Maintain sterile technique for dressing changes. d. Prepare the client for eventual amputation.

Consult with the Wound Ostomy Care Nurse A nonhealing wound needs the expertise of the Wound Ostomy Care Nurse (or Wound Ostomy Continence Nurse). Premedicating prior to painful procedures and maintaining sterile technique are helpful, but if the wound is not healing, more needs to be done. The client may need an amputation, but other options need to be tried first.

What are the factors that may lead to osteoarthritis (OA)? Select all that apply. 1 Aging 2 Obesity 3 Genetics 4 Rheumatoid arthritis (RA) 5 Systemic lupus erythematosus (SLE

Correct = 1 2 3 OA is a common connective tissue disease characterized by progressive degradation of the cartilage in movable joints. Aging, obesity, and genetic factors are responsible for OA. Aging contributes to a decrease in bone density. Obesity can increase pressure on the joints. Recent studies support genetic changes may contribute to cartilage destruction, osteophyte formation, or the inability of cartilage to repair itself.

What is the cause of primary osteoarthritis? 1 Aging 2 Obesity 3 Joint injury 4 Heavy manual work

Correct = 1 Aging Primary osteoarthritis is caused by aging and genetic factors. As people age, proteoglycans, synovial fluid, and water decrease in the joint. Enzymes in the joint break down the articular matrix and the cartilage erodes. Eventually, inflammation will continue to cause deterioration in the joint space. Obesity, joint injury, and heavy manual work are causes of secondary osteoarthritis.

Which drug may be administered to a patient undergoing knee arthroplasty to reduce postoperative pain? 1 Capsaicin 2 Cefazolin 3 Dalteparin 4 Enoxaparin

Correct = 1 Capsaicin Capsaicin is a product used for reducing postoperative pain following total knee arthroplasty. It binds to C-fiber receptors and facilitates the release of extra calcium to enter the nerve cells.

A patient who is undergoing treatment for osteoarthritis reports black-colored stools, stomach upset, and difficulty breathing. The patient's blood work reveals abnormal levels of urea and creatinine. The nurse identifies that which medication is the likely cause of the patient's symptoms? 1 Ibuprofen 2 Lidocaine 3 Glucosamine 4 Cyclobenzaprine

Correct = 1 Ibuprofen The side effects associated with ibuprofen, which is a nonsteroidal anti-inflammatory medication, include dark, tarry (black-colored) stools indicative of gastrointestinal bleeding, indigestion that may result in stomach upset, and renal failure associated with abnormal levels of urea and creatinine in the blood.

Which drug may help slow the progression of systemic sclerosis? 1 Imatinib 2 Bosentan 3 Abatacept 4 Methotrexate

Correct = 1 Imatinib Imatinib is an oral tyrosine kinase inhibitor used to treat systemic sclerosis. It helps in reducing inflammation and slows the progression of the disease.

After assessing a patient with osteoarthritis, the nurse suspects crepitus. Which finding supports the nurse's assessment? 1 Inflammation near the joint 2 Grating heard when the joint is palpated 3 Enlarged joint with bone hypertrophy 4 Bony nodules at the distal interphalangeal joint

Correct = 2 Grating heard when the joint is palpated Crepitus is a condition in which the cartilage disintegrates and pieces of bone and cartilage float in the diseased joint. A grating sound can be heard because of loosened bone and cartilage in the joint.

Before administering low-molecular-weight heparin (LMWH) to an older adult patient after total knee arthroplasty, the nurse notes that the patient's platelet count is 50,000/mm 3. What action is most important for the nurse to take? 1 Notify the health care provider of the platelet count. 2 Administer the prescribed LMWH on schedule. 3 Assess the activated partial thromboplastin time (aPTT). 4 Assess the International Normalized Ratio (INR).

Correct = 1 Notify the health care provider of the platelet count. If the platelet count falls below 20,000/mm 3, spontaneous bleeding could occur. Notifying the health care provider before the LMWH is given is essential. LMWH can cause thrombocytopenia, so it should not be administered when the patient's platelet count is low.

What is a cardiovascular manifestation of systemic lupus erythematosus (SLE)? Pericarditis Pleural effusion Interstitial fibrosis Myocardial fibrosis

Correct = 1 Pericarditis Pericarditis is a cardiovascular manifestation of systemic lupus erythematosus (SLE).

What teaching should the nurse provide to the older adult patient with osteoarthritis who is prescribed cyclobenzaprine hydrochloride? Select all that apply. 1 "Avoid driving." 2 "Avoid alcohol intake." 3 "Do not operate heavy machinery." 4 "Do not stand for a prolonged time." 5 "Take the drug before breakfast in the morning.

Correct = 1, 2, 3 Cyclobenzaprine hydrochloride is a skeletal muscle relaxant used to treat painful muscle spasms. The drug may also cause acute confusion. Patients taking this drug should not drive or operate heavy machinery. Alcohol use with cyclobenzaprine hydrochloride, a CNS depressant, may impair physical or mental abilities. Standing for a prolonged time might increase pain in patients with osteoarthritis, but this is not a teaching related to the cyclobenzaprine hydrochloride prescription. The drug is taken two or three times daily after food.

Which laboratory and diagnostic tests are used to detect osteoarthritis (OA)? Select all that apply. 1 C-reactive protein assay 2 X-ray studies 3 Rheumatoid factor assay 4 Antinuclear antibody test 5 Magnetic resonance imaging

Correct = 1, 2, 5 Assessment for OA includes serological and imaging studies. The high-sensitivity C-reactive protein (hsCRP) may be slightly elevated when synovial inflammation occurs. Imaging studies of OA include routine x-ray images and magnetic resonance imaging (MRI) to determine structural joint changes.

Which statements may be used to describe osteoarthritis (OA)? Select all that apply. 1 Obesity is a risk factor. 2 It is an autoimmune disorder. 3 Its disease process is degenerative. 4 It usually affects the upper extremities first. 5 Its disease pattern is bilateral and symmetric.

Correct = 1, 3 Obesity is a risk factor and Its disease process is degenerative.

What nursing interventions may be performed to prevent dislocation of the hip joint in a patient who has undergone total hip arthroplasty (THA)? Select all that apply. 1 Avoid hip flexion beyond 90 degrees. 2 Use aseptic technique for wound care. 3 Instruct the patient to do leg exercises. 4 Instruct the patient to wear elastic stockings. 5 Assess acute pain, rotation, and extremity shortening.

Correct = 1, 5 Hip flexion beyond 90 degrees should be avoided because it and may lead to dislocation and also causes pain in the hip. Acute pain, rotation, and extremity shortening should be assessed after a THA. Using aseptic techniques for wound care would help in preventing infections. Instructing the patient to do leg exercises or to wear elastic stockings would help to prevent venous thromboembolism.

The nurse is explaining the clinical manifestations of joint involvement of rheumatoid arthritis with a nursing student. What is the best way to describe a Baker's cyst? 1 Enlarged liver and spleen 2 Enlarged popliteal bursae behind the knee 3 Presence of rheumatoid nodules in the lungs 4 Presence of small, brownish spots around the nail bed

Correct = 2 Baker's cysts are enlarged popliteal bursae behind the knee. They may occur in patients with rheumatoid arthritis and can cause tissue compression and pain.

The nurse is assisting a patient who had a right total hip arthroplasty with getting up from bed. What action does the nurse take? 1 Stands on the left side of the bed when assisting patient 2 Reminds the patient to stand on the left leg after getting out of bed 3 Lifts the patient and assists the patient to stand 4 Asks the patient to pivot on the right side to sit in the chair

Correct = 2 Reminds the patient to stand on the left leg after getting out of bed The nurse should remind the patient to stand on the left leg after getting out of bed, which prevents the patient from putting undue pressure on the right leg. The nurse should stand on the same side of the bed as the patient's affected side to provide support. The nurse should not lift the patient and assist the patient to stand. The patient should pivot on the stronger left side to sit in the chair.

The student nurse is preparing a patient with osteoarthritis for home care self-management. Which nursing instruction needs correction? 1 "Do not bend at your waist." 2 "Turn doorknobs clockwise." 3 "Use a small pillow under your head." 4 "Use utensils with an extended handle for eating."

Correct = 2 "Turn doorknobs clockwise." Patients with osteoarthritis should turn doorknobs counterclockwise to avoid twisting the arm, which promotes ulnar deviation. Patients should bend at the knees and not the waist because the back should be kept straight. A small pillow should be used under the head to support the neck. Utensils with an extended handle should be used to assist with eating.

Which drugs need to be discontinued before surgery to decrease the risk for venous thromboembolism? Select all that apply. 1 Vitamin C 2 Vitamin K 3 Antianginal drugs 4 Oral contraceptive drugs 5 Nonsteroidal anti-inflammatory drugs

Correct = 2 4 5 Vitamin C, oral contraceptive drugs, and nonsteroidal anti-inflammatory drugs increase the risk of bleeding and clotting, which results in venous thromboembolism. Therefore, these drugs should be discontinued about a week before surgery.

Which factor indicates to the nurse the only similarity between discoid lupus erythematosus (DLE) and systemic lupus erythematosus (SLE)? 1 Feeling tired and having a temperature that runs about 100° F (37.8° C) during the day 2 Disfiguring and embarrassing rash 3 Peripheral neuropathies and cranial nerve palsies 4 High risk for renal inflammation

Correct = 2 Disfiguring and embarrassing rash Skin lesions are common to SLE and DLE. Fatigue and fever are common only to SLE. Neurologic manifestations and inflammation of the kidneys are common in SLE.

What physical assessment findings does the nurse anticipate for a patient diagnosed with systemic lupus erythematosus (SLE)? Select all that apply. 1 Weight gain 2 Abdominal pain 3 Discoid lesions 4 Inflamed red rash 5 Deep vein thrombosis

Correct = 2, 3, 4

Which parameters should be monitored in an older adult patient with osteoarthritis (OA) who is taking 3500 g of acetaminophen daily? Select all that apply. 1 Serum glucose 2 Serum bilirubin 3 Serum albumin 4 Prothrombin time 5 Low-density lipoprotein

Correct = 2, 3, 4 Serum bilirubin, serum albumin, and prothrombin time are measured to assess liver function. Acetaminophen causes liver damage if taken in large amounts, especially in older adult patients because of aging and decreased excretion rates; therefore, liver function tests should be performed.

A patient who recently underwent total hip arthroplasty and is on anticoagulants is preparing for discharge from the hospital. Which information is most important for the nurse to provide to the patient and caregiver? 1 Use an abduction pillow between the legs. 2 Keep heels off the bed. 3 Avoid using a straight razor. 4 Reorient frequently.

Correct = 3 Avoid using a straight razor The patient will be on anticoagulants for 4-6 weeks at home and should avoid injury to the skin, including when shaving.

A patient with suspected rheumatoid arthritis had an arthrocentesis. Which components present in the synovial fluid would indicate rheumatoid arthritis? Select all that apply. 1 Platelets 2 Red blood cells 3 Immune complex 4 Inflammatory cells 5 Rheumatoid factor

Correct = 3, 4, 5 Arthrocentesis is an invasive procedure that aspirates synovial fluid. The fluid is analyzed for immune complexes, inflammatory cells, and rheumatoid factor.

Which symptoms does the nurse anticipate in a patient who is in the late stage of rheumatoid arthritis (RA)? Select all that apply. 1 Anorexia 2 Paresthesias 3 Deformities 4 Osteoporosis 5 Severe fatigue 6 Peripheral neuropathy

Correct = 3, 4, 5, 6

Which statement given by the patient would make the nurse suspect a diagnosis of Raynaud's phenomenon? 1 "I have difficulty swallowing." 2 "I find red-colored lesions around my nail beds." 3 "I find redness and swelling on my great toe." 4 "I have severe pain in my fingers, and they turn blue in cold conditions."

Correct = 4 "I have severe pain in my fingers, and they turn blue in cold conditions." Raynaud's phenomenon is associated with reduced blood flow in response to emotional stress or the cold. It is characterized by severe pain and fingers that turn blue, red, or white in cold or stressed conditions.

What is the gastrointestinal sign of systemic lupus erythematosus? 1 Ulcers 2 Dysphagia 3 Esophagitis 4 Abdominal pain

Correct = 4 Abdominal Pain Abdominal pain, which occurs due to peritoneal involvement, is the gastrointestinal manifestation of systemic lupus erythematosus (SLE). Ulcers, dysphagia, and esophagitis are the gastrointestinal manifestations of systemic sclerosis.

Which condition is the consequence of pannus formation in a patient with rheumatoid arthritis? 1 Vasculitis 2 Pericarditis 3 Paresthesias 4 Bony ankylosis

Correct = 4 Bony ankylosis In rheumatoid arthritis patients, the synovium may thicken and become hyperemic, causing fluid accumulation in the joint space. This condition may lead to the formation of pannus. This pannus formation may erode articular cartilage and eventually destroy the bone. Therefore, as the destruction of the bone progresses, bony ankylosis may occur. Vasculitis is one of the systemic complications of rheumatoid arthritis. Pericarditis is one of the complications of rheumatoid arthritis that may occur in the later stages. Paresthesias may occur as a result of vasculitis in patients with rheumatoid arthritis.

What is the characteristic feature of Bouchard's nodes? 1 Excess fluid in the joint 2 Atrophy of the skeletal muscle 3 Bony nodules at the distal interphalangeal joints 4 Bony nodules at the proximal interphalangeal joints

Correct = 4 Bony nodules at the proximal interphalangeal joints Bouchard's nodes are nodes at the proximal interphalangeal joints caused by bony hypertrophy. Excess fluid in the joint indicates joint effusion. Heberden's nodes are bony nodules at the distal interphalangeal joints. Skeletal muscle atrophy occurs with muscle disuse related to pain and stiffness associated with osteoarthritis.

Which statement describes systemic lupus erythematosus (SLE)? 1 It is nonfatal. 2 It affects men more often than women. 3 It is an acute, nonprogressive disorder. 4 It is characterized by spontaneous remissions and exacerbations.

Correct = 4 It is characterized by spontaneous remissions and exacerbations. It is fatal, but patients with SLE can lead a productive life. It affects women 10 times more often than men. It is not an acute, progressive disorder, but it is a chronic, progressive disorder.

Which is a late manifestation of rheumatoid arthritis? 1 Anorexia 2 Paresthesias 3 Joint inflammation 4 Morning stiffness

Correct = 4 Morning stiffness Symptoms such as anorexia, paresthesias, and inflammation are EARLY manifestations of rheumatoid arthritis.

While caring for a patient with rheumatoid arthritis, the nurse finds that the patient has a single hot, swollen, and painful wrist joint. Which is the best nursing intervention for this patient? 1 Applying ice immediately 2 Administering analgesics immediately 3 Administering antibiotics immediately 4 Notifying the primary health care provider

Correct = 4 Notifying the Primary health provider The single, hot, swollen and painful wrist joint indicates the joint is infected. The primary health care provider (PHP) should be notified immediately. Ice should be applied only if the PHP advises it. The nurse should not administer analgesics or antibiotics unless the PHP prescribes them.

Which statement is true regarding osteoarthritis? 1 Pain worsens after rest in early stage of the disease. 2 Pain is caused by the nerve supply to the cartilage. 3 Pain diminishes after activity in early stage of the disease. 4 Pain is caused by spasms of the surrounding muscles

Correct = 4 Pain is caused by spasms of the surrounding muscles Because cartilage does not have a nerve supply, the pain is caused by joint and soft tissue involvement and by spasms of the surrounding muscles. The pain diminishes after rest in the early stage of the disease. Pain will increase after activity in the early stages of the disease.

The nurse encourages a patient who has undergone total hip joint arthroplasty to drink fluids and instructs the patient's caregivers not to massage the leg. The nurse also observes for signs of redness, swelling, and pain in the patient's leg. Which complication is the nurse seeking to avoid with these interventions? 1 Dislocation 2 Infection 3 Hypotension 4 Venous thromboembolism

Correct = 4 Venous thromboembolism A patient with venous thromboembolism may show redness, swelling, and pain in the region that underwent surgery. The patient should drink a high amount of fluids and avoid massaging the region of surgery in order to decrease the risk of venous thromboembolism.

The nurse is developing a teaching plan for a patient diagnosed with osteoarthritis (OA). The nurse includes which instruction in the teaching plan? 1 Begin a running program. 2 Take up knitting to slow down joint degeneration. 3 Eat at least two cups of yogurt per day. 4 Wear supportive shoes.

Correct = 4 Wear supportive shoes. Wearing supportive shoes will help prevent falls and damage to foot joints, especially metatarsal joints. Running promotes stress on joints and should be avoided. Repetitive stress activities such as knitting or typing should be avoided for prolonged periods. No single food can cure OA; a well-balanced diet is recommended.

What medication used in the treatment of lupus erythematosus decreases the risk for skin lesions? 1 Belimumab 2 Azathioprine 3 Methotrexate 4 Hydroxychloroquine

Correct = Hydroxychloroquine Hydroxychloroquine is an anti-malarial medication used to treat lupus erythematosus. It decreases the absorption of ultraviolet light by the skin and thereby decreases the risk for skin lesions.

Which drug is beneficial for a patient with osteoarthritis who is having painful muscle spasms? Diclofenac Hyaluronic acid Acetaminophen Cyclobenzaprine hydrochloride

Cyclobenzaprine hydrochloride Cyclobenzaprine hydrochloride is a muscle relaxant used to treat muscle spasms. Diclofenac, hyaluronic acid, and acetaminophen are used to treat pain associated with osteoarthritis but are not used to relieve muscle spasms.

A nurse is caring for a client who is scheduled to undergo a thoracentesis. Which intervention should the nurse complete prior to the procedure? a. Measure oxygen saturation before and after a 12-minute walk. b. Verify that the client understands all possible complications. c. Explain the procedure in detail to the client and the family. d. Validate that informed consent has been given by the client.

D A thoracentesis is an invasive procedure with many potentially serious complications. Verifying that the client understands complications and explaining the procedure to be performed will be done by the physician or nurse practitioner, not the nurse. Measurement of oxygen saturation before and after a 12-minute walk is not a procedure unique to a thoracentesis.

A nurse assesses a clients respiratory status. Which information is of highest priority for the nurse to obtain? a. Average daily fluid intake b. Neck circumference c. Height and weight d. Occupation and hobbies

D Many respiratory problems occur as a result of chronic exposure to inhalation irritants used in a clients occupation and hobbies. Although it will be important for the nurse to assess the clients fluid intake, height, and weight, these will not be as important as determining his occupation and hobbies. Determining the clients neck circumference will not be an important part of a respiratory assessment.

The nurse is assessing a patient admitted with a stroke. What assessment finding would indicate that the patient experienced a stroke to the right hemisphere? Slowness Denial of the illness Anger and frustration Deficit in the right visual field

Denial of the illness The patient who has a right hemisphere stroke is often unaware of any deficits and may be disoriented to time and place. Deficit in the right visual field, slowness, and anger and frustration are the symptoms of a left hemisphere stroke.

A client is being discharged on warfarin ( Coumadin ) therapy. What discharge instructions is the nurse required to provide? A. Dietary restrictions b. Driving restrictions c. Follow-up laboratory monitoring d. Possible drug-drug interactions e. Reason to take medication

Dietary restrictions Follow up laboratory monitoring Possible drug-drug interactions Reason to take medication The Joint Commission's Core Measures state that clients being discharged on warfarin need instruction on follow-up monitoring, dietary restrictions, drug-drug interactions, and reason for compliance. Driving is typically not restricted.

A client had a percutaneous transluminal coronary angioplasty for peripheral arterial disease. What assessment finding by the nurse indicates a priority outcome for this client has been met? a. Pain rated as 2/10 after medication b. Distal pulse on affected extremity 2+/4+ c. Remains on bedrest as directed d. Verbalizes understanding of procedure

Distal pulse on affected extremity 2+/4+ Assessing circulation distal to the puncture site is a critical nursing action. A pulse of 2+/4+ indicates good perfusion. Pain control, remaining on bedrest as directed after the procedure, and understanding are all important, but do not take priority over perfusion.

A client presents to the emergency department with a severely lacerated artery. What is the priority action for the nurse? a. Administer oxygen via non-rebreather mask. b. Ensure the client has a patent airway. c. Prepare to assist with suturing the artery. d. Start two large-bore IVs with normal saline

Ensure the client has a patent airway Airway always takes priority, followed by breathing and circulation. The nurse ensures the client has a patent airway prior to providing any other care measures.

Which type of fracture does not divide the bone into two portions because the break is through only part of the bone? Fragility Complete Incomplete Compression

Incomplete An incomplete fracture does not divide the bone into two portions; it is only broken through a part of the bone. A fragility fracture occurs after minimal trauma to a bone that has been weakened by disease. A complete fracture refers to a break that is across the entire width of the bone in such a way that the bone is divided into two distinct sections. Compression fractures are produced by a loading force applied to the long axis of cancellous bone.

When checking a patient with fractures, a wound, and extensive soft tissue damage for signs of infection, which results indicate that there is no infection present? Select all that apply. Lack of fever Negative wound culture Increased phosphorous level Increased serum calcium level No increase in white blood cell count Decreased erythrocyte sedimentation rate

Lack of fever, negative wound culture, no increase in WBC No fever, a negative wound culture, and no increase in white blood cell count indicate a lack of infection. During the healing stages, phosphorous and serum calcium levels are often increased because the bone releases these minerals into the bloodstream. This has nothing to do with the presence of an infection. If extensive soft tissue damage is present, the erythrocyte sedimentation rate might be elevated. This indicates inflammation. If both the erythrocyte sedimentation rate and the white blood cell count are elevated, this might indicate an infection.

A nurse is assessing an obese client in the clinic for follow-up after an episode of deep vein thrombosis. The client has lost 20 pounds since the last visit. What action by the nurse is best? a. Ask if the weight loss was intended. b. Encourage a high-protein, high-fiber diet. c. Measure for new compression stockings. d. Review a 3-day food recall diary.

Measure for new compression stockings Compression stockings must fit correctly in order to work. After losing a significant amount of weight, the client should be re-measured and new stockings ordered if needed. The other options are appropriate, but not the most important.

A nurse collaborates with an unlicensed assistive personnel (UAP) to provide care for a client with congestive heart failure. Which instructions should the nurse provide to the UAP when delegating care for this client? (Select all that apply.) a. "Reposition the client every 2 hours." b. "Teach the client to perform deep-breathing exercises." c. "Accurately record intake and output." d. "Use the same scale to weigh the client each morning." e. "Place the client on oxygen if the client becomes short of breath."

NS: A, C, D The UAP should reposition the client every 2 hours to improve oxygenation and prevent atelectasis. The UAP can also accurately record intake and output, and use the same scale to weigh the client each morning before breakfast. UAPs are not qualified to teach clients or assess the need for and provide oxygen therapy.

A client is receiving an infusion of alteplase ( Activase ) for an intra-arterial clot. The client begins to mumble and is disoriented. What action by the nurse takes priority? a. Assess the client's neurologic status. b. Notify the Rapid Response Team. c. Prepare to administer vitamin K. d. Turn down the infusion rate.

Notify the Rapid Response Team Clients on fibrinolytic therapy are at high risk of bleeding. The sudden onset of neurologic signs may indicate the client is having a hemorrhagic stroke. The nurse does need to complete a thorough neurological examination, but should first call the Rapid Response Team based on the client's manifestations. The nurse notifies the Rapid Response Team first. Vitamin K is not the antidote for this drug. Turning down the infusion rate will not be helpful if the client is still receiving any of the drug.

Which factors contraindicate administering alteplase more than three hours after stroke onset? Select all that apply. Age older than 80 years History of both diabetes and stroke Use of warfarin or other anticoagulants Imaging evidence of middle cerebral artery involvement National Institutes of Health Stroke Scale (NIHSS) score less than 25

Older than 80, Hx of diabetes and stroke, Use of warfarin and other anticoag, Imaging evidence of middle cerebral artery involvement. The optimal time for administering alteplase is within three hours; however, in some cases that window can be extended to 4.5 hours. Factors precluding this extension include advanced age (over 80 years), imaging evidence of middle cerebral artery involvement, history of both diabetes and stroke, and treatment with anticoagulants. An NIHSS score of greater than, not less than, 25 also precludes this extension.

Which type of trauma increases a patient's risk for hospital-acquired infection? Malunion Osteonecrosis Open fractures Compartment syndrome

Open fracture Open fractures increase the risk of osteomyelitis, which also increases the risk for hospital-acquired infections. Malunion is incorrect bone healing. Osteonecrosis and compartment syndrome are complications of closed fractures.

A nurse is caring for a client with deep vein thrombosis ( DVT ). What nursing assessment indicates a priority outcome has been met? a. Ambulates with assistance b. Oxygen saturation of 98% c. Pain of 2/10 after medication d. Verbalizing risk factors

Oxygen saturation of 98% A critical complication of DVT is pulmonary embolism. A normal oxygen saturation indicates that this has not occurred. The other assessments are also positive, but not the priority.

A nursing student is caring for a client with an abdominal aneurysm. What action by the student requires the registered nurse to intervene? a. Assesses the client for back pain b. Auscultates over abdominal bruit c. Measures the abdominal girth d. Palpates the abdomen in four quadrants

Palpates the abdomen in four quadrants Abdominal aneurysms should never be palpated as this increases the risk of rupture. The registered nurse should intervene when the student attempts to do this. The other actions are appropriate.

A student nurse is assessing the peripheral vascular system of an older adult. What action by the student would cause the faculty member to intervene? a. Assessing blood pressure in both upper extremities b. Auscultating the carotid arteries for any bruits c. Classifying capillary refill of 4 seconds as normal d. Palpating both carotid arteries at the same time

Palpating both carotid arteries at the same time The student should not compress both carotid arteries at the same time to avoid brain ischemia. Blood pressure should be taken and compared in both arms. Prolonged capillary refill is considered to be greater than 5 seconds in an older adult, so classifying refill of 4 seconds as normal would not require intervention. Bruits should be auscultated.

Which type of traction is used for strain or sprain in the lower back? Pelvic belt Pelvic sling Russell traction Buck extension traction

Pelvic Belt A pelvic belt is used for strain or sprain in the lower back. A pelvic sling is used for pelvic fractures. Russell traction is used for fractures of the hip or the distal end of the femur. Buck extension traction is used for fractures of the hip or femur and hip dislocation.

Which disease manifests as fever, weight loss, generalized weakness, fatigue, and the inflammation of the proximal interphalangeal (PIP) and metacarpophalangeal (MCP) joints of the hands? Rheumatoid arthritis (RA) Fibromyalgia syndrome (FMS) Chronic fatigue syndrome (CFS) Systemic lupus erythematosus (SLE)

Rheumatoid Arthritis Fever, weight loss, generalized weakness, and fatigue are early manifestations of RA. This disease is also characterized by inflammation of the PIP and MCP joints of the hands. FMS is characterized by heartburn, dysuria, urinary frequency, urgency, and pelvic pain. CFS is characterized by severe fatigue, sore throat, muscle pain, and unrefreshing sleep. SLE is characterized by abdominal pain, vasculitis, anorexia, and Raynaud phenomenon.

Which statement best describes the symptoms of a transient ischemic attack (TIA)? They resolve within 60 minutes. They are limited to the speech area. They manifest in the upper extremities. They last longer than 24 hours but less than a week.

They resolve within 60 minutes. By definition, symptoms of a TIA resolve typically within 30-60 minutes. TIA symptoms can manifest as weakness in the arms, hands, or legs, and gait disturbance is typical. Speech deficits (aphasia, dysarthria) can result from TIA, but symptoms are not limited to this area.

The nurse is preparing to teach a diabetic patient how to select appropriate shoes. Which points must be included in the teaching plan? Select all that apply. a. "It is best to have the shoes fitted by an experienced shoe fitter such as a podiatrist." b. "The shoes should be 1-1.5 inches longer than your longest toe." c. "The heels of the shoes should be less than 2 inches high." d. "Avoid tight fitting shoes, which can cause tissue damage to your feet." e. "You should get at least two pairs of shes so you can change them at midday and in the evening."

a. "It is best to have the shoes fitted by an experienced shoe fitter such as a podiatrist." c. "The heels of the shoes should be less than 2 inches high." d. "Avoid tight fitting shoes, which can cause tissue damage to your feet." e. "You should get at least two pairs of shes so you can change them at midday and in the evening." QUESTION FROM THE STUDY GUIDE

A patient asks the nurse how insulin injection site rotation should be accomplished. What is the nurse's best response? a. "Rotation within one site is preferred to avoid changes in insulin absorption." b. "Change rotation sites after a week or two to avoid lipohypertrophy." c. "Rotation from site to site each day is best for the best insulin absorption." d. "Always rotate insulin injection sites within 4-5 inches from the umbilicus."

a. "Rotation within one site is preferred to avoid changes in insulin absorption." QUESTION FROM THE STUDY GUIDE

A patient with diabetic ketoacidosis is on an insulin drip of t50 units of regular insulin in 250 mL of normal saline. The current blood glucose level is 549 mg/dL. According to insulin protocol, the insulin drip needs to be changed to 8 units per hour. At what rate does the nurse set the pump? a. 40mL/hr b. 50mL/hr c. 60mL/hr d. 75mL/hr

a. 40mL/hr QUESTION FROM THE STUDY GUIDE

In which situations does the nurse teach a patient to perform urine ketone testing? Select all that apply. a. Acute illness or stress b. When blood glucose levels are above 240 mg/dL. c. When symptoms of DKA are present d. To evaluate the effectiveness of DKA treatment e. When a diabetic patient is in a weight-loss program

a. Acute illness or stress c. When symptoms of DKA are present e. When a diabetic patient is in a weight-loss program QUESTION FROM THE STUDY GUIDE

Which class of antidiabetic medication should be taken with the first bite of a meal to be fully effective? a. Alpha-glucosidase inhibitors, which include miglitol (Glyset) b. Biguanides, which include metformin (Glucophage) c. Meglitinides, which include nateglinide (Starlix) d. Second generation sulfonylureas, which include glipizide (Glucotrol)

a. Alpha-glucosidase inhibitors, which include miglitol (Glyset) QUESTION FROM THE STUDY GUIDE

Which infection control measures must ghe nurse teach a patient who will be performing SMBG? Select all that apply. a. Always wash hands before monitoring glucose. b. Regular cleaning of the meter is critical. c. Do not reuse lancets. d. Do not share blood glucose monitoring equipment. e. Sterilize blood glucose monitor before each use.

a. Always wash hands before monitoring glucose. b. Regular cleaning of the meter is critical. c. Do not reuse lancets. d. Do not share blood glucose monitoring equipment. QUESTION FROM THE STUDY GUIDE

The nurse is caring for a diabetic patient in the ED. The patient's lab values include serum glucose 353 mg/dL, positive serum ketones, and positive urine ketones. What complication does the nurse suspect? a. DKA b. HHS c. Hyperglycemia d. Hypoglycemia

a. DKA QUESTION FROM THE STUDY GUIDE

Which complications of DM are considered emergencies? Select all that apply. a. Diabetic ketoacidosis (DKA) b. Hypoglycemia c. Diabetic retinopathy d. Hyperglycemic - hyperosomolar state (HHS) e. Diabetic neuropathy

a. Diabetic ketoacidosis (DKA) b. Hypoglycemia d. Hyperglycemic - hyperosomolar state (HHS) QUESTION FROM THE STUDY GUIDE

A diabetic patient is scheduled to have a blood glucose test the next morning. What does the nurse tell the patient to do before coming in for the test? a. Eat the usual diet but have noting after midnight. b. Take the usual oral hypoglycemic tablet in the morning. c. Eat a clear liquid breakfast in the morning. d. Follow the usual diet and medication regimen.

a. Eat the usual diet but have noting after midnight. QUESTION FROM THE STUDY GUIDE

Which are signs and symptoms of mild hypoglycemia? Select all that apply. a. Headache b. Weakness c. Cold, clammy skin d. Irritability e. Pallor f. Tachycardia

a. Headache b. Weakness d. Irritability QUESTION FROM THE STUDY GUIDE

A patient is admitted with a blood glucose level of 900 mg/dL. IV fluids and insulin are administered. Two hours after treatment is initiated, the blood glucose level is 400 mg/dL. Which complication is the patient most at risk for developing? a. Hypoglycemia b. Pulmonary embolus c. Renal shutdown d. Pulmonary edema

a. Hypoglycemia QUESTION FROM THE STUDY GUIDE

Which statements about type 1 DM are accurate? Select all that apply. a. It is an autoimmune disorder. b. Most people with type 1 DM are obese. c. Age of onset is typically younger than 30. d. Etiology can be attributed to viral infections. e. It can be treated with oral antidiabetic medications and insulin.

a. It is an autoimmune disorder. c. Age of onset is typically younger than 30. d. Etiology can be attributed to viral infections. QUESTION FROM THE STUDY GUIDE

Which statements about type 2 DM are accurate? Select all that apply. a. It peaks at about the age of 50. b. Most people with type 2 DM are obese. c. It typically has an abrupt onset. d. People with type 2 DM have insulin resistance. e. It can be treated with oral antidiabetic medications and insulin.

a. It peaks at about the age of 50. b. Most people with type 2 DM are obese. d. People with type 2 DM have insulin resistance. e. It can be treated with oral antidiabetic medications and insulin. QUESTION FROM THE STUDY GUIDE

Which factors differentiate DKA from HHS? Select all that apply. a. Level of hyperglycemia b. Amount of ketones produced c. Serum bicarbonate levels d. Amount of volume depletion e. Dosage of insulin needed

a. Level of hyperglycemia b. Amount of keytones produced c. Serum bicarbonate levels d. Amount of volume depletion e. Dosage of insulin needed QUESTION FROM THE STUDY GUIDE

Intensive therapy with good glucose control results in delays in which diabetic complications? Select all that apply. a. Macrovascular disease b. Cardiovascular disease c. Stroke d. Retinopathy e. Nephropathy f. Neuropathy

a. Macrovascular disease b. Cardiovascular disease d. Retinopathy e. Nephropathy f. Neuropathy QUESTION FROM THE STUDY GUIDE

In developing an individualized meal plan for a patient with diabetes, which goals will be focal points of the plan? Select all that apply. a. Maintaining blood glucose levels at or as close to the normal range as possible b. Patient food preferences c. Allowing patients to eat as much as they desire d. Patient cultural preferences e. Limiting food choices only when guided by scientific evidence

a. Maintaining blood glucose levels at or as close to the normal range as possible b. Patient food preferences d. Patient cultural preferences e. Limiting food choices only when guided by scientific evidence QUESTION FROM THE STUDY GUIDE

Which cultures tend to have a higher incidence of DM? Select all that apply. a. Mexican American b. African American c. Caucasian d. American Indian e. Eastern European

a. Mexican American b. African American d. American Indian QUESTION FROM THE STUDY GUIDE

The patient's urinalysis shows proteinuria. Which pathophysiology does the nurse suspect? a. Nephropathy b. Neuropathy c. Retinopathy d. Gastroparesis

a. Nephropathy QUESTION FROM THE STUDY GUIDE

What type of exercise does the nurse recommend for the patient with diabetic retionpahy? a. Non weight bearing activities such as swimming b. Weight-bearing activities such as jogging c. Vigorous aerobic and resistance exercises d. Weight training and heavy lifting

a. Non weight bearing activities such as swimming QUESTION FROM THE STUDY GUIDE

For which patient should the health care provider avoid prescribing rosiglitazone (Avandia)? a. Patient with symptomatic heart failure b. Patient with new onset asthma c. Patient with kidney disease d. Patient with hyperthyroidism

a. Patient with symptomatic heart failure QUESTION FROM THE STUDY GUIDE

SMBG levels is most important in which patients? Select all that apply. a. Patients taking multiple daily insulin injections b. Patients with mild type 2 diabetes c. Patients with hypoglycemic unawareness d. Patients using a portable infusion device for insulin administration e. Patients with acute illnesses f. Pregnant patients

a. Patients taking multiple daily insulin injections c. Patients with hypoglycemic unawareness d. Patients using a portable infusion device for insulin administration e. Patients with acute illnesses f. Pregnant patients QUESTION FROM THE STUDY GUIDE

The patient with type 2 diabetes is prescribed sitagliptin (Januvia) for glucose regulation. Which key changes does the nurse teach a patient to report to the health care provider immediately? Select all that apply. a. Report any signs of jaundice. b. Report any signs of bleeding. c. Report any blue-grey discoloration of the abdomen. d. Report any cough or flu symptoms. e. Report any sudden onset of abdominal pain.

a. Report any signs of jaundice. c. Report any blue-grey discoloration of the abdomen. e. Report any sudden onset of abdominal pain. QUESTION FROM THE STUDY GUIDE

A patient will be using an external insulin pump. What does the nurse tell the patient about the pump? a. SMBG levels should be done three or more times a day. b. The insulin supply must be replaced every 2-4 weeks. c. The pump's batery should be checked on a regular weekly schedule. d. The needle site must be changed every day.

a. SMBG levels should be done three or more times a day. QUESTION FROM THE STUDY GUIDE

A patient has been diagnosed with DM. Which aspects does the nurse consider in formulating the teaching plan for this patient? Select all that apply. a. Covering all needed information in one teaching session b. Assessing visual impairment regarding insulin labels and markings on syringes c. Assessing manual dexterity to determine if the patient is able to draw insulin into a syringe d. Assessing patient motivation to learn and comprehend instructions e. Assessing the patient's ability to read printed material

b. Assessing visual impairment regarding insulin labels and markings on syringes c. Assessing manual dexterity to determine if the patient is able to draw insulin into a syringe d. Assessing patient motivation to learn and comprehend instructions e. Assessing the patient's ability to read printed material QUESTION FROM THE STUDY GUIDE

Which class of antidiabetic medication must be held after using contrast media until adequate kidney function is established? a. Alpha-glucosidase inhibitors, which include miglitol (Glyset) b. Biguanides, which include metformin (Glucophage) c. Meglitinides, which include nateglinide (Starlix) d. Second generation sulfonylureas, which include glipizide (Glucotrol)

b. Biguanides, which include metformin (Glucophage) QUESTION FROM THE STUDY GUIDE

The critical care nurse is caring for an older patient admitted with HHS. What is the first priority in caring for this patient? a. Slowly decreasing blood glucose b. Fluid replacement to increase blood volume c. Potassium replacement to prevent hypokalemia d. Diuretic therapy to maintain kidney function

b. Fluid replacement to increase blood volume QUESTION FROM THE STUDY GUIDE

In determining if a patient is hypoglycemic, the nurse looks for which characteristics in addition to checking the patient's blood glucose? Select all that apply. a. Nausea b. Hunger c. Irritability d. Palpitations e. Profuse perspiration f. Rapid, deep respirations

b. Hunger c. Irritability d. Palpitations e. Profuse perspiration QUESTION FROM THE STUDY GUIDE

Why is glucose vital to the body's cells? a. It is used to build cell membranes. b. It is used by cells to produce energy. c. It affects the process of protein metabolism. d. It provides nutrients for genetic material.

b. It is used by cells to produce energy. QUESTION FROM THE STUDY GUIDE

Which diabetic complication is associated with neuropathy? a. End stage kidney disease b. Muscle weakness c. Permanent blindness d. Eye hemorrhage

b. Muscle weakness QUESTION FROM THE STUDY GUIDE

The older adult with DM asks the nurse for advice about beginning an exercise program. What is the nurse's best response? Select all that apply. a. Begin with high intensity activities. b. Start low intensity activities in short sessions. c. Be sure to include warm up and cool down periods. d. Start with periods of 20 minutes or less. e. Changes in activity should be gradual.

b. Start low intensity activities in short sessions. c. Be sure to include warm up and cool down periods. e. Changes in activity should be gradual. QUESTION FROM THE STUDY GUIDE

A diabetic patient is on a mixed dose insulin protocol of 8 unites regular insulin and 12 units NPH insulin at 7am. At 10:30am, the patient reports feeling uneasy, shaky, and has a headache. Which is the probable explanation for this? a. The NPH insulin's action is peaking, and there is an insufficient blood glucose level. b. The regular insulin's action is peaking, and there is an insufficient blood glucose level. c. The patient consumed too many calories at breakfast and now has an elevated blood glucose level. d. They symptoms are unrelated to the insulin administered in the early morning or food taken in at lunchtime.

b. The regular insulin's action is peaking, and there is an insufficient blood glucose level. QUESTION FROM THE STUDY GUIDE

The nurse is providing discharge teaching to a patient about self-monitoring of blood glucose (SMBG). What information does the nurse include? Select all that apply. a. Only perform SMBG before breakfast. b. Wash hands before using the meter. c. Do a retest if the results seem unusual. d. It is okay to reuse lancets in the home setting. e. Do not share the meter.

b. Wash hands before using the meter. c. Do a retest if the results seem unusual. e. Do not share the meter. QUESTION FROM THE STUDY GUIDE

Which are characteristics of regular insulin? Select all that apply. a. This insulin does not have a peak time. b. When mixing types of insulin, this insulin is always drawn up first. c. This insulin is given once daily for basal insulin coverage. d. This insulin should be given 30 minutes before meals. e. This insulin should not be diluted or mixed with any other insulin.

b. When mixing types of insulin, this insulin is always drawn up first. d. This insulin should be given 30 minutes before meals. QUESTION FROM THE STUDY GUIDE

Which statement by a patient with DM indicates an understanding of the principles of self-care? a. "I don't like the idea of sticking myself so often to measure my sugar." b. "I plan to measure the sugar in my urine at least four times a day." c. "I plan to get my spouse to exercise with me to keep me company." d. "If I get a cold, I can take my regular cough medication until I feel better."

c. "I plan to get my spouse to exercise with me to keep me company." QUESTION FROM THE STUDY GUIDE

Which individual is at greatest risk for developing type 2 DM? a. 25 year old African American woman b. 36 year old African American man c. 56 year old Hispanic woman d. 40 year old Hispanic man

c. 56 year old Hispanic woman QUESTION FROM THE STUDY GUIDE

After a 2 hour glucose challenge, which result demonstrates impaired glucose tolerance? a. Less than 100 mg/dL b. Less than 140 mg/dL c. Greater than 140 mg/dL d. Greater than 250 mg/dL

c. Greater than 140 mg/dL QUESTION FROM THE STUDY GUIDE

Which statement about sexual intercourse for patients with diabetes is true? a. The incidence of sexual dysfunction is lower in men than women. b. Retrograde ejaculation does not interfere with male fertility. c. Impotence is associated with DM in male patients. d. Sexual dysfunction in female patients includes inability to achieve pregnancy.

c. Impotence is associated with DM in male patients. QUESTION FROM THE STUDY GUIDE

A 47 year old patient with a history of type 2 DM and emphysema who reports smoking three packs of cigaretes per day is admitted to the hospital with a diagnosis of acute pneumonia. The patient is placed on the regular oral antidiabetic agents, sliding-scale insulin, and antibiotic medications. On day 2 of hospitalization, the health care provider orders prednisone therapy. What does the nurse expect the blood glucose to do? a. Decrease b. Stay the same c. Increase d. Return to normal

c. Increase QUESTION FROM THE STUDY GUIDE

Which statement is true about insulin? a. It is secreted by alpha cells in the islets of Langerhans. b. It is a catabolic hormone that builds up glucagon reserves. c. It is necessary for glucose transport across cell membranes. d. It is stored in muscles and converted to fat for storage.

c. It is necessary for glucose transport across cell membranes. QUESTION FROM THE STUDY GUIDE

In a patient with hyperglycemia, the respiratory center is triggered in an attempt to excrete more carbon dioxide and acid, thus causing a rapid and deep respiratory pattern. What is the term for this respiratory pattern? a. Tachypnea b. Cheyne-Stokes respiration c. Kussmaul respiration d. Biot respiration

c. Kussmaul respiration QUESTION FROM THE STUDY GUIDE

Which statements about sensory alteration in patients with diabetes are accurate? Select all that apply. a. Healing of foot wounds is reduced because of impaired sensation. b. Very few patients with diabetic foot ulcers have peripheral sensory neuropathy. c. Loss of pain, pressure, and temperature sensation in the foot increases the risk for injury. d. Sensory neuropathy causes loss of normal sweating and skin temperature regulation. e. It can be delayed by keeping the blood glucose level as close to normal as possible.

c. Loss of pain, pressure, and temperature sensation in the foot increases the risk for injury. d. Sensory neuropathy causes loss of normal sweating and skin temperature regulation. e. It can be delayed by keeping the blood glucose level as close to normal as possible. QUESTION FROM THE STUDY GUIDE

Which class of antidiabetic medication should be given 1-30 minutes before meals? a. Alpha-glucosidase inhibitors, which include miglitol (Glyset) b. Biguanides, which include metformin (Glucophage) c. Meglitnides, which include nateglinide (Starlix) d. Sulfonylureas, which include chlorpromadine (Diabinese)

c. Meglitnides, which include nateglinide (Starlix) QUESTION FROM THE STUDY GUIDE

What is the recommended protocol for patients with type 2 DM who must lose weight? a. Participate in an aerobic program twice a week for 20 minutes each session. b. Slowly increase insulin dosage until mild hypoglycemia occurs. c. Reduce calorie intake moderately and increase exercise. d. Reduce daily calorie intake to 1000 calories and monitor urine for ketones.

c. Reduce calorie intake moderately and increase exercise. QUESTION FROM THE STUDY GUIDE

Which statement about insulin administration is correct? a. Insulin may be given orally, intravenously, or subcutaneously. b. Insulin injections should be spaced no closer than one-half inch apart. c. Rotating injection sites improves absorption and prevents lipohypertrophy. d. Shake the bottle of intermediate-acting insulin, and then draw it into the syringe.

c. Rotating injection sites improves absorption and prevents lipohypertrophy. QUESTION FROM THE STUDY GUIDE

Glucagon is used primarily to treat the patient with which disorder? a. KDA b. Idiosyncratic reaction to insulin c. Severe hypoglycemia d. HHNS

c. Severe hypoglycemia QUESTION FROM THE STUDY GUIDE

The male diabetic patient asks the nurse for advice about alcohol consumption. What is the nurse's best response? a. "It is best to have alcohol near bedtime." b. "As long as your diabetes is under control you can drink as much as you like." c. "You should drink only one alcoholic beverage with each meal." d. "Avoid more than two drinks a day and have them with or shortly after meals."

d. "Avoid more than two drinks a day and have them with or shortly after meals." QUESTION FROM THE STUDY GUIDE

A 25 year old female patient with type 1 DM tells the nurse, "I have two kidneys and I'm still young. I expect to be around for a long time, so why should I worry about my blood sugar?" What is the nurse's best response? a. "You have little to worry about as long as your kidneys keep making urine." b. "You should discuss this with your physician because you are being unrealistic." c. "You would be right if your diabetes was managed with insulin." d. "Keeping your blood sugar under control now can help to prevent damage to both kidneys."

d. "Keeping your blood sugar under control now can help to prevent damage to both kidneys." QUESTION FROM THE STUDY GUIDE

According to the American Diabetes Association (ADA), which laboratory finding is most indicative of DM? a. Fasting blood glucose = 80 mg/dL b. 2 hour postprandial blood glucose = 110 mg/dL c. 1 hour glucose tolerance blood glucose = 110mg/dL d. 2 hour glucose tolerance blood glucose = 210 mg/dL

d. 2 hour glucose tolerance blood glucose = 210 mg/dL QUESTION FROM THE STUDY GUIDE

A patient with diabetes has signs and symptoms of hypoglycemia. The aptient has a blood glucose of 56 mg/dL, is not alert but responds to voice, and is confused and is unable to swallow fluids. What does the nurse do next? a. Give a glass of orange juice with two packets of sugar and continue to monitor the patient. b. Give a glass of orange or other type of juice and continue to monitor the patient. c. Give a complex carbohydrate and continue to monitor the patient. d. Administer D50 IV push.

d. Administer D50 IV push. QUESTION FROM THE STUDY GUIDE

What is the basic principle of meal planning for a patient with type 1 DM? a. Five small meals per day plus a bedtime snack b. Taking extra insulin when planning to eat sweet foods c. High protein, low carbohydrate, and low fiber foods d. Considering the effects and peak action times of the patient's insulin

d. Considering the effects and peak action times of the patient's insulin QUESTION FROM THE STUDY GUIDE

A nurse plans care for a client admitted with a snakebite to the right leg. With whom should the nurse collaborate? a. The facilitys neurologist b. The poison control center c. The physical therapy department d. A herpetologist (snake specialist)

ANS: B For the client with a snakebite, the nurse should contact the regional poison control center immediately for specific advice on antivenom administration and client management.

ANS: A Right lower quadrant pain, specifically at McBurney's point, is characteristic of appendicitis. Usually if nausea and vomiting begin first, the client has gastroenteritis. Marked peristalsis and hyperactive bowel sounds are not indicative of appendicitis. Abdominal pain due to appendicitis decreases with knee flexion.

A nurse assesses a client who has appendicitis. Which clinical manifestation should the nurse expect to find? a. Severe, steady right lower quadrant pain b. Abdominal pain associated with nausea and vomiting c. Marked peristalsis and hyperactive bowel sounds d. Abdominal pain that increases with knee flexion

ANS: C Although the client with severe diarrhea may experience skin irritation and hypovolemia, the client is most at risk for cardiac dysrhythmias secondary to potassium and magnesium loss from severe diarrhea. The client should have her or his electrolyte levels monitored, and electrolyte replacement may be necessary. Oral mucosa inspection, recent dietary intake, and abdominal percussion are important parts of physical assessment but are lower priority for this client than heart rate and rhythm.

A nurse assesses a client who has ulcerative colitis and severe diarrhea. Which assessment should the nurse complete first? a. Inspection of oral mucosa b. Recent dietary intake c. Heart rate and rhythm d. Percussion of abdomen

ANS: B A client with botulism is at risk for respiratory failure. This client's respiratory rate is slow, which could indicate impending respiratory distress or failure. The nurse should remain with the client while another nurse notifies the provider. The nurse should monitor and document the IV infusion per protocol, but this client does not require additional intravenous fluids. Allowing the client to rest or checking the client's blood glucose and administering orange juice are not appropriate actions.

A nurse assesses a client who is hospitalized for botulism. The client's vital signs are temperature: 99.8° F (37.6° C), heart rate: 100 beats/min, respiratory rate: 10 breaths/min, and blood pressure: 100/62 mm Hg. Which action should the nurse take? a. Decrease stimulation and allow the client to rest. b. Stay with the client while another nurse calls the provider. c. Increase the client's intravenous fluid replacement rate. d. Check the client's blood glucose and administer orange juice.

ANS: C The nurse expects high-pitched, rushing bowel sounds due to narrowing of the bowel lumen in Crohn's disease. A positive Murphy's sign is indicative of gallbladder disease, and rebound tenderness often indicates peritonitis. Dullness in the lower abdominal quadrants and hypoactive bowel sounds are not commonly found with Crohn's disease. Nightly worsening of abdominal cramping is not consistent with Crohn's disease.

A nurse assesses a client who is hospitalized with an exacerbation of Crohn's disease. Which clinical manifestation should the nurse expect to find? a. Positive Murphy's sign with rebound tenderness to palpitation b. Dull, hypoactive bowel sounds in the lower abdominal quadrants c. High-pitched, rushing bowel sounds in the right lower quadrant d. Reports of abdominal cramping that is worse at night

ANS: A The nurse should assess the stoma for color and contact the health care provider if the stoma is pale, bluish, or dark. The nurse should expect the client to have an intact ostomy pouch with dark green liquid stool that may contain some blood.

A nurse assesses a client who is recovering from an ileostomy placement. Which clinical manifestation should alert the nurse to urgently contact the health care provider? a. Pale and bluish stoma b. Liquid stool c. Ostomy pouch intact d. Blood-smeared output

ANS: A The presence of strictures predisposes the client to intestinal obstruction. Abdominal distention may indicate that the client has developed an obstruction of the large bowel, and the client's provider should be notified right away. Low-grade fever, bloody diarrhea, and abdominal cramps are common symptoms of Crohn's disease.

A nurse assesses a client with Crohn's disease and colonic strictures. Which clinical manifestation should alert the nurse to urgently contact the health care provider? a. Distended abdomen b. Temperature of 100.0° F (37.8° C) c. Loose and bloody stool d. Lower abdominal cramps

ANS: A, B, D Lower GI bleeding can lead to erosion of the bowel wall. Abscesses are localized pockets of infection that develop in the ulcerated bowel lining. Nonmechanical bowel obstruction is paralysis of the colon that results from colorectal cancer. When the inflammation is transmural, fistulas can occur between the bowel and bladder resulting in pyuria and fecaluria. Paralysis of the colon causing dilation and subsequent colonic ileus is known as a toxic megacolon.

A nurse assesses a client with ulcerative colitis. Which complications are paired correctly with their physiologic processes? (Select all that apply.) a. Lower gastrointestinal bleeding - Erosion of the bowel wall b. Abscess formation - Localized pockets of infection develop in the ulcerated bowel lining c. Toxic megacolon - Transmural inflammation resulting in pyuria and fecaluria d. Nonmechanical bowel obstruction - Paralysis of colon resulting from colorectal cancer e. Fistula - Dilation and colonic ileus caused by paralysis of the colon

ANS: A Metronidazole is the drug of choice for a Giardia infection. Ciprofloxacin and ceftriaxone are antibiotics used for bacterial infections. Sulfasalazine is used for ulcerative colitis and Crohn's disease.

A nurse cares for a client who has a Giardia infection. Which medication should the nurse anticipate being prescribed for this client? a. Metronidazole (Flagyl) b. Ciprofloxacin (Cipro) c. Sulfasalazine (Azulfidine) d. Ceftriaxone (Rocephin)

ANS: D Severe infection with C. botulinum can lead to respiratory failure, so assessments of oxygen saturation and respiratory rate are of high priority for clients with suspected C. botulinum infection. The other assessments may be completed after the respiratory system has been assessed.

A nurse cares for a client who has food poisoning resulting from a Clostridium botulinum infection. Which assessment should the nurse complete first? a. Heart rate and rhythm b. Bowel sounds c. Urinary output d. Respiratory rate

ANS: C Asacol is the oral formula for mesalamine and is produced as an enteric-coated pill that should not be crushed, chewed, or broken. Asacol is not available as a suspension or elixir. If the client is unable to swallow the Asacol pill, a mesalamine enema (Rowasa) may be administered instead, with a provider's order.

A nurse cares for a client who is prescribed mesalamine (Asacol) for ulcerative colitis. The client states, "I am having trouble swallowing this pill." Which action should the nurse take? a. Contact the clinical pharmacist and request the medication in suspension form. b. Empty the contents of the capsule into applesauce or pudding for administration. c. Ask the health care provider to prescribe the medication as an enema instead. d. Crush the pill carefully and administer it in applesauce or pudding.

ANS: C The ostomy nurse is a valuable resource for clients, providing suggestions for supplies and methods to manage the ostomy. A larger dress size will not necessarily help hide the ostomy appliance. Avoiding broccoli and carbonated drinks does not offer reassurance for the client. Ileostomies have an almost constant liquid effluent, so pouch removal during the prom is not feasible.

A nurse cares for a teenage girl with a new ileostomy. The client states, "I cannot go to prom with an ostomy." How should the nurse respond? a. "Sure you can. Purchase a prom dress one size larger to hide the ostomy appliance." b. "The pouch won't be as noticeable if you avoid broccoli and carbonated drinks prior to the prom." c. "Let's talk to the enterostomal therapist about options for ostomy supplies and dress styles." d. "You can remove the pouch from your ostomy appliance when you are at the prom so that it is less noticeable."

ANS: B Dehydration caused by diarrhea can occur quickly in older clients with Salmonella food poisoning, so maintenance of fluid balance is a high priority. Monitoring vital signs and providing perineal care are important nursing actions but are of lower priority than fluid replacement. The nurse should teach the client about proper hand hygiene to prevent the spread of infection, and preparation of food and beverages to prevent contamination.

A nurse cares for an older adult client who has Salmonella food poisoning. The client's vital signs are heart rate: 102 beats/min, blood pressure: 98/55 mm Hg, respiratory rate: 22 breaths/min, and oxygen saturation: 92%. Which action should the nurse complete first? a. Apply oxygen via nasal cannula. b. Administer intravenous fluids. c. Provide perineal care with a premedicated wipe. d. Teach proper food preparation to prevent contamination.

ANS: A, C, D, E Salmonella is usually contracted via contaminated eggs, beef, poultry, and green leafy vegetables. It is not transmitted through water in garden hoses or pools. Clients should wash leafy vegetables well, wash hands before and after using the restroom, make sure meat and eggs are cooked properly, and, because it can be transmitted by flies, keep flies off of food.

A nurse teaches a client how to avoid becoming ill with Salmonella infection again. Which statements should the nurse include in this client's teaching? (Select all that apply.) a. "Wash leafy vegetables carefully before eating or cooking them." b. "Do not ingest water from the garden hose or the pool." c. "Wash your hands before and after using the bathroom." d. "Be sure meat is cooked to the proper temperature." e. "Avoid eating eggs that are sunny side up or undercooked."

ANS: A The client should drink plenty of fluids to prevent dehydration. Milk products may not be tolerated. Caffeinated beverages increase intestinal motility and should be avoided.

A nurse teaches a client who has viral gastroenteritis. Which dietary instruction should the nurse include in this client's teaching? a. "Drink plenty of fluids to prevent dehydration." b. "You should only drink 1 liter of fluids daily." c. "Increase your protein intake by drinking more milk." d. "Sips of cola or tea may help to relieve your nausea."

ANS: A, C, E Rotavirus is more common among the youngest of clients. Not drinking water while swimming can help prevent E. coli infection. Parasitic diseases may take up to 2 weeks to become symptomatic. People with botulism need to be hospitalized to monitor for respiratory failure and paralysis. Escherichia coli is not transmitted by contact with infected animals.

A nurse teaches a community group about food poisoning and gastroenteritis. Which statements should the nurse include in this group's teaching? (Select all that apply.) a. "Rotavirus is more common among infants and younger children." b. "Escherichia coli diarrhea is transmitted by contact with infected animals." c. "To prevent E. coli infection, don't drink water when swimming." d. "Clients who have botulism should be quarantined within their home." e. "Parasitic diseases may not show up for 1 to 2 weeks after infection."

While on a camping trip, a nurse cares for an adult client who had a drowning incident in a lake and is experiencing agonal breathing with a palpable pulse. Which action should the nurse take first? a. Deliver rescue breaths. b. Wrap the client in dry blankets. c. Assess for signs of bleeding. d. Check for a carotid pulse.

ANS: A In this emergency situation, the nurse should immediately initiate airway clearance and ventilator support measures, including delivering rescue breaths.

A nurse teaches a community health class about water safety. Which statement by a participant indicates that additional teaching is needed? a. I can go swimming all by myself because I am a certified lifeguard. b. I cannot leave my toddler alone in the bathtub for even a minute. c. I will appoint one adult to supervise the pool at all times during a party. d. I will make sure that there is a phone near my pool in case of an emergency.

ANS: A People should never swim alone, regardless of lifeguard status. The other statements indicate good understanding of the teaching.

A provider prescribes a rewarming bath for a client who presents with partial-thickness frostbite. Which action should the nurse take prior to starting this treatment? a. Administer intravenous morphine. b. Wrap the limb with a compression dressing. c. Massage the frostbitten areas. d. Assess the limb for compartment syndrome.

ANS: A Rapid rewarming in a water bath is recommended for all instances of partial-thickness and full-thickness frostbite. Clients experience severe pain during the rewarming process and nurses should administer intravenous analgesics

An emergency department nurse cares for a middle-aged mountain climber who is confused and exhibits bizarre behaviors. After administering oxygen, which priority intervention should the nurse implement? a. Administer dexamethasone (Decadron). b. Complete a minimental state examination. c. Prepare the client for computed tomography of the brain. d. Request a psychiatric consult.

ANS: A The client is exhibiting signs of mountain sickness and high altitude cerebral edema (HACE). Dexamethasone (Decadron) reduces cerebral edema by acting as an anti-inflammatory in the central nervous system. The other interventions will not treat mountain sickness or HACE.

A nurse prepares to discharge a client with chronic pancreatitis. Which question should the nurse ask to ensure safety upon discharge? a. "Do you have a one- or two-story home?" b. "Can you check your own pulse rate?" c. "Do you have any alcohol in your home?" d. "Can you prepare your own meals?"

ANS: A A client recovering from chronic pancreatitis should be limited to one floor until strength and activity increase. The client will need a bathroom on the same floor for frequent defecation. Assessing pulse rate and preparation of meals is not specific to chronic pancreatitis. Although the client should be encouraged to stop drinking alcoholic beverages, asking about alcohol availability is not adequate to assess this client's safety.

A nurse cares for a client who has obstructive jaundice. The client asks, "Why is my skin so itchy?" How should the nurse respond? a. "Bile salts accumulate in the skin and cause the itching." b. "Toxins released from an inflamed gallbladder lead to itching." c. "Itching is caused by the release of calcium into the skin." d. "Itching is caused by a hypersensitivity reaction."

ANS: A In obstructive jaundice, the normal flow of bile into the duodenum is blocked, allowing excess bile salts to accumulate on the skin. This leads to itching, or pruritus. The other statements are not accurate.

A nurse cares for a client who presents with tachycardia and prostration related to biliary colic. Which actions should the nurse take? (Select all that apply.) a. Contact the provider immediately. b. Lower the head of the bed. c. Decrease intravenous fluids. d. Ask the client to bear down. e. Administer prescribed opioids.

ANS: A, B Clients who are experiencing biliary colic may present with tachycardia, pallor, diaphoresis, prostration, or other signs of shock. The nurse should stay with the client, lower the client's head, and contact the provider or Rapid Response Team for immediate assistance. Treatment for shock usually includes intravenous fluids; therefore, decreasing fluids would be an incorrect intervention. The client's tachycardia is a result of shock, not pain. Performing the vagal maneuver or administering opioids could knock out the client's compensation mechanism.

A nurse assesses a client with irritable bowel syndrome (IBS). Which questions should the nurse include in this client's assessment? (Select all that apply.) a. "Which food types cause an exacerbation of symptoms?" b. "Where is your pain and what does it feel like?" c. "Have you lost a significant amount of weight lately?" d. "Are your stools soft, watery, and black in color?" e. "Do you experience nausea associated with defecation?"

ANS: A, B, E The nurse should ask the client about factors that may cause exacerbations of symptoms, including food, stress, and anxiety. The nurse should also assess the location, intensity, and quality of the client's pain, and nausea associated with defecation or meals. Clients who have IBS do not usually lose weight and stools are not black in color.

A provider prescribes diazepam (Valium) to a client who was bitten by a black widow spider. The client asks, What is this medication for? How should the nurse respond? a. This medication is an antivenom for this type of bite. b. It will relieve your muscle rigidity and spasms. c. It prevents respiratory difficulty from excessive secretions. d. This medication will prevent respiratory failure.

ANS: B Black widow spider venom produces a syndrome known as latrodectism, which manifests as severe abdominal pain, muscle rigidity and spasm, hypertension, and nausea and vomiting. Diazepam is a muscle relaxant that can relieve pain related to muscle rigidity and spasms. It does not prevent respiratory difficulty or failure.

A client presents to the emergency department after prolonged exposure to the cold. The client is difficult to arouse and speech is incoherent. Which action should the nurse take first? a. Reposition the client into a prone position. b. Administer warmed intravenous fluids to the client. c. Wrap the clients extremities in warm blankets. d. Initiate extracorporeal rewarming via hemodialysis.

ANS: B Moderate hypothermia manifests with muscle weakness, increased loss of coordination, acute confusion, apathy, incoherence, stupor, and impaired clotting. Moderate hypothermia should be treated by core rewarming methods, which include administration of warm IV fluids, heated oxygen, and heated peritoneal, pleural, gastric, or bladder lavage, and by positioning the client in a supine position to prevent orthostatic changes. The clients trunk should be warmed prior to the extremities to prevent peripheral vasodilation. Extracorporeal warming with cardiopulmonary bypass or hemodialysis is a treatment for severe hypothermia.

On a hot humid day, an emergency department nurse is caring for a client who is confused and has these vital signs: temperature 104.1 F (40.1 C), pulse 132 beats/min, respirations 26 breaths/min, blood pressure 106/66 mm Hg. Which action should the nurse take? a. Encourage the client to drink cool water or sports drinks. b. Start an intravenous line and infuse 0.9% saline solution. c. Administer acetaminophen (Tylenol) 650 mg orally. d. Encourage rest and re-assess in 15 minutes.

ANS: B The client demonstrates signs of heat stroke. This is a medical emergency and priority care includes oxygen therapy, IV infusion with 0.9% saline solution, insertion of a urinary catheter, and aggressive interventions to cool the client, including external cooling and internal cooling methods. Oral hydration would not be appropriate for a client who has symptoms of heat stroke because oral fluids would not provide necessary rapid rehydration, and the confused client would be at risk for aspiration. Acetaminophen would not decrease this clients temperature or improve the clients symptoms. The client needs immediate medical treatment; therefore, rest and re-assessing in 15 minutes is inappropriate.

A nurse cares for a client who is recovering from a hemorrhoidectomy. The client states, "I need to have a bowel movement." Which action should the nurse take? a. Obtain a bedside commode for the client to use. b. Stay with the client while providing privacy. c. Make sure the call light is in reach to signal completion. d. Gather supplies to collect a stool sample for the laboratory.

ANS: B The first bowel movement after hemorrhoidectomy can be painful enough to induce syncope. The nurse should stay with the client. The nurse should instruct clients who are discharged the same day to have someone nearby when they have their first postoperative bowel movement. Making sure the call light is within reach is an important nursing action too, but it does not take priority over client safety. Obtaining a bedside commode and taking a stool sample are not needed in this situation.

A nurse teaches a client who is recovering from acute pancreatitis. Which statements should the nurse include in this client's teaching? (Select all that apply.) a. "Take a 20-minute walk at least 5 days each week." b. "Attend local Alcoholics Anonymous (AA) meetings weekly." c. "Choose whole grains rather than foods with simple sugars." d. "Use cooking spray when you cook rather than margarine or butter." e. "Stay away from milk and dairy products that contain lactose." f. "We can talk to your doctor about a prescription for nicotine patches."

ANS: B, D, F The client should be advised to stay sober, and AA is a great resource. The client requires a low-fat diet, and cooking spray is low in fat compared with butter or margarine. If the client smokes, he or she must stop because nicotine can precipitate an exacerbation. A nicotine patch may help the client quit smoking. The client must rest until his or her strength returns. The client requires high carbohydrates and calories for healing; complex carbohydrates are not preferred over simple ones. Dairy products do not cause a problem.

A nurse prepares to assess the emotional state of a client with end-stage pancreatic cancer. Which action should the nurse take first? a. Bring the client to a quiet room for privacy. b. Pull up a chair and sit next to the client's bed. c. Determine whether the client feels like talking about his or her feelings. d. Review the health care provider's notes about the prognosis for the client.

ANS: C Before conducting an assessment about the client's feelings, the nurse should determine whether he or she is willing and able to talk about them. If the client is open to the conversation and his or her room is not appropriate, an alternative meeting space may be located. The nurse should be present for the client during this time, and pulling up a chair and sitting with the client indicates that presence. Because the nurse is assessing the client's response to a terminal diagnosis, it is not necessary to have detailed information about the projected prognosis; the nurse knows that the client is facing an end-of-life illness.

After teaching a client who is prescribed pancreatic enzyme replacement therapy, the nurse assesses the client's understanding. Which statement made by the client indicates a need for additional teaching? a. "The capsules can be opened and the powder sprinkled on applesauce if needed." b. "I will wipe my lips carefully after I drink the enzyme preparation." c. "The best time to take the enzymes is immediately after I have a meal or a snack." d. "I will not mix the enzyme powder with food or liquids that contain protein."

ANS: C The enzymes should be taken immediately before eating meals or snacks. If the client cannot swallow the capsules whole, they can be opened up and the powder sprinkled on applesauce, mashed fruit, or rice cereal. The client should wipe his or her lips carefully after drinking the enzyme preparation because the liquid could damage the skin. Protein items will be dissolved by the enzymes if they are mixed together.

A nurse assessing a client with colorectal cancer auscultates high-pitched bowel sounds and notes the presence of visible peristaltic waves. Which action should the nurse take? a. Ask if the client is experiencing pain in the right shoulder. b. Perform a rectal examination and assess for polyps. c. Contact the provider and recommend computed tomography. d. Administer a laxative to increase bowel movement activity.

ANS: C The presence of visible peristaltic waves, accompanied by high-pitched or tingling bowel sounds, is indicative of partial obstruction caused by the tumor. The nurse should contact the provider with these results and recommend a computed tomography scan for further diagnostic testing. This assessment finding is not associated with right shoulder pain; peritonitis and cholecystitis are associated with referred pain to the right shoulder. The registered nurse is not qualified to complete a rectal examination for polyps, and laxatives would not help this client.

While at a public park, a nurse encounters a person immediately after a bee sting. The persons lips are swollen, and wheezes are audible. Which action should the nurse take first? a. Elevate the site and notify the persons next of kin. b. Remove the stinger with tweezers and encourage rest. c. Administer diphenhydramine (Benadryl) and apply ice. d. Administer an EpiPen from the first aid kit and call 911.

ANS: D The clients swollen lips indicate that anaphylaxis may be developing, and this is a medical emergency. 911 should be called immediately, and the client transported to the emergency department as quickly as possible. If an EpiPen is available, it should be administered at the first sign of an anaphylactic reaction. The other answers do not provide adequate interventions to treat airway obstruction due to anaphylaxis.

After teaching a client who has a history of cholelithiasis, the nurse assesses the client's understanding. Which menu selection made by the client indicates the client clearly understands the dietary teaching? a. Lasagna, tossed salad with Italian dressing, and low-fat milk b. Grilled cheese sandwich, tomato soup, and coffee with cream c. Cream of potato soup, Caesar salad with chicken, and a diet cola d. Roasted chicken breast, baked potato with chives, and orange juice

ANS: D Clients with cholelithiasis should avoid foods high in fat and cholesterol, such as whole milk, butter, and fried foods. Lasagna, low-fat milk, grilled cheese, cream, and cream of potato soup all have high levels of fat. The meal with the least amount of fat is the chicken breast dinner.

ANS: B The nurse should provide both approval and room for improvement in feedback after a teaching session. Feedback should be objective and constructive, and not evaluative. Reassuring the client that things will improve does not offer anything concrete for the client to work on, nor does it let him or her know what was done well. The nurse should not make the client convey learning needs because the client may not know what else he or she needs to understand. The client needs to become the expert in self-management of the ostomy, and the nurse should not offer to teach the daughter instead of the client.

After teaching a client who has a new colostomy, the nurse provides feedback based on the client's ability to complete self-care activities. Which statement should the nurse include in this feedback? a. "I realize that you had a tough time today, but it will get easier with practice." b. "You cleaned the stoma well. Now you need to practice putting on the appliance." c. "You seem to understand what I taught you today. What else can I help you with?" d. "You seem uncomfortable. Do you want your daughter to care for your ostomy?"

ANS: C Laxatives are not recommended for clients with diverticulitis because they can increase pressure in the bowel, causing additional outpouching of the lumen. Exercise and a high-fiber diet are recommended for clients with diverticulitis because they promote regular bowel function. Using the leg muscles rather than the back for lifting prevents abdominal straining.

After teaching a client who has diverticulitis, a nurse assesses the client's understanding. Which statement made by the client indicates a need for additional teaching? a. "I'll ride my bike or take a long walk at least three times a week." b. "I must try to include at least 25 grams of fiber in my diet every day." c. "I will take a laxative nightly at bedtime to avoid becoming constipated." d. "I should use my legs rather than my back muscles when I lift heavy objects."

ANS: B Ciprofloxacin should be taken for 10 to 14 days to treat Salmonella infection, and should not be stopped once the diarrhea has cleared. Clients should be advised to take the entire course of medication. People with Salmonella should not prepare foods for others because the infection may be spread in this way. Hands should be washed with antibacterial soap before and after eating to prevent spread of the bacteria. Dishes and eating utensils should not be shared and should be cleaned thoroughly. Clients can be carriers for up to 1 year.

After teaching a client who was hospitalized for Salmonella food poisoning, a nurse assesses the client's understanding. Which statement made by the client indicates a need for additional teaching? a. "I will let my husband do all of the cooking for my family." b. "I'll take the ciprofloxacin until the diarrhea has resolved." c. "I should wash my hands with antibacterial soap before each meal." d. "I must place my dishes into the dishwasher after each meal."

ANS: B, C, D Parasitic infections can be transmitted to other people. The client himself or herself should keep the toilet area clean instead of possibly exposing another person to the disease. Parasites are transmitted via unclean water sources and sexual practices with rectal contact. The client should test his or her well water and ask sexual partners to have their stool examined for parasites. Raw vegetables are not associated with parasitic gastrointestinal infections. The client can eat vegetables from the store or a home garden as long as the water source is clean.

After teaching a client with a parasitic gastrointestinal infection, a nurse assesses the client's understanding. Which statements made by the client indicate that the client correctly understands the teaching? (Select all that apply.) a. "I'll have my housekeeper keep my toilet clean." b. "I must take a shower or bathe every day." c. "I should have my well water tested." d. "I will ask my sexual partner to have a stool test." e. "I must only eat raw vegetables from my own garden."

ANS: A, C, D Taking warm sitz baths each day, using bulk-producing agents, and administering anti-inflammatory suppositories are all appropriate actions for the client with an anal fissure. The client should not use enemas or laxatives to promote elimination, but rather should rely on bulk-producing agents such as psyllium hydrophilic mucilloid (Metamucil).

After teaching a client with an anal fissure, a nurse assesses the client's understanding. Which client actions indicate that the client correctly understands the teaching? (Select all that apply.) a. Taking a warm sitz bath several times each day b. Utilizing a daily enema to prevent constipation c. Using bulk-producing agents to aid elimination d. Self-administering anti-inflammatory suppositories e. Taking a laxative each morning

ANS: B Toilet paper can irritate the sensitive perineal skin, so warm water rinses or soft cotton washcloths should be used instead. Although aloe vera may facilitate healing of superficial abrasions, it is not an effective skin barrier for diarrhea. Skin barriers such as zinc oxide and vitamin A and D ointment help protect the rectal area from the excoriating effects of liquid stools. Patting the skin is recommended instead of rubbing the skin dry.

After teaching a client with perineal excoriation caused by diarrhea from acute gastroenteritis, a nurse assesses the client's understanding. Which statement by the client indicates a need for additional teaching? a. "I'll rinse my rectal area with warm water after each stool and apply zinc oxide ointment." b. "I will clean my rectal area thoroughly with toilet paper after each stool and then apply aloe vera gel." c. "I must take a sitz bath three times a day and then pat my rectal area gently but thoroughly to make sure I am dry." d. "I shall clean my rectal area with a soft cotton washcloth and then apply vitamin A and D ointment."

A nurse cares for a client who is scheduled for a paracentesis, Which intervention should the nurse delegate to an unlicensed assistive personnel (UAP)? a. Have the client sign the informed consent form. b. Assist the client to void before the procedure. c. Help the client lie flat in bed on the right side. d. Get the client into a chair after the procedure.

B

After teaching a client who has alcohol-induced cirrhosis, a nurse assesses the clients understanding. Which statement made by the client indicates a need for additional teaching? a. I cannot drink any alcohol at all anymore. b. I need to avoid protein in my diet. c. I should not take over-the-counter medications. d. I should eat small, frequent, balanced meals.

B


Related study sets

Geology Lesson 9b: Absolute and Geologic Time

View Set

Nursing Exam 4-Chapter 24 Questions

View Set